You are on page 1of 322

2021 – Feb – medicine

Salah Lashin !
0991907360
1. Knee-jerk reflex is absent in a person. Where is the Reflexes
spinal cord damaged in this case?
A. L3-L4 segments 1. Knee-jerk / Ankle-jerk / myotatic reflexes:
B. C5-C7 segments 1. Function: one muscle contract and the opposite
C. Th7-Th8 segments muscle relax
D. Th9-Th10 segments 2. Spinal nerve origin: L2, L3 and L4.
E. Ll-L2 segments 3. Check for
1. Muscle sensory (Muscle spindles / Strech
receptor (proprioceptor).
2. Conduction of afferent and efferent
neurons.
4. Note: sensation of muscles is called
proprioception and conducted through dorsal
tracts.
2. Diving reflex: Reflex apnea
3. Viscero-motor reflexes:
1. Definition: Between organ and and muscles.
2. Example: peptic ulcer and change in position
4. Viscerovisceral reflex
1. Definition: Between organ and organ.
2. Example: peptic ulcer and heart pain
Salah Lashin ! 5. Aschner reflex: press on the eye to decrease heart rate
0991907360 6. Fatigue reflex: Nerve centres.
Reflexes
1. Knee-jerk / Ankle-jerk / myotatic reflexes:
1. Function: one muscle contract and the opposite
muscle relax
2. Spinal nerve origin: L2, L3 and L4.
3. Check for
1. Muscle sensory (Muscle spindles / Strech
receptor (proprioceptor).
2. Conduction of afferent and efferent
neurons.
4. Note: sensation of muscles is called
proprioception and conducted through dorsal
tracts.
2. Diving reflex: Reflex apnea
3. Viscero-motor reflexes:
1. Definition: Between organ and and muscles.
2. Example: peptic ulcer and change in position
4. Viscerovisceral reflex
1. Definition: Between organ and organ.
2. Example: peptic ulcer and heart pain
Salah Lashin ! 5. Aschner reflex: press on the eye to decrease heart rate
0991907360 6. Fatigue reflex: Nerve centres.
Reflexes
The right leg of a 40-year-old woman measured at the
shin level is by 2 cm smaller in the diameter than the left 1. Knee-jerk / Ankle-jerk / myotatic reflexes:
leg. Ankle- jerk (Achilles) and knee-jerk reflexes 1. Function: one muscle contract and the opposite
are absent on the right. What is the most likely muscle relax
mechanism of hyporeflexia development during 2. Spinal nerve origin: L2, L3 and L4.
peripheral paralysis? 3. Check for
A. Disturbed conduction of stimulation 1. Muscle sensory (Muscle spindles / Strech
B. Inhibition of pyramidal motoneuron receptor (proprioceptor).
C. Disturbed synaptic impulse transmission 2. Conduction of afferent and efferent
D. Activation of excitatory impulses from the CNS neurons.
E. Disturbed perception of stimulation 4. Note: sensation of muscles is called
proprioception and conducted through dorsal
During examination a neurologist taps the tendon under tracts.
the patient’s kneecap with a reflex hammer to evaluate 2. Diving reflex: Reflex apnea
reflex extension of the knee. This response is provoked 3. Viscero-motor reflexes:
by stimulation of the following receptors: 1. Definition: Between organ and and muscles.
A. Muscle spindles 2. Example: peptic ulcer and change in position
B. Golgi tendon organ 4. Viscerovisceral reflex
C. Articular receptors 1. Definition: Between organ and organ.
D. Tactile receptors 2. Example: peptic ulcer and heart pain
E. Nociceptors 5. Aschner reflex: press on the eye to decrease heart rate
Salah Lashin !
0991907360 6. Fatigue reflex: Nerve centres.
Reflexes
1. Knee-jerk / Ankle-jerk / myotatic reflexes:
During an experiment the myotatic reflex has been 1. Function: one muscle contract and the opposite
studied in frogs. After extension in a skeletal muscle muscle relax
its reflectory contraction was absent. The reason for it 2. Spinal nerve origin: L2, L3 and L4.
might be a dysfunction of the following receptors: 3. Check for
A. Muscle spindles 1. Muscle sensory (Muscle spindles / Strech
B. Nociceptors receptor (proprioceptor).
C. Articular 2. Conduction of afferent and efferent
D. Golgi tendon organs neurons.
E. Tactile 4. Note: sensation of muscles is called
proprioception and conducted through dorsal
Knee-jerk reflex is absent in a person. Where is the tracts.
spinal cord damaged in this case? 2. Diving reflex: Reflex apnea
A. L3-L4 segments 3. Viscero-motor reflexes:
B. C5-C7 segments 1. Definition: Between organ and and muscles.
C. Th7-Th8 segments 2. Example: peptic ulcer and change in position
D. Th9-Th10 segments 4. Viscerovisceral reflex
E. Ll-L2 segments 1. Definition: Between organ and organ.
2. Example: peptic ulcer and heart pain
Salah Lashin ! 5. Aschner reflex: press on the eye to decrease heart rate
0991907360 6. Fatigue reflex: Nerve centres.
Spinal positions:

1. Kidney location:
• XI thoracic and III lumbar
• The 12th rib bisects the kidney in the middle
2. Phrenic / diphragmatic nerve:
• Orgin: C3 – C4 – C5.
• Function: Innervates the diaphragm (‫)اﻟﺣﺟﺎب اﻟﺣﺎﺟز‬,
so it’s responsible for breathing.
3. Sympathetic cardiac nerves:
• Origin: Th1-Th5
• Function: Sympathetic activity of the heart.
4. CSF sample:
• Origin: L3 - L4
• Function: get Sample from CSF to check for
meningitis.
5. Carotid artery:
1. Tubercle on the transverse process of the C6
vertebra
6. Urethral sphincter: S2 − S4
7. Knee-jerk reflex: L3-L4 segments.
Salah Lashin !
0991907360
2. When eating food that contains gluten protein, a person
can develop celiac disease (gluten disease). The following
signs are characteristic for this condition: the intestinal villi Gluten disease (Celiac disease)
degenerate and lose their absorptive function, while the Immune disease triggered by eating gluten in wheat which
patient develops diarrhea, steatorrhea, abdominal results in damage (degeneration) in the villi of mucosa.
distention, weight loss, and other extraintestinal signs.
Gluten is a protein of:
A. Corn
B. Wheat
C. Eggs
D. Wild strawberries
E. Rice

Salah Lashin !
0991907360
3. A young man came to a hospital with complaints of
disturbed urination. Examination of his external genitalia
revealed the urethra to be split on the top, with urine
flowing out of this opening. What type of external
genitalia maldevelopment is observed in this case?
A. Phimosis
B. Epispadia
C. Hermaphroditism
D. Hypospadias
E. Paraphimosis

Related
On examination of a newborn boy’s external
genitalia a fissure in the urethra opening on the
inferior surface of his penis is detected. What
maldevelopment is it?
A. Hypospadias
B. Hermaphroditism
C. Epispadia
Salah Lashin ! D. Monorchism
0991907360 E. Cryptorchidism
4. An older woman experienced a severe stress. Her Monoamine oxidase (MAO) Enzyme
blood levels of adrenaline and noradrenaline sharply
increased. What enzymes catalyze the process of • Function: Kills the exciting hormones: noradrenaline,
catecholamine inactivation? adrenaline, dopamine. So, this enzyme causes depression.
A. Glycosidase • MOA: Oxidative deamination.
B. Peptidase
C. Carboxylase MAO inhibitors Antidepressants)
D. Tyrosinase • MOA: Inhibits Monoamine oxidase =
E. Monoamine oxidase • No inhibition for catecholamines =
• Increased concentration "

Salah Lashin !
0991907360
Depressions and emotional disorders result from Monoamine oxidase (MAO) Enzyme
noradrenaline, serotonin and other biogenic amines
deficiency in brain. Concentration of these compounds • Function: Kills the exciting hormones: noradrenaline,
in synapses can be increased by means of adrenaline, dopamine. So, this enzyme causes depression.
antidepressants that inhibit the activity of the following • MOA: Oxidative deamination.
enzyme:
A. Monoamine oxidase Antidepressants (MAO inhibitors)
B. Diamine oxidase • MOA: Inhibits Monoamine oxidase =
C. L-amino acid oxidase • No inhibition for catecholamines =
D. D-amino acid oxidase • Increased concentration "
E. Phenylalanine-4-monooxigenase

A 50-year-old man experienced a severe stress. His


blood levels of adrenaline and noradrenaline sharply Monoamine oxidase inhibitors are widely used in clinics as
increased. What enzyme catalyze the process of psychopharmacological drugs. They change the level of the
noradrenaline inactivation? following neurotransmitter in the synapses:
A. Tyrosinase A. ATP
B. Glycosidases B. Acetylcholine
C. Carboxylases C. Norepinephrine
D. Monoamine oxidases D. Substance P
E. Salah Lashin !
Peptidases E. L-glutamate
0991907360
Heart failure drugs

Acute heart failure drugs


1. Glycoside:
• Corglycon.
• Strophanthin
2. Non-glycoside
• Dobutamine

Chronic heart failure drugs


• Digoxin (Glycoside):
• Uses: Chronic heart failure & Anti-
arrhythmia.
• Note:
• Hypokalemia or hypercalcemia will
cause toxicity.
• Drugs that causes hypokalemia are
furosemide and thiazides.
• Drugs that causes hypercalcemia are
thiazide diuretics.
Salah Lashin !
0991907360 • Antidotes: Unithiol (sulfa)
5. A patient with chronic cardiovascular failure was taking Acute heart failure drugs
digoxin. After the patient was prescribed an additional 1. Glycoside:
therapy, he developed signs of cardiac glycoside
• Corglycon.
intoxication. What drug can accelerate the process of
• Strophanthin
cardiac glycoside intoxication?
A. Potassium chloride 2. Non-glycoside
B. Magnesium chloride • Dobutamine
C. Glucose solution
D. Calcium chloride Chronic heart failure drugs
E. Asparcam (potassium aspartate and magnesium • Digoxin (Glycoside):
aspartate) • Uses: Chronic heart failure & Anti-
arrhythmia.
• Note:
• Hypokalemia or hypercalcemia will
cause toxicity.
• Drugs that causes hypokalemia are
furosemide and thiazides.
• Drugs that causes hypercalcemia are
thiazide diuretics.
• drugs that induce
Salah Lashin ! • Antidotes: Unithiol (sulfa)
0991907360
Respiratory terms
1. Perfusiton: suppling lungs with blood (V).
2. Ventilation: suppling lungs with gases (breathing) Q.
3. Diffusion: is the moving of gases between alveoli and
blood across the wall.

• Note: Apical alveoli have increased ventilation over


perfusion.

Salah Lashin !
0991907360
6. Examination of a person standing upright shows that Respiratory terms
oxygen partial pressure in the apical alveoli of the lungs is 1. Perfusiton: suppling lungs with blood (V).
140 mm Hg, because in these lung segments: 2. Ventilation: suppling lungs with gases (breathing) Q.
A. Ventilation exceeds perfusion 3. Diffusion: is the moving of gases between alveoli and
B. No ventilation occurs blood across the wall.
C. Perfusion exceeds ventilation
D. Perfusion and ventilation are balanced • Note: Apical alveoli have increased ventilation over
E. - perfusion.

Salah Lashin !
0991907360
Respiratory terms
1. Perfusiton: suppling lungs with blood (V).
2. Ventilation: suppling lungs with gases (breathing) Q.
3. Diffusion: is the moving of gases between alveoli and
blood across the wall.

Related • Note: Apical alveoli have increased ventilation over


A patient has increased thickness of alveolar-capillary perfusion.
membrane caused by a pathologic process. The direct
consequence will be reduction of the following value:
A. Diffusing lung capacity
B. Oxygen capacity of blood
C. Respiratory minute volume
D. Alveolar ventilation of lungs
E. Expiratory reserve volume

Salah Lashin !
0991907360
7. Clinical course of urolithiasis was complicated by the
passage of a renal calculus. Where in the ureter is it most
likely to stop?
A. In the middle of the abdominal segment
B. In the renal pelvis
C. At the border between the abdominal and pelvic
segments
D. 5 cm above the pelvic segment
E. 2 cm above the entrance to the urinary bladder

Related
It is necessary to perform urinary bladder catheterization of an
adult man. Resistance to the catheter can occur in the following
structure or part of the urethra:
A. Membranous part
B. Prostatic part
C. Spongiose part
Salah Lashin ! D. External urethral orifice
0991907360 E. Internal urethral orifice
8. A 12-year-old boy presents with high levels of all Amino acid metabolism disorders
aliphatic amino acids in his urine, with cystine and cysteine
excretion levels being the highest. Additionally, renal 1. Maple syrup urine disease:
ultrasound detected kidney stones. What is the most likely 1. Defect enzyme: Branched-chain ketoacid
pathology in this case? dehydrogenase
2. Pathogenesis: Valine, isoleucine and Leucine
A. Hartnup disease
accumulate = Mental retardation + Maple syrup urine
B. Cystinuria disease.
C. Phenylketonuria 2. Phenylketonuria:
D. Urolithiasis 1. Defect enzyme: Phenylalanine hydroxylase.
E. Alkaptonuria 2. Pathogenesis: Phenylalanine accumulates in brain
causing mental retardation.
3. Alkaptonuria:
1. Defect enzyme: Homogentisic acid oxidase.
2. Pathogenesis: Homogentisic acid accumulate:
1. In joints and connective tissue Ochronosis and
arthritis
2. In urine = Dark urine color.
4. Cystinuria:
• High concentration cystine and cysteine = Cystinuria.
• Leads to formation of kidney stones.

Salah Lashin !
0991907360
Amino acid metabolism disorders

1. Maple syrup urine disease:


1. Defect enzyme: Branched-chain ketoacid
dehydrogenase
2. Pathogenesis: Valine, isoleucine and Leucine
accumulate = Mental retardation + Maple syrup urine
disease.
2. Phenylketonuria:
1. Defect enzyme: Phenylalanine hydroxylase.
2. Pathogenesis: Phenylalanine accumulates in brain
causing mental retardation.
3. Alkaptonuria:
1. Defect enzyme: Homogentisic acid oxidase.
2. Pathogenesis: Homogentisic acid accumulate:
1. In joints and connective tissue Ochronosis and
arthritis
2. In urine = Dark urine color.
4. Cystinuria:
• High concentration cystine and cysteine = Cystinuria.
• Leads to formation of kidney stones.

Salah Lashin !
0991907360
9. The medicines that inhibit blood clotting
(anticoagulants) are used for prevention and treatment of Anticoagulants
thrombosis. What anticoagulant can be neutralized by Anti:
protamine sulfate as its antagonist, in case of overdosage? (platelet + clotting factors + fibrin)
A. Sodium hydrocitrate to prevent thrombosis.
B. Phenylin (Phenindione) Use: varicose veins, myocardial infarction, angina
C. Neodicoumarin (Ethyl biscoumacetate)
D. Heparin
E. Syncumar (Acenocoumarol)

Salah Lashin !
0991907360
Sample of the patient’s blood was taken for analysis in A patient with myocardium infarction was admitted to the
the presence of heparin. By its chemical structure, this resuscitation department. What drug should be injected to
anticoagulant belongs to: the patient in order to prevent thrombosis?
A. Triacylglycerols A. Heparin
B. Phospholipids B. Chingamin
C. Glycosaminoglycans C. Thyroxine
D. Hemoproteins D. Biseptol-480
E. Simple proteins E. Dimedrol

Various substances can be used as anticoagulants. After implantation of a cardiac valve a young man
Among them there is a certain naturally derived constantly takes indirect anticoagulants. His state was
polysaccharide. Name this polysaccharide: complicated by hemorrhage. What substance content has
A. Chondroitin sulfate decreased in blood?
B. Dermatan sulfate A. Prothrombin
C. Dextran B. Haptoglobin
D. Heparin C. Heparin
E. Hyaluronic acid D. Creatin
E. Ceruloplasmin
Salah Lashin !
0991907360
Review Anticoagulants
A patient with myocardial infarction in the acute Anti:
phase has been hospitalized into the cardiology unit. (platelet + clotting factors + fibrin)
To induce platelet lysis in the patient’s coronary to prevent thrombosis.
vessels during the early hours of infarction, the Use: varicose veins, myocardial infarction, angina
following enzyme should be used:
A. Lysozyme
B. Hyaluronidase
C. Streptokinase
D. Trypsin
E. Chymotrypsin

A 66 - year - old patient diagnosed with acute A patient with myocardial infarction in the acute phase
myocardial infraction was prescribed has been hospitalized into the cardiology unit. To induce
pharmacological preparation of streptokinase for platelet lysis in the patient’s coronary vessels during the
thrombolysis. what glycoprotein is activated in the early hours of infarction, the following enzyme should be
process? used:
A. Antithrombin A. Lysozyme
B. α1- antitrypsin B. Hyaluronidase
C. Thromboplastin C. Streptokinase
D. Salah Lashin !
Plasminogen D. Trypsin
E. 0991907360
α2 macroglobulin E. Chymotrypsin
10 . A person has increased pulmonary ventilation due to Lung volumes and capacities !
physical exertion. What indicator of external respiration 1. Tidal volume:
will be significantly increased compared to the resting 1. Use: quite respiration
2. Muslces: Diaphragm.
state?
2. Inspiratory reserve volume:
A. Total lung capacity 1. Use: Deep insiparion.
B. Expiratory reserve volume 2. Muscles:
C. Respiratory volume 1. Greater pectoralis
D. Inspiratory reserve volume 2. Sternocleidomastoid muscle
E. Vital lung capacity 3. Vital capacity: (blowing ballon)
1. Use: Deep expiration:
2. Muscles:
• Abdominal muscles.
4. Residual capacity:
1. Definition: the volume stayed in thel lung and can’t be
mesured
2. Note: Functional residual capacity also can’t be meaured
because it contains resuidual capicity.
5. Respiratory volume / Minute volume:
1. Definition: volume of gas inhaled or exhaled from a person's
lungs per minute.
2. Value: RR X Volume.
3. Indications:
1. Higher during physical activity (High RR).
Salah Lashin ! 2. Reduced during apnea. (Low RR).
0991907360
Lung volumes and capaci1es !

Salah Lashin !
0991907360
Spirogram analysis shows decreased depth and
frequency of respirations in the person, which in turn Terminology
leads to decrease in the: Respiratory rate / frequency ‫ﻣﻌدل اﻟﺗﻧﻔس‬
A. Vital capacity of lungs -pnea ‫ﺗﻧﻔس‬
B. Inspiratory reserve volume
C. Residual volume
D. Respiratory minute volume Respiratory volume / Minute volume
E. Expiratory reserve volume 1. Definition: volume of gas inhaled or exhaled from a
person's lungs per minute.
A person developed increased pulmonary ventilation due 2. Value: RR X Volume.
to physical exertion. What indicator of external 1. Higher during physical activity (High RR)
respiration will be significantly increased compared to 2. Reduced during apnea. (Low RR)
the resting state?
A. Vital lung capacity
B. Expiratory reserve volume
C. Total lung capacity
D. Respiratory volume
E. Inspiratory reserve volume

Salah Lashin !
0991907360
Lung ventilation in a person is increased as a result of Terminology
physical activity. Which of the following indices of the Respiratory rate / frequency ‫ﻣﻌدل اﻟﺗﻧﻔس‬
external respiration is much higher than in a state of rest? -pnea ‫ﺗﻧﻔس‬
A. Respiratory volume
B. Vital capacity of lungs
C. Inspiratory reserve volume
D. Expiratory reserve volume Respiratory volume / Minute volume
E. Total lung capacity 1. Definition: volume of gas inhaled or exhaled from a
person's lungs per minute.
A doctor recorded in the medical history that a patient had 2. Value: RR X Volume.
hypopnoe (reduced respiration depth). It means that the 1. Higher during physical activity (High RR)
following characteristic of external respiration is reduced: 2. Reduced during apnea. (Low RR)
A. Respiratory volume
B. Vital lung capacity
C. Functional residual capacity
D. Expiration capacity
E. Respiratory minute volume

Salah Lashin !
0991907360
Lung volumes and capacities !
1. Tidal volume:
1. Use: quite respiration
2. Muslces: Diaphragm.
2. Inspiratory reserve volume:
1. Use: Deep insiparion.
2. Muscles:
1. Greater pectoralis
2. Sternocleidomastoid muscle
3. Vital capacity: (blowing ballon)
1. Use: Deep expiration:
2. Muscles:
• Abdominal muscles.
4. Residual capacity:
1. Definition: the volume stayed in thel lung and can’t be
mesured
2. Note: Functional residual capacity also can’t be meaured
because it contains resuidual capicity.
5. Respiratory volume / Minute volume:
1. Definition: volume of gas inhaled or exhaled from a person's
lungs per minute.
2. Value: RR X Volume.
3. Indications:
1. Higher during physical activity (High RR).
2. Reduced during apnea. (Low RR).
Salah Lashin !
0991907360
Lung volumes and capacities !
1. Tidal volume:
During training session in the laboratory the students 1. Use: quite respiration
were performing spirography on themselves. What 2. Muslces: Diaphragm.
indicator CANNOT be measured with this method? 2. Inspiratory reserve volume:
A. Respiration rate 1. Use: Deep insiparion.
B. Vital capacity 2. Muscles:
C. Respiratory minute volume 1. Greater pectoralis
D. Functional residual capacity 2. Sternocleidomastoid muscle
3. Vital capacity: (blowing ballon)
E. Maximal breathing capacity
1. Use: Deep expiration:
2. Muscles:
• Abdominal muscles.
4. Residual capacity:
1. Definition: the volume stayed in thel lung and can’t be
mesured
2. Note: Functional residual capacity also can’t be meaured
because it contains resuidual capicity.
5. Respiratory volume / Minute volume:
1. Definition: volume of gas inhaled or exhaled from a person's
lungs per minute.
2. Value: RR X Volume.
3. Indications:
1. Higher during physical activity (High RR).
2. Reduced during apnea. (Low RR).
Salah Lashin !
0991907360
Lung volumes and capacities !
A patient has sustained a traumatic injury of the greater 1. Tidal volume:
pectoral muscle. This resulted in a decrease of: 1. Use: quite respiration
A. Inspiratory reserve volume 2. Muslces: Diaphragm.
B. Expiratory reserve volume 2. Inspiratory reserve volume:
C. Tidal volume 1. Use: Deep insiparion.
D. Residual volume 2. Muscles:
E. Functional residual lung capacity 1. Greater pectoralis
2. Sternocleidomastoid muscle
3. Vital capacity: (blowing ballon)
1. Use: Deep expiration:
2. Muscles:
1. Abdominal muscles.
4. Residual capacity:
1. Definition: the volume stayed in thel lung and can’t be
mesured
2. Note: Functional residual capacity also can’t be meaured
because it contains resuidual capicity.
5. Respiratory volume / Minute volume:
1. Definition: volume of gas inhaled or exhaled from a person's
lungs per minute.
2. Value: RR X Volume.
3. Indications:
1. Higher during physical activity (High RR).
2. Reduced during apnea. (Low RR).
Salah Lashin !
0991907360
Lung volumes and capacities !
A patient has a trauma of sternoclei-domastoid muscle. 1. Tidal volume:
This caused a decrease in value of the following indicator 1. Use: quite respiration
of external respiration: 2. Muslces: Diaphragm.
A. Inspiratory reserve volume 2. Inspiratory reserve volume:
B. Expiratory reserve volume 1. Use: Deep insiparion.
C. Respiratory capacity 2. Muscles:
D. Residual volume 1. Greater pectoralis
E. Functional residual lung capacity 2. Sternocleidomastoid muscle
3. Vital capacity: (blowing ballon)
1. Use: Deep expiration:
2. Muscles:
1. Abdominal muscles.
4. Residual capacity:
1. Definition: the volume stayed in thel lung and can’t be
mesured
2. Note: Functional residual capacity also can’t be meaured
because it contains resuidual capicity.
5. Respiratory volume / Minute volume:
1. Definition: volume of gas inhaled or exhaled from a person's
lungs per minute.
2. Value: RR X Volume.
3. Indications:
1. Higher during physical activity (High RR).
2. Reduced during apnea. (Low RR).
Salah Lashin !
0991907360
Lung volumes and capacities !
A child asked you to blow a balloon as much as you can 1. Tidal volume:
in one exhale. What air volume will you use for this 1. Use: quite respiration
purpose? A. Total lung capacity 2. Muslces: Diaphragm.
B. Functional residual lung capacity 2. Inspiratory reserve volume:
C. Inspiratory reserve volume 1. Use: Deep insiparion.
D. Inspiratory capacity 2. Muscles:
E. Vital capacity 1. Greater pectoralis
2. Sternocleidomastoid muscle
3. Vital capacity: (blowing ballon)
1. Use: Deep expiration:
2. Muscles:
• Abdominal muscles.
4. Residual capacity:
1. Definition: the volume stayed in thel lung and can’t be
mesured
2. Note: Functional residual capacity also can’t be meaured
because it contains resuidual capicity.
5. Respiratory volume / Minute volume:
1. Definition: volume of gas inhaled or exhaled from a person's
lungs per minute.
2. Value: RR X Volume.
3. Indications:
1. Higher during physical activity (High RR).
2. Reduced during apnea. (Low RR).
Salah Lashin !
0991907360
11 . A patient has crystalline lens dislocation and spidery
fingers. What syndrome can be diagnosed, considering that Marfan syndrome
the patient also has disturbed development of connective Mutation of the fibrillin gene which is a glycoprotein that
tissue and deformed hands and feet? functions as a support for elastic fibers.
A. Turner syndrome
B. Down syndrome
C. Klinefelter syndrome
D. Trisomy X
E. Marfan syndrome

spidery fingers

Salah Lashin !
0991907360
12 . To treat urticaria and remove the itching skin rash, a
patient was prescribed dimedrol (diphenhydramine). What
mechanism of action ensures that this drug is effective in
such cases?
A. Suppression of histamine release
B. Inhibition of histamine synthesis
C. Competitive H1 receptor blockade
D. Independent antagonism with histamine
E. Acceleration of histamine breakdown Antihistamine
Mast cells blocker:
1. Ketotifen
2. Cromolyn sodium

H1 blockers (allergy):
1. First generation: Dimedrol / Benadryl / (diphenhydramine)
• Use: has effect on the CNS so it’s usefull if the patient
needs to sleep.
2. Second generation: Loratadine:
• Use: No or less CNS effect, so it’s usefull if the patient
needs to stay awake (Train station worker, student).
H2 blockers (to treat ulcer):
Famous Run 2 ulcer !
1. Famotidine
Salah Lashin !
2. Ranitidine
0991907360
13 . Some people have a hereditary disease with a complex
of symptoms that include hepatic cirrhosis and dystrophic Wilson’s disease
processes in the brain. This disease is accompanied by 1. Due to: Deficiency of Ceruloplasmin (Copper transport).
decreasing plasma ceruloplasmin levels and disturbed 2. Accumulation of coper in the liver.
copper metabolism. What disease is it?
A. Tay-Sachs disease Hemosiderosis
B. Niemann-Pick disease 1. Due to: autosomal recessive defect in iron absorption
C. Marfan syndrome (primary) → accumulation of iron in the body → Dense
D. Gilbert syndrome and brown-colored and liver cirrhosis.
E. Wilson disease 2. Note:
1. Transferrin is the iron transporter.
2. Hemosiderosis focal accumulation of iron.
3. Hemochromatosis is systematic accumulatin.

Salah Lashin !
0991907360
Iron is released in the process of hemoglobin catabolism. During autopsy of a man, who had been suffering from mitral
Then, as a part of a special transport protein, it arrives stenosis, the lungs are revealed to be dense and brown-colored.
into the bone marrow and is used again for hemoglobin What pathologic process had occurred in
synthesis. Name this transport protein: the lungs?
A. Transcobalamin A. Hemosiderosis
B. Transferrin B. Hemochromatosis
C. Haptoglobin C. Jaundice
D. Albumin D. Hemomelanosis
E. Ceruloplasmin E. Lipofuscinosis

In the process of hemoglobin catabolism iron is released A 27-year-old patient presents with pathologic changes in the
and then as a part of special transport protein is returned liver and brain. Blood plasma exhibits acute decrease in copper
to the bone marrow, to be used again for levels, while urine copper level is elevated. The patient is
hemoglobin synthesis. Name this transport protein: diagnosed with Wilson’s disease. To confirm this diagnosis, it is
A. Transcobalamin necessary to measure activity of the following enzyme in the
B. Haptoglobin patient’s serum:
C. Transferrin A. Ceruloplasmin
D. Ceruloplasmin B. Leucine aminopeptidase
E. Albumin C. Alcohol dehydrogenase
D. Xanthine oxidase
Salah Lashin ! E. Carbonic anhydrase
0991907360
14 . A person was hospitalized into the intensive care unit
in a state of hypoxia, caused by aspiration of vomitus.
Objectively, the patient's condition is severe; the skin is
moist, pale, and acrocyanotic. Tachypnea, tachycardia, and
low blood pressure are noted. Which of the listed signs of
acutely progressing hypoxia is an emergency defensive and
adaptational reaction of the body?
A. Tachycardia
B. Skin pallor
C. Low blood pressure
D. Acrocyanosis development
E. Increased perspiration

Salah Lashin !
0991907360
15 . A patient with streptococcal pneumonia was prescribed
an antimicrobial drug that disrupts formation of microbial
shell. What drug is it?
A. Doxycycline hydrochloride
B. Erythromycin
C. Benzylpenicillin sodium salt
D. Azithromycin
E. Gentamycin sulfat
Antibiotics #

Salah Lashin !
0991907360
Related

A patient with streptococcal infection of the gingiva A doctor prescribed a cephalosporin antibiotic to the
was prescribed a drug with Beta-lactam ring in its patient after appendectomy for infection prevention.
structure. What drug of those listed below belongs to Antimicrobial activity of this group of antibiotics is based
this pharmacological group? upon the disturbance of the following process:
A. Erythromycin A. Microbial wall formation
B. Rifampicin B. Nucleic acid synthesis
C. Levomycetin (Chloramphenicol) C. Ribosome protein synthesis
D. Benzylpenicillin D. Energy metabolism
E. Streptomycin sulfate E. Choline esterase block

A 29-year-old patient with bacterial pneumonia has What antimicrobial drug IS NOT a cephalosporin
been administered penicillin. What is the mechanism of antibiotic?
its antimicrobial action? A. Cefpirome
A. Inhibition of cell walls synthesis in the B. Cephaloridine
microorganisms C. Cefoxitin
B. Inhibition of intracellular protein synthesis D. Ciprofloxacin
C. Inhibition of cholinesterase activity E. Cefalexin
D. Inhibition of SH-groups of microorganism enzymes
E. Antagonism of PABA
Salah Lashin !
0991907360
Urea / Ornithine cycle

• Function: converte toxic amonium which is very toxic


speacially for the brian into less toxic urea.
• Location: Liver.
• Amonia Transport
1. Muscle: Alanine
2. Intestine: NH3 – portal vein
3. Most other organs: glutamine

• Genetic disorders:
1. Deficiency of Carbamoyl phosphate synthetase I:
• → not forming of Carbamoyl phosphate →
accumulation of ammonia in blood →
hyperammonemia → Symptoms.
2. Deficiency of Ornithine transcarbamoylase:
• → not forming citrulline (joining of Carbamoyl
phosphate + Ornithine) →
• Accumulation of Carbamoyl phosphate ⬆
• Enters pyrimidine synthesis pathway → increase
formation of orotic acid → orotic aciduria.
3. Deficiency of argininosuccinate synthase 1:
• Accumulation of citrulline → Citrullinemia.

• Symptom: lethargy, vomiting, and hyperventilation; and if not


Salah Lashin
treated, ! to hepatic coma, respiratory failure, and death.
progress
• 0991907360
Indicators: decrease BUN (blood urea nitrogen).
There are several ways of ammonia neutralization in the body, with
some organs having their own specific ways. What way of ammonia
neutralization is characteristic of brain cells?
A. Glutamine formation
B. NH4+ formation
C. Creatine formation
D. Asparagine formation
E. Urea formation

During intensive muscle work there is a large amount of ammonia


produced in the muscles. What amino acid plays the main role in the
transportation of ammonia to the liver and participates in
gluconeogenesis reactions?
A. Alanine
B. Arginine
C. Lysine
D. Ornithine
E. Aspartate

Ammonia is extremely toxic for human CNS. What is the main way of
ammonia neutralization in the nervous tissue?
A. Urea synthesis
B. Ammonium salts synthesis
C. Glutamine synthesis
D. Transamination
Salah Lashin !
E. Formation of paired compounds
0991907360
Nitrogen is being excreted from the body mainly as urea.
When activity of a certain enzyme in the liver is low, it
results in inhibition of urea synthesis and
nitrogen accumulation in blood and tissues. Name this
enzyme:
A. Carbamoyl phosphate synthetase
B. Aspartate aminotransferase
C. Urease
D. Amylase
E. Pepsin

A new born presents with weak suckling, frequent


vomiting, and hypotonia. Blood and urine citrulline are
very high. What metabolic process is disturbed?
A. Gluconeogenesis
B. Ornithine cycle
C. Glycolysis
D. Cori cycle
E. Tricarboxylic acid cycle

Salah Lashin !
0991907360
Related
The greater amount of nitrogen is excreted from the organism in
form of urea. Inhibition of urea synthesis and accumulation of
ammonia in blood and tissues are induced by the decreased
activity of the following liver enzyme:
A. Urease
B. Aspartate aminotransferase
C. Carbamoyl phosphate synthetase
D. Amylase
E. Pepsin

Presence of citrulline and high ammonia levels are detected in


the urine of a newborn. This child is likely to present with
disturbed production of the following substance:
A. Creatine
B. Urea
C. Creatinine
D. Uric acid
E. Ammonia
Salah Lashin !
0991907360
Pyrimidine synthesis

Salah Lashin !
0991907360
Review
A patient with hereditary hyperammonemia due to
a disorder of ornithine cycle has developed
secondary orotaciduria. The increased synthesis of
orotic acid is caused by an increase in the
following metabolite of ornithine cycle:
A. Carbamoyl phosphate
B. Citrulline
C. Ornithine
D. Urea
E. Argininosuccinate

Secretion of orotic acid many times exceeds the


norm in patients with hereditary orotic aciduria. Pyrimidine synthesis
This pathology is associated with disturbed
synthesis of:
A. Pyrimidine nucleotides
B. Purine nucleotides
C. Biogenic amines
D. Uric acid
Urea Lashin !
E. Salah
0991907360
16 .A newborn presents with weak suckling , frequent
vomiting and hypotonia. Blood and urine citrulline are
very high. What metabolic process is disturbed?
A. Glycolysis
B. Gluconeogenesis
C. Ornithine cycle
D. Cori cycle
E. Tricarboxylic acid cycle

Salah Lashin !
0991907360
17 .A 25-year-old young man came to the doctor
complaining of general weakness, rapid fatigability,
irritability, reduced working ability, and bleeding gums.
What vitamin is likely to be deficient in this case?
A. Retinol
B. Thiamine Vitamin C (Ascorbic acid)
C. Ascorbic acid Function:
D. Riboflavin 1. Hydroxylation of proline and lysine amino acids of
E. Folic acid collagen.
2. Absorbtion of iron
Deficieny:
1. Scurvy: Defect in collagen = Bleeding gum +
Subcutaneous hemorrhage
2. Iron deficiency.
3. Note: Vitamin K deficiency can also cause bleeding
but different reason = clotting factor disorders.
Supplement:
• Ascorbic acid + Rutin (increase absorption of vitamin
C).

Salah Lashin !
0991907360
Vitamin C (Ascorbic acid)
Function:
1. Hydroxylation of proline and lysine amino acids of
collagen.
2. Absorbtion of iron
Deficieny:
1. Scurvy: Defect in collagen = Bleeding gum +
Subcutaneous hemorrhage
2. Iron deficiency.
3. Note: Vitamin K deficiency can also cause bleeding
but different reason = clotting factor disorders.
Supplement:
• Ascorbic acid + Rutin (increase absorption of vitamin
C).

Salah Lashin !
0991907360
Related

To assess the rate of collagen disintegration during certain connective tissue


disturbances, it is necessary to measure the urine content of the following:
A. Oxyproline
B. Ornithine
C. Proline
D. Lysine
E. Urea

Osteolaterism is characterized by a decrease in collagen strength caused by much


less intensive formation of cross-links in the collagen fibrils. This phenomenon is
caused by hypoactivity of the following enzyme:
A. Lysyl oxidase
B. Monoamino-oxidase
C. Prolyl hydroxylase
D. Lysyl hydroxylase
E. Collagenase

A 34-year-old patient has a history of periodontitis. As a result of increased


collagen degradation, there is a significantly increased urinary excretion of one of
the amino acids. Which one?
A. Hydroxyproline
B. Valine
C. Alanine
D. Glycine
E. Serine
Salah Lashin !
0991907360
Pediatric examination of a 10-year- old child detected numerous
petechiae on the skin, bleeding gums, and low levels of vitamin
C in urine. What process is disturbed in this case?
A. Collagen synthesis
B. Proteoglycan synthesis
C. Proteoglycan breakdown
D. Collagen breakdown
E. Hyaluronidase activation

Hydroxyproline is an important amino acid in the structure of


collagen. This amino acid forms in the result of proline
hydroxylation. What vitamin takes part in the process of proline
hydroxylation?
A. B6
B. B2
C. B1
D. D
E. C

Salah Lashin !
0991907360
Related

To assess the rate of collagen disintegration during certain connective tissue


disturbances, it is necessary to measure the urine content of the following:
A. Oxyproline
B. Ornithine
C. Proline
D. Lysine
E. Urea

Osteolaterism is characterized by a decrease in collagen strength caused by much


less intensive formation of cross-links in the collagen fibrils. This phenomenon is
caused by hypoactivity of the following enzyme:
A. Lysyl oxidase
B. Monoamino-oxidase
C. Prolyl hydroxylase
D. Lysyl hydroxylase
E. Collagenase

A 34-year-old patient has a history of periodontitis. As a result of increased


collagen degradation, there is a significantly increased urinary excretion of one of
the amino acids. Which one?
A. Hydroxyproline
B. Valine
C. Alanine
D. Glycine
E. Serine
Salah Lashin !
0991907360
Pediatric examination of a 10-year- old child detected numerous
petechiae on the skin, bleeding gums, and low levels of vitamin
C in urine. What process is disturbed in this case?
A. Collagen synthesis
B. Proteoglycan synthesis
C. Proteoglycan breakdown
D. Collagen breakdown
E. Hyaluronidase activation

Hydroxyproline is an important amino acid in the structure of


collagen. This amino acid forms in the result of proline
hydroxylation. What vitamin takes part in the process of proline
hydroxylation?
A. B6
B. B2
C. B1
D. D
E. C

Salah Lashin !
0991907360
Anti-ulcer drugs
18 . Decarboxylase activity leads to formation of
biogenic amines. What biogenic amine triggers the
multistage mechanism that regulates the HCI secretion
in the stomach?
A. Histamine
B. GABA
C. Dopamine
D. Serotonin
E. Glutamine

Salah Lashin !
0991907360
Anti-ulcer drugs

Salah Lashin !
0991907360
Related
A 25 year-old man, suffering from peptic ulcer of the
stomach, has been treated with omeprasole. In 3 weeks the
ulcer was healed. What mechanism of action does this
drug produce?
A. Blockade of H+-K+-ANP-ase (the proton pump)
B. Blockade of M-cholinoceptors cells
C. Decrease of release of hydrochloric acid
D. Decrease of pepsin release

A patient suffering from chronic hyperacidic gastritis takes


an antacid drug for heartburn elimination. After its
ingestion the patient feels better but at the same time he
has a sensation of stomach swelling. Which of the
following drugs might be the cause of such side effect?
A. Sodium hydrocarbonate
B. Magnesium oxide
C. Magnesium trisilicate
D. Aluminium hydrooxide
E. Pepsin

Salah Lashin !
0991907360
19 . 14 days after the recovery from tonsillitis ,15-year-old
teenager developed face edema in the morning, high blood Terminology
pressure, and urine resembling "meat slops" 1. Hematuria = blood in urine
Immunohistochemistry of renal biopsy material revealed 2. renal failure = ‫ﻓﺸﻞ ﻛﻠﻮي‬
immune complex deposits on the capillary basement 3. Crescent = ‫ﺷﻜﻞ اﻟﮭﻼل‬
membrane and in the glomerular mesangium. What disease 4. Subacute = ‫ﺷﺒﮫ ﺣﺎد‬
is it?
A. Acute glomerulonephritis
B. Lipoid nephrosis
C. Acute interstitial nephritis
D. Acute pyelonephritis
E. Necrotic nephrosis

Glomerulonephritis
Inflammation in the renal glomeruli

1. Subacute glomerulonephritis:
1. Time: within months (6 months).
2. Morphology: Crescent shape ‫ھﻼل‬
2. Acute glomerulonephritis:
1. Times: days-weeks.
Salah Lashin ! 2. Morphology: Immune complexa
0991907360
20 . It is known that people with genetically Drug effect
determined glucosc-6- phosphate dehydrogenase 1. Material Cumulation: Accumulation of drug for along time =
insufficiency can develop hemolysis of red blood
cells in response to certain antimalarial drugs. Such increased effect (toxicity). Common with digoxin.
atypical reaction to medicines is called: 2. Tolerance / Habituation : decrease of drug response.
A. Tachyphylaxis
B. Tolerance 3. Tachyphylaxis: Sudden tolerance (decrease of drug response).
C. Allergy 4. Idiosyncrasy: abnormal response to the drug due to genetic
D. Idiosyncrasy
E. Sensitization deficiency in the person.
5. Dependence / Addiction : When the patient is addicted abusing
substances like morphine
6. Withdrawal syndrome: The symptoms which start when you
stop (cessation) taking the drug after prolonged use.
7. Incombustibility: Can’t work together.

Salah Lashin !
0991907360
A patient noticed symptoms of approaching attack of Terminology
bronchial asthma and took several tablets one by one at Cumulation : ‫ﺗراﻛم‬
short intervals out of the doctor’s control. Short-term Idio: personal
improvement of his condition came only after taking the Tolerance: (‫ﺗﺣﻣل )اﻧﻌدام اﻟﺗﺄﺛﯾر‬
first two tablets. Next intakes of a drug didn’t improve his Habituation: ‫اﻋﺗﯾﺎد‬
condition. Reduction of the drug effectiveness was caused Diminish: ‫ﯾﺗﻼﺷﻰ – ﯾﺧﺗﻔﻲ‬
by: Tachy: ‫ﺳرﯾﻊ‬
A. Dependence Dependence / addiction: ‫ ادﻣﺎن‬- ‫اﻋﺗﻣﺎد‬
B. Cumulation Euphoria: ‫ﺗﺄﺛﯾر اﻟﻧﺷوة‬
C. Addiction
D. Tachyphylaxis Drug effect
E. Idiosyncrasy 1. Material Cumulation: Accumulation of drug for along time =
increased effect (toxicity). Common with digoxin.
A patient with frequent attacks of stenocardia was
prescribed sustak-forte to be taken one tablet twice a day. 2. Tolerance / Habituation : decrease of drug response.
At first the effect was positive but on the second day
3. Tachyphylaxis: Sudden tolerance (decrease of drug response).
stenocardia attacks resumed. What can explain
inefficiency of the prescribed drug? 4. Idiosyncrasy: abnormal response to the drug due to genetic
A. Sensibilization
B. Cumulation deficiency in the person.
C. Tachyphylaxis 5. Dependence / Addiction : When the patient is addicted abusing
D. Idiosyncrasy
Salah Lashin !
E. Dependence substances like morphine.
0991907360 6. Incombustibility: Can’t work together.
A patient diagnosed with morphinism has been Drug effect
admitted to the narcological department. A doctor 1. Material Cumulation: Accumulation of drug for along time =
noted a decrease in pharmacological activity of
morphine. Repetitive use of a drug may result in increased effect (toxicity). Common with digoxin.
tolerance to its effect, and this phenomenon is called: 2. Tolerance / Habituation : decrease of drug response).
A. Antagonism
B. Cumulation 3. Tachyphylaxis: Sudden tolerance (decrease of drug response).
C. Tachyphylaxis
4. Idiosyncrasy: abnormal response to the drug due to genetic
D. Addiction
E. Allergy deficiency in the person.
5. Dependence / Addiction : When the patient is addicted
A cessation of morphine administration after its long-
term use leads to the development of severe mental, abusing substances like morphine.
neurological, and somatic disorders. Name this
A patient who has been taking a certain drug for a long time
condition:
cannot discontinue the use of it because this causes psychic and
A. Cumulation somatic disfunctions. The syndrome occurring at refraining from
B. Sensitization the use of a drug is called:
A. Abstinence
C. Idiosyncrasy B. Sensitization
D. Withdrawal C. Idiosyncrasy
E. Salah Lashin
Tolerance ! D. Tachyphylaxis
0991907360 E. Cumulation
It is known that individuals with genetically caused
deficiency of glucose- 6-phosphate dehydrogenase may
develop RBC hemolysis in response to the administration
of some antimalarial drugs. Manifestation Manifestation of
adverse reactions to drugs is called: Drug effect
A. Sensibilization
1. Material Cumulation: Accumulation of drug for along time =
B. Allergy
C. Idiosyncrasy increased effect (toxicity). Common with digoxin.
D. Tachyphylaxis
2. Tolerance / Habituation : decrease of drug response).
E. Tolerance
3. Tachyphylaxis: Sudden tolerance (decrease of drug response).
A patient with heart failure and tachycardia has been 4. Idiosyncrasy: abnormal response to the drug due to genetic
prescribed digoxin. After 5 days of taking digoxin the
patient’s heart rate was normalized. After 2 weeks the deficiency in the person.
patient addressed a doctor due to continuous decrease of 5. Dependence / Addiction When the patient is addicted abusing
heart rate down to 52/min. What phenomenon has caused
such changes of heart rate? substances like morphine.
A. Cumulation
6. Incompatibility: Two drugs can’t work together.
B. Idiosyncrasy
C. Tachyphylaxis
D. Tolerance
Salah Lashin !
E. Allergy
0991907360
A patient used an indirect-acting adrenergic agonist to Drug effect
treat rhinitis. After the patient has been putting in the 1. Material Cumulation: Accumulation of drug for along time =
nose drops for several days, the vasoconstrictive effect
of the drug gradually diminished. Name this increased effect (toxicity). Common with digoxin.
phenomenon: 2. Tolerance / Habituation : decrease of drug response).
A. Teratogenicity
B. Idiosyncrasy 3. Tachyphylaxis: Sudden tolerance (decrease of drug response).
C. Tachyphylaxis
4. Idiosyncrasy: abnormal response to the drug due to genetic
D. Allergy
E. Cumulation deficiency in the person.
5. Dependence / Addiction : When the patient is addicted abusing
A man with chronic constipation was taking daily one
tablet of Bisacodyl. After a time, he had to start taking 2 substances like morphine.
tablets daily to achieve the same effect. This type of 6. Incompatibility: two drugs can’t work together.
change in the action of active substances is called:
A. Cumulation A patient has been taking bisacodyl for a long time to treat
B. Habituation chronic constipation. However, several weeks later the aperient
C. Idiosyncrasy effect of the drug diminished. What is the possible cause of this?
D. Drug dependence A. Drug dependence
E. Sensitization B. Sensitization
C. Acquired tolerance
Salah Lashin ! D. Functional cumulation
0991907360 E. Material cumulation
21 . A child underwent a tuberculin skin test (Mantoux TB
test). 48 hours later the child developed a papule reaching Diagnoisis:
10 mm in diameter at the injection site. These changes are 1. Ziehl-Neelsen stain: acid fast bacteria like
caused by the following mechanism of hypersensitivity: tuberculosis.
A. Cellular cytotoxicity 2. Mantux test: Allergy skin test contains tuberculin
B. Granulomatosis and check for tuberculin antibodies.
C. Antibody-dependent cytotoxicity 1. Use: to detect any antibody immunity for
D. Anaphylaxis tuberculosis and the need for BCG vaccine.
E. Immune complex cytotoxicity 2. Response: Cellular / Type 4 hypersensitivity
(Mononuclear cells, T-lymphocytes )
3. Features:
1. Hourseshoe / crescent granuloma.
1. Note: syphilis also causes granuloma but
not horseshoe.

Salah Lashin !
0991907360
Related
TB
A child with suspected tuberculosis was given Mantoux test.
Diagnoisis:
After 24 hours the site of the allergen injection got swollen,
1. Ziehl-Neelsen stain: acid fast bacteria like
hyperemic and painful. What are the main components that
tuberculosis.
determine such response of the body?
2. Mantux test: Allergy skin test contains
A. Plasma cells, T-lymphocytes and lymphokines
tuberculin and check for tuberculin
B. Granulocytes, T-lymphocytes and IgG
antibodies.
C. Mononuclear cells, T-lymphocytes and lymphokines
1. Use: to detect any antibody immunity
D. B-lymphocytes, IgM
for tuberculosis and the need for BCG
E. Macrophages, B-lymphocytes and Monocytes
vaccine.
2. Response: Cellular / Type 4
A5-year-old girl for diagnostic purpose underwent Mantoux
hypersensitivity (Mononuclear cells,
tuberculin skin test. 48 hours later in the place of tuberculin
T-lymphocytes )
injection developed a dense papule 1.5cm in diameter with
signs of hyperemia and necrosis. What is the mechanism of
hypersensitivity that takes place in these changes?
A. Granulomatosis
B. Antibody-dependent cytotoxicity
C. Immune complex cytotoxicity
D. Cellular cytotoxicity
E. Anaphylactic reaction

Salah Lashin !
0991907360
TB
First-year schoolchildren have received tuberculin Diagnoisis:
skin test (Mantoux test) at the school nurse’s office. 1. Ziehl-Neelsen stain: acid fast bacteria like
The purpose of this test was: tuberculosis.
A. To preventively vaccinate tuberculosis 2. Mantux test: Allergy skin test contains
B. To measure immunity stress due to diphtheria tuberculin and check for tuberculin
C. To measure allergisation rate against rickettsia antibodies.
D. To detect parotitis in the schoolchildren 1. Use: to detect any antibody immunity
E. To determine the children that need to receive for tuberculosis and the need for BCG
BCG vaccination vaccine.
2. Response: Cellular / Type 4
The first grade pupils were examined in order to sort hypersensitivity (Mononuclear cells,
out children for tuberculosis revaccination. What test T-lymphocytes )
was applied for this purpose?
A. Mantoux test
B. Schick test
C. Supracutaneous tularin test
D. Burnet test
E. Anthraxine test

Salah Lashin !
0991907360
TB
Diagnoisis:
1. Ziehl-Neelsen stain: acid fast bacteria like
tuberculosis.
2. Mantux test: Allergy skin test contains
tuberculin and check for tuberculin
antibodies.
A 36-year-old man provisionally diagnosed with renal
1. Use: to detect any antibody immunity
tuberculosis has undergone urinary sediment analysis.
for tuberculosis and the need for BCG
Microscopy revealed acid-fast bacteria, but Pryce method
vaccine.
detected no cord factor. Name the most reliable method of
2. Response: Cellular / Type 4
investigation that can confirm or refute this provisional
hypersensitivity (Mononuclear cells,
diagnosis:
T-lymphocytes )
A. Serological identification of the causative agent
B. Inoculation of laboratory animals
C. Allergy skin test
D. Toxigenicity testing
E. Phage typing of the obtained culture

Salah Lashin !
0991907360
22 . Systemic arterial pressure of an adult person lowered from
120/70 to 90/50 mm Hg and caused reflexive vasoconstriction.
In what organ will the vessels be the most constricted?
A. Adrenal glands
B. Brain
C. Heart
D. Intestine
E. Kidneys

Related
Systemic arterial pressure of an adult dropped from 120/70 to
90/50 mm Hg that led to reflectory vasoconstriction. The
vasoconstriction will be maximal in the following organ:
A. Bowels
B. Heart
C. Brain
D. Kidneys
Salah Lashin !
E. Adrenals
0991907360
23. A patient has gradually developed a skin plaque on his
face. In the center of this plaque there are necrotic patch
and an ulcer. Histopathological analysis of the biopsy
material reveals proliferation of atypical epithelial cells
with a large number of pathologic mitoses. What is the
most likely diagnosis?
A. Skin cancer
B. Sarcoma
C. Fibroma
D. Papilloma
E. Trophic ulcer

Salah Lashin !
0991907360
24. Due to a trauma, the olfactory filaments, emerging
from the nasal cavity, were torn. Through what bone do
they pass normally?
A. Cuneiform bone
B. Nasal bone
C. Maxilla
D. Inferior nasal concha
E. Ethmoid bone

Salah Lashin !
0991907360
25. A patient with bilateral adrenal damage Cell functions
1. Melanocytes: responsible for producing melanin. ‫ﺻﺑﻐﺔ اﻟﻣﯾﻼﻧﯾن‬
developed dark-brown skin color. Histochemical 2. Platelets: forms the clot to prevent hemorrhage.
analysis of the patient's skin shows negative Perls 3. Pinealocytes: secretes melatonin.
reaction, What pigment caused the skin 4. Piriform / Pyriform : Present in the cerebellum – middle layer.
Leucocytes:
discoloration in this case? 1. Eosinophilia (increase): Allergy/type 1 hypersensitivity (Asthma) & helminthic
A. Biliverdine infection.
B. Hemosiderin 2. Schwann cells: glial cells responsible for regeneration of neurons and formation
C. Porphyrin of myelin.
3. Mast cells: Secrete Histamin and heparin = vasodilation (inflammation).
D. Lipofuscin 4. Basophils: Secrete Histamin and heparin = vasodilation (inflammation).
E. Melanin 5. Antigen-presenting cells: Macrophages, monocytes: presents the antigen on its
MHC to the T-helper cells.
6. Neutrophilia (increase) : Acute inflammation. (eg, bacterial infection) + Necrosis
+ Pus
7. Monocytosis (increase):
1. Chronic inflammation (eg, inflammatory bowel disease & TB).
2. Main producers of endogenous pyrogen: interleukin 1 (increase
temperature).
8. B - cells/Plasma cells: Secretes antibodies / immunglubuline to fight bacteria
9. Macrophages: phagocytosis of bacteria + Secrete interleukin 1 ( increase
temperature / Fever).
10. T- cells
1. T-killer / Suppressor : Recognize and kill foreign bodies liker new organ.
2. Natural killers cells: protects body from tumor like t killer cells.
3. T-helper: Stimulate B cells and Eosinophils. Contains CD4. Killed by HIV
Salah Lashin ! virus.

0991907360
26 . Gram-negative bacilli that form a capsule on a Klebsiella
nutrient medium were obtained from the nasopharynx of • Diseases:
a patient provisionally diagnosed with ozena. What 1. Ozena (atrophic rhinitis): caused by the bacterial
microorganisms are the cause of the patient's condition? Klebsiella Ozaenae
A. Mycoplasma
B . Salmonella 2. Rhinoscleroma: disease caused by Klebsiella
C. Klebsiella rhinoscleromatis that infect the upper respiratory tract
D. Shigella and divided into 3 stages: catarrhal/atrophic,
E. Chlamydia granulomatous, and sclerotic stages.
• Diagnosis: MacConkey agar → Mikulicz foamy
cells.

Salah Lashin !
0991907360
Klebsiella
• Diseases:
1. Ozena (atrophic rhinitis): caused by the bacterial
species pneumoniae ozaenae.

2. Rhinoscleroma: disease cuased by Klebsiella


Study of the biopsy material revealed a granuloma rhinoscleromatis that infect the upper respiratory tract
consisting of lymphocytes, plasma cells, macrophages and divided into 3 stages: catarrhal/atrophic,
with foamy cytoplasm (Mikulicz cells), many hyaline granulomatous, and sclerotic stages.
globules. What disease can you think of? • Diagnosis: MacConkey agar → Mikulicz foamy
A. Rhinoscleroma cells.
B. Leprosy
C. Syphilis
D. Tuberculosis
E. Actinomycosis

Salah Lashin !
0991907360
Organelles ‫ﻣﻛوﻧﺎت اﻟﺧﻠﯾﺔ‬
1. Mitochondria:
27 . Electronic microscopy of the pancreatic • Function: ATP synthesis (by ATP synthase) in electron trasport chain.
cells shows the structures that separate the • Contains: 2 memberane with cristea.
• Inhertited only from mother to sons and daughters.
cell into a large number of sections, canals, • Kreans-Sayre: is one of the disease that affect Mitochondria.
and cisterns and are connected to
2. Granular / rough endoplasmic reticulum:
plasmalemma. Name these organelles:
• Contains the ribosomes which is the place of translation (protein production), for examples:
A. Ribosomes ceruloplasmin / transferrin protein.
B. Mitochondria 3. Golgi complex: Packaging and secretary function by exocytosis attached to the plasmalemma.
C. Centrosomes 4. Ribosomes:
D. Endoplasmic reticulum
1. Procaryotes (bacteria): 30s and 50s subunits
E. Golgi complex
2. Eucaryotes (humans): 40s and 60s subunits
3. Location: in the Granular / rough endoplasmic reticulum.
4. Function: Translation = protein synthesis = Regeneration.
5. Glycosomes:
1. Location: Granular and Agranular endoplasmic reticulum
2. Function: synthesize glycogen in muscles and liver.
6. Lysosomes:
• Contains: hydrolytic enzymes which are destroying enzymes (bactericidal)
• Location: Present in saliva and mucus.
• Function:
1. Immune function: Killing bacteria (Non-specific immunity).
Salah Lashin ! 2. Heterophagy: The process of digesting extracellular material by lysosomes
0991907360 3. Autophagy: The process of digesting intracellular material by lysosomes
28 . For the last three years a 45-yearold man had been
suffering from dry cough, progressing dyspnea, pulmonary
failure, and rapid weight loss. Autopsy of his body shows
cor pulmonale. The lungs are markedly fibrotic, with
cavities that resemble a honeycomb pattern. Histology
revealed interstitial fibrosis and marked lymphohistiocytic
stromal infiltration with neutrophilic admixture. Make the
diagnosis:
A. Postinflammatory pneumosclerosis
B. Multiple bronchiectasis
C. Dust-induced pneumosclerosis
D. Bronchial asthma
E. Chronic bullous emphysema

Salah Lashin !
0991907360 Check COPD collec8on!
29 . A specimen of a 10-day-old human embryo shows two
interconnected sacs (amniotic and yolk sacs). Name the Embryonic Shield
structure located in the place where these two sacs connect: Extraembryonic organ between amniotic and
vitelline/yolk sac
A. Roof of the amniotic sac
B . Extraembryonic mesoderm
C. Embryonic shield Related
In the preparation of a 10-day-old human embryo there
D. Amniotic stalk
are 2 contacting sacs visible (amniotic and vitelline).
E. Bottom of the amniotic sac Name the structure situated at the place of the contact:
A. Embryonic shield
B. Fundus of the amniotic sac
C. Fornix of the vitelline sac
D. Amniotic pedicle
E. Extraembryonic mesoderm

A histological specimen shows an extraembryonic organ


in form of a vesicle linked to the entodermal canal. Its
wall is lined with epithelium, exteriorly it is made up by
connective tissue. In the early stages of embryogenesis
this organ has hematopoietic function. Name this organ:
A. Vitelline sac
B. Allantois
C. Amnion
D. Umbillical cord
Salah Lashin ! E. Placenta
0991907360
Emberiology
Embryonic Shield / Extraembryonic organ: Present between Amniotic and vitelline/yolk sac.
Chord: slow down Hensen’s node (Primitive node)
Gastrulation:
1. The process of forming the three germ layers from epiblast and hypoblast.
2. Early gastrulation contains only endoderm and ectoderm, and later we form Mesoderm in between
them.
3. The proccess consists of these stages in the picture.
4. Stages to form the 3 layers: delamination & immigration.
1. Mesoderm:
• Mytome: Skeletal muscles.
• Cardiovascular System.
• Bone, Cartilage, tendon and ligaments.
• Derms.
2. Endoderm:
• Stomach, liver, pancreas, lung, thymus, prostate, thyroid.
3. Ectoderm:
1. Enamel and epiderms
2. Neural tube → CNS (encephalopathy, hydrocephalus, …).
4. Yolk sac stage:
• First blood that is formed in humans during embryo and provide nutrition to the embryo until the
placenta is formed. Then it degenerates ‫ ﺗﺗﻘﻠص‬and become remanent ‫ﺑﻘﺎﯾﺎ‬
5. Histototrophic stage:
• The embryo is having the from the endometrium tissue, by trophoblast.
6. Hematotrophic stage:
• The embryo is having food from the mother blood by placenta.
7. Chorion:
• Allow the transfer between the two stages.
Salah Lashin ! 8. Umbilical cord:
0991907360 • Connects with the mother’s placenta. So, it is the connection between fetus and mother ciruclation.
30 . A 35-year-old woman started starving herself. What
depot of nutrients is used during the initial stage of
starvation? How does respiratory quotient (RQ) change
during this period?
A. Carbohydrates, RQ approaches 1
B. Lipids, RQ approaches 0.85
C. Lipids, RQ approaches 0.72
D. Proteins, RQ approaches 0.7
E. Proteins, RQ approaches 1

Salah Lashin !
0991907360
Energy Collection
1. Rage of normal of BMR:
1. 60-70%.
2. ~ 8 % Above and below is normal.
2. Basal metabolic rate calculation needs: to measure
Sex, age, height and weight.
2. Highest energy expenditure: Action of skeletal
muscles
3. Respiratory coefficient of Macromolecules (Carbon /
oxygen):
1. Carbohydrates: 1
2. Protein: 0.8
3. Fat: 0.7

Salah Lashin !
0991907360
A person presents with base metabolism that exceeds Following the estimation of a person’s energy expenditures it
normal by 8%. It means that the processes of energy was established that the respiratory quotient was equal to 1,0.
metabolism in this person are: This means that the compound that is mainly oxidized in the
A. Within normal range cells is:
B. Moderately increased A. Carbohydrates
C. Significantly inhibited B. Proteins
D. Moderately inhibited C. Fats
E. Significantly increased D. Proteins and carbohydrates
E. Carbohydrates and fats
The value of basal metabolism of a man under
examination exceeds the due value by 8%. This Estimation of heat expenditures of a man’s organism by means
means that the man has the following intensity of of indirect calomitery had the following results: the organism
energy metabolism processes: consumed 1000 ml of oxygen and emitted 800 ml of carbonic
A. Normal acid per minute. What is the respiratory quotient of a man under
B. Moderately increased examination?
C. Moderately decreased A. 0,8
D. Considerably increased B. 1,25
E. Considerably decreased C. 0,9
D. 0,84
E. 1,0
Salah Lashin !
0991907360
What factor results in the highest energy expenditure under
the normal vital activity conditions?
A. Action of skeletal muscles
B. Increase of environment temperature
When measuring power inputs of a person by the method of
C. Decrease of environment temperature
indirect calorimetry the following results were obtained: oxygen
D. Mental work
consumption is 1000 ml and carbon dioxide production is 800
E. Food rich in calories
ml per minute. The person under examination has the following
respiratory coefficient:
Normal metabolic rate and energy expenditure should be
A. 0,8
taken into account when actual basal metabolic rate of a
B. 1,25
patient is being determied by means of indirect
C. 0,9
calorimetry. Normal metabolic rate can be determined
D. 0,84
most accurately based on the patient’s:
E. 1,0
A. Sex, age, height and weight
B. Body surface area and weight
C. Respiratory coefficient and body surface area
D. Height and respiratory coefficient
E. Respiratory coefficient and caloric coefficient of
oxygen
Salah Lashin !
0991907360
Microspherocytic hemolytic anemia
(Minkowsky-Shauffard disease / hereditary spherocytosis)

1. Definition: Inherited defect of RBC cytoskeleton-


membrane (Membranopathy) → Membrane blebs are
formed and lost over time → Loss of membrane renders
cells round (spherocytes) instead of disc-shaped →
Spherocytes are less able to maneuver through splenic
sinusoids and are consumed by splenic macrophages,
resulting in anemia.

2. Diagnosis:
• Spherocytes with loss of central pallor.
• Haemolytic Jaundice (extravascular homolysis).
• Osmotic fragility test: increased spherocyte fragility in
hypotonic solution.

Salah Lashin !
0991907360
A 34 year old woman was diagnosed with hereditary A patient with microspherocytic hemolytic anemia
microspherocytic hemolytic anemia (Minkowsky-Shauffard (Minkowski-Chauffard disease) presents with
disease). What mechanism caused haemolysis of erythrocytes? permeability of crythrocyte membrane, which causes
A. Membranopathy increased entry of sodium ions and water into the
B. Enzymopathy cell. As a result, erythrocytes assume the shape of
C. Hemoglobinopathy spherocytes and can be easily destroyed. What is the
D. Autoimmune disorder leading mechanism of erythrocyte damage in this
E. Bone marrow hypoploasia case?
A. Protein
As a result of increased permeability of the erythrocyte B. Acidotic
membrane in a patient with microspherocytic anaemia C. Nucleic
(Minkowsky-Shauffard disease) cells receive sodium ions and D. Electrolyte-osmotic
water. Erythrocytes take form of spherocytes and can be easily E. Calcium
broken down. What is the leading mechanism of erythrocyte
damage in this case?
A. Electrolytic osmotic
B. Calcium
C. Acidotic
D. Protein
E. Nucleic
Salah Lashin !
0991907360
31. A patient with microspherocytic hemolytic anemia Microspherocytic hemolytic anemia
(Minkowski-Chauffard disease) presents with permeability (Minkowsky-Shauffard disease / hereditary
of crythrocyte membrane, which causes increased entry of spherocytosis)
sodium ions and water into the cell. As a result,
erythrocytes assume the shape of spherocytes and can be 1. Definition: Inherited defect of RBC cytoskeleton-
easily destroyed. What is the leading mechanism of membrane (Membranopathy) → Membrane blebs are
erythrocyte damage in this case? formed and lost over time → Loss of membrane
A. Protein renders cells round (spherocytes) instead of disc-
B. Acidotic shaped → Spherocytes are less able to maneuver
C. Nucleic through splenic sinusoids and are consumed by
D. Electrolyte-osmotic splenic macrophages, resulting in anemia.
E. Calcium

Salah Lashin !
0991907360
32 . In the hematology unit a patient with leukemia was
prescribed 5- Fluorouracil. This drug:
A. Catalyzes replication
B . Stimulates DNase Anti-tumor durgs
C. Inhibits translation
D. Inhibits DNA synthesis
E. Inhibits transcription

Salah Lashin !
0991907360
Related
Methotrexate (structural analogue of the folic acid which is
competitive inhibitor of the dihydrofolatreductase) is
prescribed for treatment of the malignant tumour. On which
level does methotrexate hinder synthesis of the nucleic
acids?
A. Mononucleotide synthesis
B. Replication
C. Transcription
D. Reparation
E. Processing

An oncological patient had been administered methotrexate.


With time target cells of the tumour lost sensitivity to this
drug. At the same time the change in gene expression of the
following enzyme is observed: Mthotrexate:
A. Dehydropholate reductase 1. Inhibits the enzyme:
B. Thiaminase 2. Dehydropholate reductase =
C. Deaminase 3. No folate activation =
D. Pholate oxidase 4. No DNA synthesis =
E.Salah
Pholate !
decarboxylase
Lashin 5. Treat cancer.
0991907360
33 . A 56-year-old patient with heart failure has edema Edema
of feet and shins. The skin in the edematous area is pale (Accumulation of fluid in the interstitial / intracellular space)
and cold. What is the leading link of edema
pathogenesis in this patient? 1. Increase in hydrostatic / dynamic pressure: ‫ﺿﻐط اﻟدﻓﻊ‬
1. Mechanism: Pressure on the vessel wall = Increase filtration to
A. Positive fluid balance
the interstitial space = edema.
B. Increased hydrostatic pressure in the venules
2. Causes:
C. Decreased oncotic pressure in the capillaries 1. Standing for a long time.
D. Increased permeability of the capillaries 2. Heart failure → Decreased CO → poor blood circulation
E. Disturbed lymph efflux leads to pressure down in the vein →.
3. Valvular damage: blood acumulates in the veins.
2. Decrease in oncotic pressure: ‫ﺿﻐط اﻟﺳﺣب‬
• Mechanism: No Albumin = No oncotic pressure = edema.
• Causes:
1. Glumerulonephritis / nephrotic syndrome = proteinuria.
2. Diet: no protein intake.
3. Liver issues.
3. Inflammation: histamine = vasodilation = more blood filtration ( ⬆
Capillary permeability) = edema.
4. Increased work of antidiuretic hormones: (ADH and Aldosterone) =
No diuresis = Fluid accumulation = Edema.
5. Combined (mixed): two causes.
Salah Lashin ! Treatment: diuretics like furosemide = loss water = less edema.
0991907360
34 . A 64-year-old man has signs of acute metabolic and
energy exchange problems. Computed tomography
detected a tumor in one of the brain regions. What cerebral
structure that plays a major role in metabolism regulation
can be affected in this case?
A. Hypothalamus
B. Thalamus
C. Reticular formation
D. Red nucleus
E. Substantia nigra

Salah Lashin !
0991907360
35. The patient's examination detected an inflammation of
a certain anatomical structure that equalizes the pressure
between the tympanic cavity and the pharynx. Name this
structure:
A. External auditory meatus
B. Inner ear
C. Internal auditory meatus
D. Major mastoid air cell
E. Eustachian tube

Salah Lashin !
0991907360
36 .ECG of a woman with ischemic heart disease shows
the following: heart rate -230/min., deformed P wave,
ventricular complexes remain unchanged. What heart
rhythm disorder is it?
A. Atrial flutter
B. Ventricular extrasystole
C. Ciliary arrhythmia
D. Atrial paroxysmal tachycardia
E. Ventricular fibrillation

Salah Lashin !
0991907360
Related
A 17-year-old girl suffers from periodical palpitations that
last several minutes. Her heart rate is 200/min., rhythmic.
What heart rhythm disorder developed in this patient?
A. Sinus tachycardia
B. Sinus bradycardia
C. Paroxysmal tachycardia
D. Extrasystole
E. Atrioventricular block

A 15-year-old teenager complains of lack of air, general


weakness, palpitations. Heart rate is 130/min., BP is 100/60
mm Hg. ECG: QRS complex has normal shape and duration.
The number of P waves and ventricular complexes is equal,
T wave merges with P wave. What type of cardiac
arrhythmia is observed in the teenager?
A. Atrial thrill
B. Sinus extrasystole
C. Atrial fibrillation
D. Sinus tachycardia
E.Salah
Paroxysmal ! tachycardia
Lashin atrial Watch arrhythmia!
0991907360
37 . A 48-year-old man died with signs of heart failure.
Macroscopy of the heart shows that the mitral valve cusps
Stains:
are dense, thickened, and moderately deformed. 1. Sudan III: white fats
2. Hemomelanin: Malaria
Microscopically, the bundles of collagen fibrils are
3. Bipolar/Burri-Gins: Plague
homogenized, eosinophilic, and surrounded with slight
4. Congo red: Amyloidosis
macrophage infiltration; no metachromasia. Picrofuchsin
staining reveals yellow foci. Diagnose the type of 5. Ziehl-Neelsen: tuberculosis
connective tissue damage: 6. Metachromasia (toluidine blue): Mucoid edema.
7. Picrofuchsin: turns yellow with fibrinoid swelling.
A. Mucoid swelling
B. Sclerosis
C. Amyloidosis
D. Hyalinosis
E. Fibrinoid swelling

Salah Lashin !
0991907360
38. Intracellular examination of biopotentials in an isolated
tissue culture shows that the action potentials that develop
in the cells can be characterized by a plateau for up to 300
milliseconds in the repolarization phase. What tissue is
being studied?
A. Contractile myocardium
B. Skeletal muscle
C. Nerve fiber
D. Smooth muscle
E. Atypical cardiac muscle cells

Salah Lashin !
0991907360
39. A patient developed punctate hemorrhages after a
tourniquet had been applied. It occurred due to functional
disturbance of the following blood corpuscles:
A. Erythrocytes
B. Neutrophils Platelets / Thrombocytes ‫اﻟﺻﻔﺎﺋﺢ اﻟدﻣوﯾﺔ‬
C. Platelets 1. Fuction: form the plug to prepare for cloting.
D. Lymphocytes 2. Orgin: megkaryocytes in the bone marrow.
E. Monocytes 3. Deficiency: superficial bleeding.
4. Asprin /Acetylsalicylic : Anti-platelet. (prevent platelet aggregatoin)

Related question Related question.


A histologic study of a red bone marrow puncture Examination of a 35-year-old patient included histological
sample had been conducted in the course of analysis of the red bone marrow punctate that revealed a
examination of a 35-year-old patient, and significant significant increase in the megakaryocyte number. This will
megakaryocyte number depletion was detected. What cause the following alterations in the peripheral blood:
peripheral blood changes will result from that? A. Agranulocytosis
A. Thrombocytopenia B. Leukocytosis
B. Leukocytosis C. Thrombocytopenia
C. Thrombocytosis D. Thrombocytosis
Salah Lashin !
D. Agranulocytosis E. Leukopenia
0991907360
E. Leukopenia
Related
An examination of a 26-year-old patient involved histological
analysis of bone marrow punctate which revealed a significant
decrease in the number of megakaryocytes. At the same time the
following blood corpuscles should be decreased in number:
A. Eosinophils
B. Erythrocytes
C. Platelets
D. Neutrophils

Salah Lashin !
0991907360
40. Pathomorphology of the gallbladder after
Phlegmon ‫اﻟﺻدﯾد اﻟﻣﻧﺗﺷر‬:
cholecystectomy shows that it is enlarged, its walls are Purulent infiltration spreads diffusely
thickened, its serous tunic is dull and plethoric; there are between the tissue.
viscous yellow-green masses in the gallbladder cavity.
Microscopically, a diffuse infiltration of segmented
neutrophils is observed in the gallbladder wall. What type
of cholecystitis is the most likely in this case?
A. Acute gangrenous cholecystitis
B. Granulomatous cholecystitis
C. Chronic cholecystitis
D. Acute catarrhal cholecystitis
E. Acute phlegmonous cholecystitis

Salah Lashin !
0991907360
Pathomorphology
☠""
Inflammation
Terminology
1. Pus = purulent = suppurative = pyo- = ‫ﺻدﯾد‬
2. Diffuse: ‫ﯾﻧﺗﺷر‬
3. Infiltration: ‫ اﻟﻣرﺷﺢ‬-‫اﻟﺷﺊ اﻟﻣﻧﺗش‬

Salah Lashin !
0991907360
Types of inflammation
Purulent inflammation ‫اﻻﻟﺗﮭﺎب اﻟﺻدﯾدي‬
• Dead tissue fluid.
• Usual color of pus is yellow.
• Secreted from Neutrophils
Abscess
• Green / blue pus is suggestive for: Pseudomonas aeruginosa.
Types of purulent:
1. Abscess (‫)اﻟﺻدﯾد اﻟﻣﺟوف‬: Formation of a cavity filled with pus.
1. Acute:
• Cavity wall is thin. One membrane.
• Surrounded by the tissue organ.
2. Chronic:
• Cavity wall is thick (dense fibres).
• Surrounded by more than one membrane.
Phlegmon
2. Phlegmon (‫ )اﻟﺻﯾد اﻟﻣﻧﺗﺷر‬: Purulent infiltration spreads diffusely between the tissue.

3. Empyema (‫ )ﺻدﯾد اﻷﻣﺎﻛن اﻟﻣﻐﻠﻘﺔ‬: Accumulation of pus in isolated anatomical cavities.


• Pleural cavity - pleural empyema.
• Pericardial cavity - pericardial empyema.
4. Furuncle:
• Purulent inflammation of the hair follicle.
• If didn’t mention several hair follicle choose furuncle. (krok)
5. Carbuncle:
• Purulent inflammation of multiple hair follicles.(multiple furuncles)
Salah Lashin !
• Deep Subcutaneous tissue death.
0991907360 Empyema
Phlegmon ‫اﻟﺻدﯾد اﻟﻣﻧﺗﺷر‬:
Purulent infiltration spreads diffusely
between the tissue.
A 65 years old man presents acute mandibular
osteomyelitis. 3 days after the disease onset he developed Phlegmon
marked edema of the skin and soft submandibular cervical
tissues. Microscopically there is a diffuse infiltration with
neutrophils. What complication of the main disease
occurred in the patient’s skin tissues?
A. Carbuncle
B. Phlegmon
C. Abscess

Salah Lashin
Salah Lashin !
0991907360 0991907360
Autopsy of a patient, who died of bilateral Abscess ‫اﻟﺻدﯾد اﻟﻣﺟوف‬:
bronchopneumonia, shows in the left lung lower lobe a Formation of a cavity filled with pus.
cavity 5 cm in diameter, filled with liquid yellowish-white 1. Acute:
substance. What complication of the patient’s pneumonia • Cavity wall is thin. One membrane.
had developed? • Surrounded by the tissue organ.
A. Tuberculoma 2. Chronic:
B. Gangrene • Cavity wall is thick (dense fibres).
C. Granuloma • Surrounded by more than one
D. Sequestrum membrane.
E. Abscess

Abscess

Salah Lashin
Salah Lashin !
0991907360 0991907360
A patient developed a tender red nodule in the lower jaw 1. Furuncle:
area. Histologically there is accumulation of purulent • Purulent inflammation of the hair follicle.
exudate in several hair follicles. What clinicopathological • Furuncle with diabetes mellitus (krok)
type of inflammation is observed? 2. Carbuncle:
A. Abscess • Purulent inflammation of multiple hair
B. Phlegmon follicles.(multiple furuncles)
C. Furuncle • Deep Subcutaneous tissue death.
D. Carbuncle
E. Hypostatic abscess

A 53-year-old man suffering from diabetes mellitus has


developed a painful conical induration, bluish-red with
yellow centre, on the skin of his neck. Such changes are
characteristic of:
A. Furuncle
B. Abscess
C. Carbuncle
D. Phlegmon
E. Empyema Salah Lashin
Salah Lashin !
0991907360 0991907360
Autopsy of a patient who suffered from croupous Empyema
pneumonia and died from pneumococcal sepsis Accumulation of pus isolated anatomical cavities.
revealed 900 ml of turbid greenish-yellow liquid in the
right pleural cavity. Pleural leaves are dull, plephoric. • Pleural cavity - pleural empyema.
Name the clinicopathological form of inflammation in • Pericardial cavity - pericardial empyema.
the pleural cavity:
A. Phlegmon
B. Fibrinous inflammation
C. Empyema
D. Chronic abscess
E. Acute abscess

Salah Lashin !
0991907360
41. A medical student was hospitalized into the infectious
diseases unit on the 2nd day after the disease onset. The B cells
Gamma globulins / Immunoglobulins / Antibodies
patient is suspected to have infectious mononucleosis.
What results of laboratory analysis can confirm this
1. IgM: First to appear (Man) $
diagnosis immediately on the day of the hospitalization?
2. IgA: presnent in mucusa.
A. Fourfold increase in the number of antibodies detected
3. IgE: Allergy
to Epstein-Barr virus was detected
4. IgG:
B. IgM antibodies to Epstein-Barr virus were detected
• Later response.
C. IgM antibodies to herpes simplex virus were detected
• Immunocomplex response
D. Cytomegalovirus antibodies were detected
• Can pass to placenta.
E. Herpesvirus was isolated

Salah Lashin !
0991907360
Note: Interferon is the natural antiviral agents. Viruses 8. Human Herpes virus (HHV):
1. Rabies: ‫ﻣرض اﻟﺳﻌﺎر‬
• Management: Antirabic vaccine. • Note: HHV can be latent and produce a reactivation form of the
• Transmission: rabid animals (ex, dogs) disease.
• Diagnosis: Presence of Negri bodies in the brain • Treatment:
2. AIDS (HIV). 1. Drug: Acyclovir.
• Contains gp 120 on its envelope and bound to CD 4 marker on the T 2. MOA: It inhibits synthesis of nucleic acids of viruses.
helper cells.
• Causes: Reverse transcription 1. HHV1(oral cavity):
• Test: ELISA and Western blot (confirmation). • Symptoms: Causes Oral rash (cold sores) → Infect
3. Infectious mononucleosis: trigeminal ganglion and cause latent infection.
1. Epstein-Barr virus • Diagnosis: sample from the vesicular fluid.
2. Diagnosis: IgM antibodies. 2. HHV2: infects Genitalia.
4. Hepatisis B: 3. HHV3 - Varicella zoster virus (VZV)
1. Transmissition: Blood and sexual contact. Same as Hepatitis C. 1. Disease: Chickenpox → latent.
2. Diagnosis: ELISA and Councilman bodies and 2. Reactivated disease: shingles (zoster).
5. Hepatisis D: 3. Note: smallpox is caused by Variola virus another virus and
• Comes with Hepatitis (coinfection) B or after B (suprainfection). characterized by Paschen’s bodies/ corpuscle.
• Transmissition: Blood and sexual contact, like B and C. 4. HHV5-Cytomegalovirus:
6. Hepatisis E and A: 1. Symptoms: usually asymptomatic but can causes digestive
• Transmissition: food and water (fecael oral). and respiratory symptoms in immunocompromised patient
7. Smallpox (Variola virus): characterized by Paschen’s bodies / corpuscle. (ex, HIV).
8. Rotavirus: looks like wheel. 2. Diagnosis: Enlargement of the cells (owl eye cells).
9. Retrovirus: universality property. 5. HHV8: Causes Kaposi’s sarcoma: a malignant tumour originates
10. Influenza: hemorrhage in the lung and transmit through airborne. from endothelial cells.

Salah Lashin !
0991907360
42. A patient was diagnosed with bronchial asthma. What
changes in the patient's pulmonary ventilation will be
observed?
A. Increase of vital capacity
B. Increase of expiratory reserve volume
C. Decrease of forced expiratory volume
D. Decrease of residual volume
E. Increase of forced expiratory volume

Salah Lashin !
0991907360
43. A 35-year-old man was prescribed atropine sulfate
eyedrops for examination of the fundus of the eye. To restore
accommodation, he was given pilocarpine hydrochloride
eyedrops, which turned out to be ineffective. Why was
pilocarpine hydrochloride ineffective in this case?
A. One-way antagonism
B. Synergism
C. Tolerance
D. Two-way antagonism
E. Tachyphylaxis

Salah Lashin !
0991907360
44 . Normally, the maternal and fetal blood circulation
have no direct connection. The fetal blood flows through
the vessels in the chorionic villi, while maternal blood
circulates through the intervillous space of the
endometrium. What separates fetal and maternal blood?
A. Rohr amorphous fibrinoid
B. Fibrinoid oxyphilic mass (Langhans fibrinoid)
C. Hemochorial barrier
D. Closing plate of the decidua basalis
E. Connective tissue septa

Salah Lashin !
0991907360
45 . A sample of pus discharged from the patient's urethra
was inoculated onto a special nutrient medium, which
resulted in growth of pale-blue colonies. Microscopy of
these preparations detected Gram- negative bean-shaped
diplococci. It is the causative agent of:
A. Tularemia
B . Gonorrhea
C. Melioidosis
D. Clamidiosis
E. Syphilis

Salah Lashin !
0991907360
46 . A patient underwent trepanation and curettage of the
air cells in a certain process of the temporal bone, because
of an inflammation that spread there from the middle ear.
What process of the temporal bone underwent the surgery?
A. Processus styloideus
B. Processus zygomaticus
C. -
D. Processus mastoideus
E. Processus pterygoideus

Salah Lashin !
0991907360
47 . A 40-year-old man complains of general weakness,
headache, wet cough, and shortness of breath. After
clinical examination he was diagnosed with pneumonia.
What type of hypoxia is observed in the patient?
A. Circulatory hypoxia
B. Hemic hypoxia
C. Hypoxic hypoxia
D. Respiratory hypoxia
E. Tissue hypoxia

Check for the collection in the topic file !

Salah Lashin !
0991907360
48. After sensitization a test animal received Hypersensitivity ‫ﻓرط اﻟﺣﺳﺎﺳﯾﺔ‬
subcutaneously a dose of antigen. At the site of
injection a fibrinous inflammation developed with When the normally protective immune system has a harmful effect on
alteration of vessel walls, basal substance, and the body = ‫ﻏﺷوﻣﯾﮫ ﻣن ﺟﮭﺎز اﻟﻣﻧﺎﻋﺔ‬
fibrous structures of connective tissue. The
inflammation took form of mucoid fibrinoid Type 1 Hypersensitivity
degeneration, and fibrinoid necrosis. What (Allergy/ Anaphlaxsis / Atopic / reagionic/ Immediate)
immune response occurred in the tesi animal?
A. Transplantation immune reaction 1. Definition: an abnormal immunological response to a normal
B. Delayed hypersensitivity harmless stimulus (Allergen) e.g, food, pollen, drug or animals.
C. Normergic reaction 2. Mechanism:
D. Immediate hypersensitivity • Allergen: Stimulates B cells = Secreste IgE = activates
E. Granulomatosis 1. Mast cells, basophils = secrets Histamine =
vasodilatation (allergy symptoms)
2. Eosinophils.
3. Later: it took becomes mucoid and fibrinoid swelling
3. Time: Very fast = within minutes
4. Management:
- IV Epinephrin/Adrenaline.
- Then cortisol and antihistamine.
5. Prevention:
Salah Lashin ! • Bezredka Method: Binding to IgE fixed to mast cells.
0991907360 • Desensitization: Intermittent administration of allergen.
Candidiasis - Oral thrush
1. Pathogen Candida
2. Features:
1. Pain and burring sensation.
2. White patches (Easily removed) in tongue.
3. Opportunistic infection.
3. Test:
1. Gram positive.
2. Colonies: budding cells arranged in chains.
3. KOH.
4. Treatment: Nystatin and Fluconazole.

Salah Lashin !
0991907360
Candidiasis - Oral thrush
1. Pathogen Candida
2. Features:
An 18-year-old patient has developed candidiasis after
1. Pain and burring sensation.
the case of pneumonia treated with lactam antibiotic.
What antimycotic agent should be prescribed? 2. White patches (Easily removed) in tongue.
3. Opportunistic infection.
A. Fluconazole
3. Test:
B. Streptomycin
1. Gram positive.
C. Ampicillin
2. Colonies: budding cells arranged in chains.
D. Phthalylsulfathiazole
3. KOH.
E. Trimethoprim/sulfamethoxazole (Biseptol)
4. Treatment: Nystatin and Fluconazole.
A patient developed burning sensation in the oral cavity
and white fuzzy coating on the tongue. Oral thrush is
diagnosed. What drug of those listed below should be
used?
A. Nystatin
B. Amphotericin
C. Griseofulvin
D. Tetracycline
E. Gentamicin

Salah Lashin !
0991907360
Candidiasis - Oral thrush
A 58-year-old male patient visited his dentist with the chief 1. Pathogen Candida
complaint of itching and burning sensation in his mouth. On 2. Features:
intraoral examination, diffuse white patches were seen on his 1. Pain and burring sensation.
tongue, right and left buccal mucosa, as well as on his hard 2. White patches (Easily removed) in tongue.
palate and soft palatal region. The potassium hydroxide (KOH) 3. Opportunistic infection.
preparation of the specimen revealed nonpigmented septate 3. Test:
hyphae. Administration of which of the following is the most 1. Gram positive.
appropriate initial step in treatment of this patient? 2. Colonies: budding cells arranged in chains.
A. Nystatin 3. KOH.
B. Gentamicin 4. Treatment: Nystatin and Fluconazole.
D. Penicillin

Mucosa and sputum of a patient, who fora long time taking was
immunosuppressants, contain large Gram- positive oval budding
cells that are arranged chaotically and elongated cells, arranged
in chains. What causative agent was detected?
A. Streptobacteria
B. Candida
C. Yersinia
D. Streptococci
E. Actinomycetales
Salah Lashin !
0991907360
49. Mucosa and sputum of a patient, who fora long time Candidiasis - Oral thrush
taking was immunosuppressants, contain large Gram- 1. Pathogen Candida
positive oval budding cells that are arranged chaotically 2. Features:
and elongated cells, arranged in chains. What causative 1. Pain and burring sensation.
agent was detected? 2. White patches (Easily removed) in tongue.
A. Streptobacteria 3. Opportunistic infection.
B. Candida 3. Test:
C. Yersinia 1. Gram positive.
D. Streptococci 2. Colonies: budding cells arranged in chains.
E. Actinomycetales 3. KOH.
4. Treatment: Nystatin and Fluconazole.

Salah Lashin !
0991907360
50. Obturation of the patient's bile duct resulted in decreased
bile flow into the duodenum, which, in turn, caused problems
Diges8ve enzyme
with absorption of:
A. Lipids
B. Carbohydrates
C. Proteins
D. Proteins and carbohydrate
E. Mineral salts

Salah Lashin !
0991907360
Edema
51. An 11-year-old girl is brought to the doctor's (Accumulation of fluid in the interstitial / intracellular space)
office by her mother who complains that her
daughter presents with weakness and swollen face 1. Increase in hydrostatic / dynamic pressure: ‫ﺿﻐط اﻟدﻓﻊ‬
for 3 days. The mother states that her daughter had 1. Mechanism: Pressure on the vessel wall = Increase filtration to
always been healthy and active until the onset of the interstitial space = edema.
symptoms. Physical examination reveals generalized 2. Causes:
swelling of the face and pitting edema on the lower 1. Standing for a long time.
limbs. Upon inquiry, the girl describes a foamy 2. Heart failure → Decreased CO → poor blood circulation
appearance of her urine but denies blood in urine, leads to pressure down in the vein →.
nocturnal urinary frequency, or pain during 3. Valvular damage: blood acumulates in the veins.
urination. Laboratory study shows proteinuria and 2. Decrease in oncotic pressure: ‫ﺿﻐط اﻟﺳﺣب‬
microscopic hematuria. Which of the following is • Mechanism: No Albumin = No oncotic pressure = edema.
the most likely cause of the findings in the • Causes:
laboratory study of urine? 1. Glumerulonephritis / nephrotic syndrome = proteinuria.
2. Diet: no protein intake.
A. -
3. Liver issues.
B. Increased hydrostatic pressure in the Bowman
3. Inflammation: histamine = vasodilation = more blood filtration ( ⬆
capsule
Capillary permeability) = edema.
C. Increased permeability of the glomerular 4. Increased work of antidiuretic hormones: (ADH and Aldosterone) =
capillary wall No diuresis = Fluid accumulation = Edema.
D. Increased glomerular hydrostatic pressure 5. Combined (mixed): two causes.
E. Increased plasma oncotic pressure
Salah Lashin ! Treatment: diuretics like furosemide = loss water = less edema.
0991907360
52 . Abdominal cavity revision detected a venous bleeding
from the hepatoduodenal ligament. What vein is damaged?
A. Inferior vena cava
B. Superior mesenteric vein
C. Inferior mesenteric vein
D. Hepatic portal vein
E. Splenic vein

Hepatoduodenal ligament content:


1. Proper hepatic artery.
2. Portal vein.
3. Common bile duct.
4. Common hepatic duct.

Salah Lashin !
0991907360
Related
A surgeon has to find the common hepatic duct during the
operative intervention on account of concrements in the gall
ducts. The common hepatic duct is located between the
leaves of:
A. Hepatoduodenal ligament
B. Hepatogastric ligament
C. Hepatorenal ligament
D. Round ligament of liver
E. Venous ligament

During an invasive operation the surgeon needs to access the


omental bursa of the peritoneal cavity via the omental
foremen (foramen of Winslow). What anatomical structure
makes up the anterior border of this foramen?
A. Visceral surface of liver
B. Hepatoduodenal ligament
C. Hepatorenal ligament
D. Greater omentum
E. Superior part of duodenum

Salah Lashin !
0991907360
53 .Because of a common bile duct obstruction detected on
the X-ray, the bile stopped flowing into the duodenum.
Diges8ve enzyme
What process is expected to become disturbed in this case?
A. Emulsification of lipids
B . Inhibition of salivation
C. Carbohydrate hydrolysis
D. Hydrochloric acid secretion in the stomach
E. Protein absorption

Salah Lashin !
0991907360
54. Before tooth extraction the patient was given Hypersensitivity ‫ﻓرط اﻟﺣﺳﺎﺳﯾﺔ‬
conduction anesthesia with lidocaine. After
lidocaine was administered, the patient developed When the normally protective immune system has a harmful effect on
edema and hyperemia around the injection site, the body = ‫ﻏﺷوﻣﯾﮫ ﻣن ﺟﮭﺎز اﻟﻣﻧﺎﻋﺔ‬
itching skin, general weakness, hypotension, and
motor excitation. What complication occurred in Type 1 Hypersensitivity
this case? (Allergy/ Anaphlaxsis / Atopic / reagionic/ Immediate)
A. Tachyphylaxis
B. Toxic effect 1. Definition: an abnormal immunological response to a normal
C. Tolerance harmless stimulus (Allergen) e.g, food, pollen, drug or animals.
D. Allergic reaction 2. Mechanism:
E. Drug dependence • Allergen: Stimulates B cells = Secrete IgE = activates
1. Mast cells, basophils = secrets Histamine =
vasodilatation (allergy symptoms)
2. Eosinophils.
3. Later: it took becomes mucoid and fibrinoid swelling
3. Time: Very fast = within minutes
4. Management:
- IV Epinephrin/Adrenaline.
- Then cortisol and antihistamine.
5. Prevention:
Salah Lashin ! • Bezredka Method: Binding to IgE fixed to mast cells.
0991907360 • Desensitization: Intermittent administration of allergen.
55. General adaptation syndrome is realized
Pituitary-adrenal axis
predominantly through neuroendocrine system. What
Part of neuroendocrine system which is
link of this system is leading in the pathogenesis of the
responsible for response to environmental stress
developing rection?
A. Pituitary-insular
B. Pituitary-juxtaglomerular
C. Pituitary-adrenal
D. Pituitary-thyroid
E. Pituitary-adrenogenital

Salah Lashin !
0991907360
Diabetics mellitus
56 . A man was hospitalized in a comatose state. He 1. If glucose acumulates in eyes will cause retinal disorder → glucoma and
has a 5-year-long history of diabetes mellitus type 2. Microangiopathy → blindness.
2. Increased keto acids = Kussmaul’s respiration.
Objectively, the patient's respiration is noisy, deep, 3. Polyuria.
with acetone breath odor. Blood glucose is 18.2 4. Type 1 diabetes: beta cells of pancreas don’t secrete insulin.
mmol/L, ketone bodies - 100 micromol/L. These 5. Has incomplete oxidation
signs are characteristic of the following diabetes 6. Ketoacidic coma: increased ketone bodies → Aceton smell → Metabolic acidosis
7. Hyperglycaemic / hyperosmolar coma: Glucose ~ 20 micromole/l
complication: 8. Deficiency of insulin → proteolysis → aminoacidemia
A. Ketoacidotic coma 9. Renal diabetes: increased glucose levels in urine due to disturbance in glucose
B. Hepatic coma reabsorption from proximal convoluted tubules.
Tests:
C. Hyperosmolar coma 1. Blood sugar: Normal Glucose: 3.3 – 5.5. Diabetes: > 12.
D. Hypoglycemic coma 2. Glycated haemoglobin: the test use to detect history of diabetics (≥ 6.5).
E. Hyperglycemic coma 3. Glucose tolerance: is measured after fasting glucose test.
Treatment:
1. Insulin (replacment therapy).
1. Use:
1. Type 1 diabetes mellitus.
2. Inhibit the process of ketosis
3. Note: Rapid-acting insulin is used during coma.
2. Side effect: Hypoglycemia = coma (Carbohydrate starvation).
3. Antidote: Glucose or adrenaline.
2. Glibenclamide:
1. Use: type 2
2. Sulphonyl urease derivate
3. Stimulates generation of insulin from beta cells.
Salah Lashin ! 3. Metformin:
1. Use: type 2 (drug of choice).
0991907360
57. A 50-year-old woman has a round neoplasm, 2.5 cm in
diameter, in her ovary. On section the neoplasm has a
cavity with pale yellow transparent liquid. The inner walls
of the cavity are smooth. Macroscopically, this neoplasm
can be classified as a:
A. Infiltration
B. Node with an infiltration in its center
C. Cyst
D. Ulcer
E. Node

Related
At autopsy, section of the right ovary shows a round lesion 2.5
cm in diameter with a clear serous fluid, surrounded by a
smooth glistening membrane. Which of the following
macroscopic lesions best represents the autopsy findings?
A. Cyst
B. Nodule
C. Ulcer
D. Infiltrate
Salah Lashin ! E. Nodule with central necrosis
0991907360
58 . Blood test shows low hemoglobin levels. What
function of the blood will be disturbed in this case?
A. Coagulability
B. Hormone transport
C. Gas transport
D. Immunity maintenance
E. Nutrient transport

Salah Lashin !
0991907360
59 . A 15-year-old teenager was hospitalized into Hypersensitivity ‫ﻓرط اﻟﺣﺳﺎﺳﯾﺔ‬
the allergology department with diagnosis of
bronchial asthma. In this case the development of When the normally protective immune system has a harmful effect on
main clinical signs is caused by overproduction of the body = ‫ﻏﺷوﻣﯾﮫ ﻣن ﺟﮭﺎز اﻟﻣﻧﺎﻋﺔ‬
certain antibodies. Name these antibodies:
A. IgA Type 1 Hypersensitivity
B. - (Allergy/ Anaphlaxsis / Atopic / reagionic/ Immediate)
C. IgD
D. IgE 1. Definition: an abnormal immunological response to a normal
E. IgM harmless stimulus (Allergen) e.g, food, pollen, drug or animals.
2. Mechanism:
• Allergen: Stimulates B cells = Secrete IgE = activates
1. Mast cells, basophils = secrets Histamine =
vasodilatation (allergy symptoms)
2. Eosinophils.
3. Later: it took becomes mucoid and fibrinoid swelling
3. Time: Very fast = within minutes
4. Management:
- IV Epinephrin/Adrenaline.
- Then cortisol and antihistamine.
5. Prevention:
Salah Lashin ! • Bezredka Method: Binding to IgE fixed to mast cells.
0991907360 • Desensitization: Intermittent administration of allergen.
60 .A 55-year-old man was examined by the
endocrinologist because of a disturbance in the pancreatic
endocrine function. This disturbance manifests as
decreased blood glucagon levels. What cells of this gland
do not function properly in this case?
A. D cells
B. D1 cells
C. B cells
D. PP cells
E. A cells

Salah Lashin !
0991907360
Analgetics (painkillers) #
61 . For relief of pain syndrome, a patient with myocardial Narcotics (opioids) $
infarction was prescribed an analgesic. The patient's • Work on the opiate / Mu receptor through G inhibitor
condition improved, but with time the patient developed protein.
• Antidote (‫)اﻟﻣﺻل‬: Naloxone.
euphoria and miosis. The doctor noted respiratory
• Will cause withdrawal syndrome (‫ )أﻋراض اﻻﻧﺳﺣﺎب‬if
depression. What medicine was prescribed in this case?
you stop ‘cessation’ it.
A. Paracetamol 1. Morphine:
B. Morphine hydrochloride 1. Use: Sever pain.
C. Meloxicam 2. Side effect:
D. Baralgin (Metamizole) 1. Respiratory depresstion. (Ventilatory
E. Ibuprofen dysregulation)
2. Chain-Stoke’s respiration.
3. Miosis ‫ﺿﯾق ﺣدﻗﺔ اﻟﻌﯾن‬
4. Ventilatory dysregulation
5. Constipation / Intestinal obstruction ‫اﻣﺳﺎك‬.

2. Promedol (Trimeperidine):
1. Uses:
1. Antispasmodic and urolithiasis ‫ﻣﺿﺎد ﻟﻠﺷد‬
2. Has less addiction (‫ )ادﻣﺎن‬effect so, it’s used
with kids
3. Used cardiogenic shock.
3. Fentanyl:
1. Uses: Sever pain can’t be managed with Morphine
Salah Lashin ! or promedol.
0991907360 2. Droperidol can potentiate it (‫)ﺗﻘوﯾﺔ‬
Urinary - Renal System
62 . An electronic microphotography of a Histology
Nephrones
part of a kidney shows cells with large
1. Collecting tubules:
secretory granules in their cytoplasm in the • Cuboidal epithelium.
wall of afferent and efferent arterioles. • Reabsorption of water.
What renal structure has such cells? 2. Proximal tubules:
A. Proximal part of nephron 1. Cortex
2. Tall cuboidal cells.
B. Loop of Henle
3. Brush border (microvilli – Cillia)
C. Distal part of nephron 4. Glucose Absorption.
D. Juxtaglomerular apparatus 3. Macula densa:
E. Renal corpuscle 1. Histology :
1. Distal tubule (between the afferent and efferent arteriole)
2. Very thin layer.
3. Basal membrane is very thin / absent.
2. Function: sense changes in sodium, and tell Juxtaglomerular cells.
4. Juxtaglomerular cells:
1. Function: Synthesize, store, and secrete the enzyme renin. (RAAS system).
2. Location: afferent arterioles
5. Filtration barrier: fenestrated endothelium on the basement membrane
6. Mesangial: Contains filaments and support the glomeruli (Man !)
7. Podocyte: large epithelial cell with numerous appendages.
Ureters and bladder:
• Transitional epithelium.
Bladder floor :
• Triangular shape without folds
Salah Lashin ! • Membranous part:
0991907360 • The most resistance part to catheter.
Digestive system %
63 . A histological microslide shows an organ with Stomach
1. Epithelium: Columnar glandular
mucosal lamina propria that contains simple tubular
2. Parietal cells - Body:
glands, consisting mostly of chief and parietal cells, 1. Histology: contains numerous canliculi and mitocondrial for regulating the
as well as of mucous neck cells. What type of gland work according to income of food and Histamine.
is it? 2. Function:
1. HCL (acid): increased activity causes heart burn and activates
A. Esophageal cardiac glands pepsinogen into pepsin.
B. Proper gastric glands 2. Intrinsic factor / Gasteromucoprotein: help in B12 absrobtion.
C. Pyloric gastric glands 3. Chief cells: pepsinogen
4. G cells –Antrum / pylorus:
D. Esophageal glands proper
• Gastrin: Activates Parietal cells.
E. Cardiac gastric glands • Anteroctomy or removel of pylurus part will cause issuse
5. Cervical mucocytes: Regeneration of gastric epithelium.
Dodenum
6. S cells:
• Secretin: When HCL comes to duodenum, it stimulates pancreas to secrete
water and bicarbonate to neutralize the pH, so intestine epithelium wouldn’t
be damaged.
7. I cells:
• Cholecystokinin / pancreozymin:
1. Stimulate the Pancreas → Pancreatic enzymes (juice)
2. Stimulate the gallbladder → bile acid.
8. Tubuloalveolar glands (Brunner gland): Secretes bicarbonate (alkaline).
9. Endocrine cells: Present in basal part of the duodenum.
10. Paneth cells: Antibacterial Function (ban bacteria)
11. Descendening part: single longitudinal fold + Major papilla
Salah Lashin ! Ileum
1. (Payer's patches).
0991907360 2. Meckel’s diverticulum.
64 . Autopsy of the body of a woman revealed the
following morphological changes: stenosis of the
atrioventricular opening, mitral insufficiency.
Histologically there are focal cardiosclerosis and Rheumatoid endocarditis
"blooming" Aschoff nodules in the myocardium. What is 1. The most common cause of mitral stenosis and
the most likely diagnosis? insufficiency
A. Systemic lupus erythematosus 2. Formation of Aschoff nodules.
B. Dermatomyositis
C. Scleroderma
D. Rheumatism Related
E. Polyarteritis Postmortem examination of the body of a man, who died of
heart failure, shows the following: mitral valve cusps are
deformed, thickened and fused together at the edges; myocardial
connective tissue contains diffusely scattered nodules with
patches of fibrinoid necrosis surrounded by accumulations of
macrophages that resemble multinucleated giant cells. These
nodules are surrounded by lymphocytes and single plasma cells.
What type of granuloma is it?
A. Rheumatic granuloma
B. Actinomycotic granuloma
C. Tuberculous granuloma
Salah Lashin ! D. Syphilitic granuloma
0991907360 E. Leprous granuloma
65 . Antileukocytic antibodies are detected in the blood of
a patient with leukopenia. What type of Coombs-Gell
hypersensitivity reaction developed in this case?
A. Cytotoxic
B. Delayed-type hypersensitivity
C. Stimulating
D. Immune complex-mediated
E. Anaphylactic
1. Definition: Immune damage due to Specific
antigens to the target tissue.
Type 2 hypersensitivity 2. Examples:
(antibody mediated) 1. Acute hemolytic transfusion reaction.
2. Anti-leukocytic antibodies (cytotoxic).
3. Myasthenia gravis.

Salah Lashin !
0991907360
Blood grouping (ABO)
66. A woman with the III (B), Rh (-) blood group gave Gnerenal rule:
birth to a child with the II (A) blood group. The child is • Doesn’t agglutinate with X, So answer is X.
diagnosed with hemolytic disease of newborn caused by • Doesn’t agglutinate with anything, So answer is 0.
rhesus incompatibility. What blood group and Rh are likely 1. 0 (I):
in the father? 1. Universal doner.
2. Doesn’t cause agglutination with anything. Neither
A. I (0), Rh (-)
anti-A or anti-B reagents
B. I (0), Rh (+) 2. A (II):
C. II (A), Rh (-) • No agglutination with Serum A (II)
D. II (A), Rh (+) 3. B (III):
E. III (B), Rh (+) • No agglutination with Serum B (III)
4. AB (IV):
• Agglutination reaction is positive with anti-A and
anti-B.

( II ) ( III ) ( IV) (I)

Salah Lashin !
0991907360
Blood grouping (ABO)
Gnerenal rule:
• Doesn’t agglutinate with X, So answer is X.
• Doesn’t agglutinate with anything, So answer is 0.
1. 0 (I):
1. Universal doner.
2. Doesn’t cause agglutination with anything. Neither anti-A or anti-B reagents
( II ) ( III ) ( IV) (I) 2. A (II):
• No agglutination with Serum A (II)
3. B (III):
• No agglutination with Serum B (III)
4. AB (IV):
• Agglutination reaction is positive with anti-A and anti-B.

Rh / D Factor: Agglutination reaction is positive with anti-D.


Rh / D Factor ‫اﻧت ﻋدو ﻣﺎ ﺗﺟﮭل‬
Questions
1. What happens if mother is Rh negative?
• If the mom doesn’t have Rh Factor and the baby does,
• Mom will make antibodies against Rh.
• The first child will survive; however the second child will have hemolysis.

2. how can the kid be Rh+ if mom is Rh –?


• From dad.
3. In which case would you fear about hemolysis in the kid?
• if the mom is Rh - and the dad is Rh +.
Salah Lashin !
0991907360
Blood grouping (ABO)
Gnerenal rule:
Related • Doesn’t agglutinate with X, So answer is X.
During AB0 blood grouping by using zoliclons • Doesn’t agglutinate with anything, So answer is 0.
(diagnostic monoclonal antibodies), hemagglutination 1. 0 (I):
1. Universal doner.
did not occur with any of the zoliclons. What is the
2. Doesn’t cause agglutination with anything. Neither
blood group of the patient under examination? anti-A or anti-B reagents
A. 0(I) 2. A (II):
B. A (II) • No agglutination with Serum A (II)
C. B (III) 3. B (III):
D. AB (IV) • No agglutination with Serum B (III)
4. AB (IV):
Before a surgery, the blood type of a patient is being • Agglutination reaction is positive with anti-A and
anti-B.
determined according to the ABO system, using
monoclonal antibodies against blood group antigens.
Neither anti-A and anti-B reagents caused agglutination. ( II ) ( III ) ( IV) (I)
What blood type is it?
A. A2 (II)
B. AB (IV)
C. A1 (II)
D. 0 (I)
E. B (III)
Salah Lashin !
0991907360
Blood grouping (ABO)
Gnerenal rule:
• Doesn’t agglutinate with X, So answer is X.
Determining a patient’s blood group with monoclonal test- • Doesn’t agglutinate with anything, So answer is 0.
reagents revealed positive agglutination reaction to anti-A 1. 0 (I):
and anti-B reagents, and negative reaction to anti-D. What 1. Universal doner.
blood group does this patient have? 2. Doesn’t cause agglutination with anything. Neither anti-
A. II (А) Rh (+) A or anti-B reagents
2. A (II):
B. IV (АВ) Rh (-)
• No agglutination with Serum A (II)
C. III (В) Rh (-) 3. B (III):
D. IV (АВ) Rh (+) • No agglutination with Serum B (III)
E. I (0) Rh (+) 4. AB (IV):
• Agglutination reaction is positive with anti-A and anti-
Blood group of a 30-year-old man has been determined B.
before a surgery. The blood was Rhesus-positive. • Rh / D Factor: Agglutination reaction is positive with anti-D.
Agglutination did not occur with standard 0 (I), А (II), and ( II ) ( III ) ( IV) (I)
В (III) serums. The blood belongs to the following group:
A. В (III)
B. А (II)
C. 0 (I)
D. АВ (IV)
E. -
Salah Lashin !
0991907360
Blood grouping (ABO)
Gnerenal rule:
A woman with the III (В), Rh (-) blood group gave birth • Doesn’t agglutinate with X, So answer is X.
to a child with the II (А) blood group. The child is • Doesn’t agglutinate with anything, So answer is 0.
1. 0 (I):
diagnosed with hemolytic disease of newborn caused by 1. Universal doner.
rhesus incompatibility. What blood group and Rh does the 2. Doesn’t cause agglutination with anything. Neither anti-
father have? A or anti-B reagents
A. II (А), Rh (+) 2. A (II):
B. I (0), Rh (+) • No agglutination with Serum A (II)
3. B (III):
C. III (B), Rh (+) • No agglutination with Serum B (III)
D. I (0), Rh (-) 4. AB (IV):
E. II (A), Rh (-) • Agglutination reaction is positive with anti-A and anti-
B.
A 25-year-old woman at her third pregnancy with • Rh / D Factor: Agglutination reaction is positive with anti-D.
impending miscarriage was brought to the hospital. What
combination of Rh-factor of the mother and the fetus ( II ) ( III ) ( IV) (I)
can be the cause of this condition?
A. Mother Rh (-), fetus Rh (+)
B. Mother Rh (-), fetus Rh (-)
C. Mother Rh (+), fetus Rh (-)
D. Mother Rh (+), fetus Rh (+)
E. -
Salah Lashin !
0991907360
Blood grouping (ABO)
Gnerenal rule:
• Doesn’t agglutinate with X, So answer is X.
• Doesn’t agglutinate with anything, So answer is 0.
The patient's blood group being determined using monoclonal 1. 0 (I):
test reagents. Agglutination reaction is positive with anti-A and 1. Universal doner.
2. Doesn’t cause agglutination with anything. Neither
anti-B reagents. Name the blood group of this patient: anti-A or anti-B reagents
A. AB (IV) 2. A (II):
B. B (III) Rh - • No agglutination with Serum A (II)
C. O (I) Rh + 3. B (III):
• No agglutination with Serum B (III)
D. A (II) Rh +
4. AB (IV):
• Agglutination reaction is positive with anti-A and
ABO blood group is being determined. Erythrocyte agglutination anti-B.
occurred when standard sera of group I and group II were introduced
into the blood being analysed, while group III serum caused no
agglutination. What agglutinogens do these erythrocytes have? ( II ) ( III ) ( IV) (I)
A. B
B. A
C. C
D. A and B
E. D and C

Salah Lashin !
0991907360
67 . During inflammation modelling in a frog's mesentery,
leukocyte margination and emigration through the vessel Chemotaxis
wall were observed. What factor causes this process? Inflammatory mediator released at the
A. Decreased oncotic pressure in the vessels site of injury (ex, leukotrienes) and
B. Effect of chemotaxic substances responsible for leukocyte migration
C. Increased oncotic pressure in the inflammation focus
D. Decreased hydrostatic pressure in the vessels
E. Increased hydrostatic pressure in the vessels

Salah Lashin !
0991907360
68 . An oligomycin antibiotic inhibits ATP synthase. In
what process does this enzyme take part?
A. Protein synthesis
B. Nucleic acid synthesis Electron transport chain / Respiratory chain – mitochondria
C. Substrate-level phosphorylation
D. Tricarbonic acid cycle • Function: Oxidative phosphorylation (ATP synthesis)
E. Oxidative phosphorylation • Components:
1. Cytochrome oxidase:
1. Function: Transfer of reduced equivalents to molecular
oxygen.
2. Poison:
1. Cyanide. Antidote: Nitrates → Methemoglobinemia.
2. Sodium fluoride.
2. NADH-coenzyme Q reductase:
1. Function: transfer of electrons from NADH to coenzyme Q.
2. Poison: Rotenone (insecticide).
3. ATP synthase:
• Poison: oligomycin.

Salah Lashin !
0991907360
69. An ophthalmologist used 1% mesaton (phenylephrine)
solution for diagnostic purposes (dilation of pupils for
examination of the fundus of the eye). In this case, drug-induced
mydriasis is caused by:
A. Activation of α1, adrenoceptors
B. Activation of α 2 adrenoceptors
C. Blockade of α adrenoceptors
D. Activation of muscarinic acetylcholine receptors
E. Activation of β1 adrenoceptors

Related
Due to overdosage of a vasodilator a 58-year-old patient has
developed acute vascular insufficiency. What drug would be
advisable for termination of this pathological condition?
A. Mesaton (Phenylephrine)
B. Euphyllin (Aminophylline)
C. Dopamine
Salah Lashin ! D. Cordiamin
0991907360 E. Aethimizolum (Methylamide
Related
Due to overdosage of a vasodilator a 58-year-old patient has
developed acute vascular insufficiency. What drug would be
advisable for termination of this pathological condition?
A. Mesaton (Phenylephrine)
B. Euphyllin (Aminophylline)
C. Dopamine
D. Cordiamin
E. Aethimizolum (Methylamide

A patient has elevated blood pressure due to increased vascular


tone. To lower the blood pressure in this case it is necessary to
prescribe the blockers of:
A. Muscarinic acetylcholine receptors
B. Alpha and beta adrenoceptors
C. Alpha adrenoceptors
D. Histamine H1 receptors
E. Beta adrenoceptors

Salah Lashin !
0991907360
70. A 38-year-old woman developed a bronchial asthma
attack. Which of the listed bronchial spasmolytics is
effective for emergency aid and belongs to beta-2- Terminology
adrenergic agonists? Bronchodilatation ‫ﺗوﺳﯾﻊ اﻟﺷﻌب اﻟﮭواﺋﯾﺔ‬
A. Platyphyllin
B. Adrenaline Asthma: ‫ﻣرض اﻟرﺑو‬.
C . Salbutamol • Hypersensitivity type 1: Very fast.
• Treatment:
D . Salmeterol
1. Salbutamol: is the drug of choice for asthma.
E. Ipratropium bromide • Selective Β 2 -adrenergic receptors.
• MOA: bronchodilatation = More air
2. Ephedrine:
1. hydrochloride: activates alpha and beta.
2. Alpha and beta are the receptor for sympathetic
3. Beta present in lungs: % brochodilation for
asthma.
4. Alpha present in vessels: & (vasoconstriction
for hypertention).
3. Isadrin (isoproterenol):
1. MOA: activates all beta = Good for lung beta 2
% and bad for heart (beta 1) &
4. Ipratropium:
1. Blcok M = no parasympathetic
Salah Lashin !
0991907360
71. Autopsy of the body of a 35-year-old drug addicted man Stains:
with a long history of fibrocavitary pulmonary tuberculosis 1. Sudan III: white fats
shows enlarged and dense spleen and kidneys. On section their 2. Hemomelanin: Malaria
tissues are grayish and have a "fatty" sheen. Microscopically, in 3. Bipolar/Burri-Gins: Plague
the red and white splenic pulp and in the renal glomerular 4. Congo red: Amyloidosis
interstitium and mesangium there are deposits of Congo red- 5. Ziehl-Neelsen: tuberculosis
positive masses. Diagnose the type of damage to the internal 6. Metachromasia (toluidine blue): Mucoid edema.
organs: 7. Picrofuchsin: turns yellow with fibrinoid
A. Secondary amyloidosis swelling.
B. Idiopathic amyloidosis
C. Senile amyloidosis
D. Diffuse hyalinosis
E. Local tumor-like amyloidosis

Salah Lashin !
0991907360
72. After a recovery from meningoencephalitis, the patient
presents with some residual signs, such as facial nerve
damage on the right. What group of muscles will be
affected because of this pathology?
A. Suprahyoid muscles
B. Deep muscles of the neck
C. Mimic muscles Mimic muscles
D. Strap muscles Group of facial muscles supplied by facial nerve.
E. Masticatory muscles

Salah Lashin !
0991907360
Endocrinology
Adrenal Gland - Cortext

Increase Na
Conn
reabosrbtion
Cushin Anti-inflamatory /Stree
Hyperandrogenism Male hormone

Salah Lashin !
0991907360
73 . Insufficient production of mineralocorticoids (Addison disease)
Adrenal insufficiency
is accompanied by muscle weakness caused by increased excretion
1. Primary (Addison disease)
of the following ions with urine:
2. Secondary adrenal insufficiency
A. Magnesium
B. Sodium
C. Calcium
D. Potassium
E. Hydrogen

Salah Lashin !
0991907360
A patient suffers from chronic adrenocortical failure (Addison's Adrenal insufficiency
disease or bronzed disease). What hormone is deficient, 1. Primary (Addison disease)
resulting in this pathological process? 2. Secondary adrenal insufficiency
A. Adrenaline
B. Vasopressin
C. Thyroxine
D. Insulin
E. Aldosterone

Which of the listed diuretic agents WILL NOT have diuretic


effect on a patient with Addison’s disease?
A. Spironolactone
B. Furosemide
C. Hydrochlorothiazide
D. Triamterene
E. Ethacrynic acid

Salah Lashin !
0991907360
74. During a surgery on the thyroid gland due to Basedow
disease (toxic diffuse goiter), the patient's parathyroid glands
were mistakenly removed. The patient developed seizures and
tetany. Metabolism of which bioelement was disturbed?
A. Calcium
B. Potassium
C. Magnesium Parathyroid hormone PTH and Calcium
D. Iron PTH is responsible for putting Ca in the blood &
E. Sodium
Hypocalcaemia – low PTH:
• Deficiency: seizure = convulsions ‫ﺗﺷﻧﺟﺎت‬.
• Muscle spasm.
• You need Calcium to suppress the nerves = to relax the
muscle.
• No PTH = No nerve inhibition = seizure / convulsions.
Hypercalcaemia – high PTH:
• Activates osteoclast = Demineralization / Decalcification =
osteoporosis, tooth defects.
• Increased inhibition to the nerve = Muscle weakness.
• Formation of stones (urolithiasis).
Note:
Salah Lashin ! • Calcium in the muscles causes contraction force and
0991907360 frequency while in the nerve causes inhibition.
A laboratory rat with chronic kidney failure
Parathyroid hormone PTH and Calcium
presents with osteoporosis, pathologic calcification
PTH is responsible for putting Ca in the blood &
of the internal organs, and arterial hypertension.
These disturbances are associated with increased
Hypocalcaemia – low PTH:
activity of the following hormone:
• Deficiency: seizure = convulsions ‫ﺗﺷﻧﺟﺎت‬.
A. Thyroxin
• Muscle spasm.
B. Triiodothyronine
• You need Calcium to suppress the nerves = to relax the
C. Adrenaline
muscle.
D. Calcitonin
• No PTH = No nerve inhibition = seizure / convulsions.
E. Parathyroid hormone
Hypercalcaemia – high PTH:
• Activates osteoclast = Demineralization / Decalcification =
osteoporosis, tooth defects.
• Increased inhibition to the nerve = Muscle weakness.
Note:
• Calcium in the muscles causes contraction force and
frequency while in the nerve causes inhibition.

Salah Lashin !
0991907360
Reveiw
has osteoporosis, pathologic calcification of the
internal organs and arterial hypertension. These
disturbances are associated with increased activity
Parathyroid hormone PTH and Calcium
of the following hormone:
PTH is responsible for putting Ca in the blood &
A. Calcitonin
B. Parathyroid hormone
Hypocalcaemia – low PTH:
C. Triiodothyronine
• Deficiency: seizure = convulsions ‫ﺗﺷﻧﺟﺎت‬.
D. Thyroxin
• Muscle spasm.
E. Adrenaline
• You need Calcium to suppress the nerves = to relax the
muscle.
After surgical removal of the thyroid gland, a
• No PTH = No nerve inhibition = seizure / convulsions.
patient developed numbness in the extremities.
Hypercalcaemia – high PTH:
Laboratory analysis shows hypocalcemia. What
• Activates osteoclast = Demineralization / Decalcification =
hormonal agent should be prescribed in this case?
osteoporosis, tooth defects.
A. Triiodothyronine
• Increased inhibition to the nerve = Muscle weakness.
B. Thyroidin
Note:
C. Thyroxine
• Calcium in the muscles causes contraction force and
D. Calcitonin
frequency while in the nerve causes inhibition.
E. Parathyrin

Salah Lashin !
0991907360
Parathyroid hormone PTH and Calcium
PTH is responsible for putting Ca in the blood &

Review Hypocalcaemia – low PTH:


What changes can be expected to occur in the • Deficiency: seizure = convulsions ‫ﺗﺷﻧﺟﺎت‬.
isolated heart of a toad , if excessive amount of • Muscle spasm.
calcium chloride is introduced into its perfusate? • You need Calcium to suppress the nerves = to relax the
A. Diastolic cardiac arrest muscle.
B. Decreased cardiac contraction force • No PTH = No nerve inhibition = seizure / convulsions.
C. Increased cardiac contraction frequency Hypercalcaemia – high PTH:
D. Increased cardiac contraction force and • Activates osteoclast = Demineralization / Decalcification =
frequency osteoporosis, tooth defects.
E. Increased cardiac contraction force • Increased inhibition to the nerve = Muscle weakness.
Note:
• Calcium in the muscles causes contraction force and
frequency while in the nerve causes inhibition.

Salah Lashin !
0991907360
Diabetics mellitus
75. In a 35-year-old woman diabetes mellitus was 1. Eye issues: If glucose acumulates in eyes will cause retinal disorder → glucoma and
complicated by development and progression of Microangiopathy → blindness.
2. Increased keto acids = Kussmaul’s respiration.
cataract that is likely to be caused by: 3. Polyuria.
A. Disturbed insulin-receptor binding 4. Type 1 diabetes: beta cells of pancreas don’t secrete insulin.
B. Cellular dehydration 5. Has incomplete oxidation
6. Ketoacidic coma: increased ketone bodies → Aceton smell → Metabolic acidosis
C. Glycosylation of proteins in the crystalline lens 7. Hyperglycaemic / hyperosmolar coma: Glucose ~ 20 micromole/l
and accumulation of sorbitol 8. Deficiency of insulin → proteolysis → aminoacidemia
D. Decreased oxygen affinity of the proteins 9. Renal diabetes: increased glucose levels in urine due to disturbance in glucose
E. NADPH2 deficiency and inhibition of fatty reabsorption from proximal convoluted tubules.
Tests:
acid synthesis 1. Blood sugar: Normal Glucose: 3.3 – 5.5. Diabetes: > 12.
2. Glycated haemoglobin: the test use to detect history of diabetics (≥ 6.5).
3. Glucose tolerance: is measured after fasting glucose test.
Treatment:
1. Insulin (replacment therapy).
1. Use:
1. Type 1 diabetes mellitus.
2. Inhibit the process of ketosis
3. Note: Rapid-acting insulin is used during coma.
2. Side effect: Hypoglycemia = coma (Carbohydrate starvation).
3. Antidote: Glucose or adrenaline.
2. Glibenclamide:
1. Use: type 2
2. Sulphonyl urease derivate
3. Stimulates generation of insulin from beta cells.
Salah Lashin ! 3. Metformin:
1. Use: type 2 (drug of choice).
0991907360
Diabetics mellitus
Related 1. Eye issues: If glucose acumulates in eyes will cause retinal disorder → glucoma and
A female patient complains of vision impairment. Microangiopathy → blindness.
2. Increased keto acids = Kussmaul’s respiration.
On examination she was found to have obesity, 3. Polyuria.
fasting hyperglycemia. What complication of 4. Type 1 diabetes: beta cells of pancreas don’t secrete insulin.
diabetes can cause vision impairment? 5. Has incomplete oxidation
6. Ketoacidic coma: increased ketone bodies → Aceton smell → Metabolic acidosis
A. Microangiopathy
7. Hyperglycaemic / hyperosmolar coma: Glucose ~ 20 micromole/l
B. Macroangiopathy 8. Deficiency of insulin → proteolysis → aminoacidemia
C. Atherosclerosis 9. Renal diabetes: increased glucose levels in urine due to disturbance in glucose
D. Neuropathy reabsorption from proximal convoluted tubules.
Tests:
E. Glomerulopathy 1. Blood sugar: Normal Glucose: 3.3 – 5.5. Diabetes: > 12.
2. Glycated haemoglobin: the test use to detect history of diabetics (≥ 6.5).
3. Glucose tolerance: is measured after fasting glucose test.
Treatment:
1. Insulin (replacment therapy).
1. Use:
1. Type 1 diabetes mellitus.
2. Inhibit the process of ketosis
3. Note: Rapid-acting insulin is used during coma.
2. Side effect: Hypoglycemia = coma (Carbohydrate starvation).
3. Antidote: Glucose or adrenaline.
2. Glibenclamide:
1. Use: type 2
2. Sulphonyl urease derivate
3. Stimulates generation of insulin from beta cells.
Salah Lashin ! 3. Metformin:
1. Use: type 2 (drug of choice).
0991907360
76. Because of a long-term starving rate a person's Question Sequence
glomerular filtration increased by 20%. What is the What will the kidney do when protein is less and water is more?
most likely cause of the change in filtration rate in 1. More filteration =
this case? 2. Loss of water =
A. Increased renal plasma flow 3. Increased ratio of protein to water.
B. Increased filtration coefficient 4. Increase oncotic pressure.
C. Increased permeability of the renal filter
D. Increased systemic arterial pressure
E. Decreased oncotic plasma pressure

Review
A man presents with glomerular filtration rate
of 180 ml/min., while norm is 125±25 ml/min.
The likely cause of it is the decreased:
A. Effective filtration pressure
B. Hydrostatic blood pressure in the
glomerular capillaries
C. Renal blood flow
D. Plasma oncotic pressure
Salah Lashin !
E. Permeability of the renal filter
0991907360
77. ECG of a patient with hyperthyroidism shows an
increased heart rate. What ECG element will be shortened,
indicating this?
A. QRS complex
B. R-R interval
C. P-Q interval
D. P-Q segment
E. P-T interval

Check arrhythmia!
Salah Lashin !
0991907360
78. Urinalysis of a patient with acute cystitis shows
leukocytes and a large number of Gram-negative bacilli.
Inoculation has resulted in the growth of mucous colonies
that produce a green soluble pigment. What microorganism Pseudomonas aeruginosa
is the most likely cause of the patient's disorder? • Feature:
A. Pseudomonas aeruginosa 1. Green blue (cyan) pus.
B. Salmonella enteritidis 2. Encapsulated
C. Klebsiella pneumoniae 3. Exotoxin
D. Proteus mirabilis 4. Oxidase Positive
E. Escherichia coli 5. Infection is common with burn
Patients
• Disease:
1. Pneumonia
2. UTI
3. Osteomyelitis
4. Otitis Externa
• Treatment:
• Amicacin and Carbenicillin

Salah Lashin !
0991907360
Pseudomonas aeruginosa
• Feature:
1. Green blue (cyan) pus.
2. Encapsulated
3. Exotoxin
4. Oxidase Positive
5. Infection is common with burn
Patients
• Disease:
1. Pneumonia
2. UTI
3. Osteomyelitis
4. Otitis Externa
• Treatment:
• Amicacin and Carbenicillin

Salah Lashin !
0991907360
During bacteriological examination of the purulent discharge
A woman hospitalized in the surgical department
obtained from a postoperative wound an inoculation on meat
complains of pain in her lower abdomen and in the small
infusion agar has been performed. The inoculation has
of her back, frequent and painful urination. Urine culture
resulted in large colorless mucous colonies that in 24 hours
test revealed gram-negative oxidase-positive bacilli that
with exposure to sunlight developed green-blue pigmentation
formed green mucoid colonies with a specific smell.
and smell of honey or jasmine. Bacterioscopy revealed gram-
What causative agent can be suspected?
negative lophotrichea. What bacterial culture is contained in
A. Pseudomonas aeruginosa
purulent discharge?
B. Proteus mirabilis
A. Klebsiella osaenae
C. E. coli
B. Proteus vulgaris
D. Str pyogenes
C. Pseudomonas aeruginosa
D. Streptomyces griseus
E. Mycoplasma pneumonic A patient being treated in the
E. Brucella abortus
burns department has suppurative complication. The pus
is of bluish-green colour that is indicative of infection
A patient in the oral surgery department has got purulent
caused by Pseudomonas aeruginosa. What factor is
complication. Bacteriological analysis of the wound material
typical for this causative agent?
found a culture that produces cyan pigment. What
A. Gram-negative stain
microorganism is the most probable causative agent?
B. Presense of spores
A. Pseudomonas aeruginosa
C. Coccal form
B. Proteus vulgaris
D. Cell pairing
C. Bacillus subtilis
E. Mycelium formation
Salah Lashin
D. Klebsiella !
pneumoniae
0991907360 epidermidis
E. Staphylococcus
Related
A 60-year-old patient was hospitalised to the surgical department
because of infection caused by blue pus bacillus (Pseudomonas
aeruginosa) which is sensative to penicillin antibiotics. Indicate
which of the given penicillins has marked activity to the
Pseudomonas aeruginosa?
A. Carbenicillin disodium
B. Benzylpenicillin
C. Phenoxymethylpenicillin
D. Oxacillin
E. Methicillin

A patient suffers from severe postoperative pseudomonadous


infection. What of the following antibiotics should be administered
in this case?
A. Amicacin sulfate
B. Benzylpenicillin
C. Cephazolin
D. Erythromycin
E. Doxycycline
Salah Lashin !
0991907360
79. Excessive intake of carbohydrates (600 g per day) that
surpasses the energy needs of a 28-year-old person will Krebs / TCA / Citric acid cycles
activate the process of:
A. Beta-oxidation of fatty acids 1. Start with the the process of oxidative decarboxylation =
B. Lipogenesis 1. Conversion of pyruvate into Acetyl CoA by the
C. Glycolysis enzyme:
D. Gluconeogenesis
2. Pyruvate dehydrogenase / decarboxylase
E. Lipolysis
3. This enzyme requires 5 coenzyme to work:
Thymine (B1), FAD (B2), NAD (B3), CoA, Lipoic
acid. (The Lovely Coenzymes For Nerds')

2. Acetyl CoA should meet with oxaloacetate to form citric


acid.
3. Pyruvate carboxylase: produce Oxaloacetate from and to
pyruvate for krebs cycle / gluconeogenesis.
4. FAD (Flavin cofactor) and NAD (niacin cofactor).
Carries the energy in krebs cycle inform of FADH2 and
NADH.
5. Amphibolic: has to pathways: lipid and ATP synthesis.
Salah Lashin ! 6. Breaking down of GABA converts to succinate and
0991907360 enters into Krebs cycle
Glycolysis Collection

Salah Lashin !
0991907360
Glycolysis ‫ﺗﻛﺳﯾر اﻟﺳﻛر‬

Anerobic glycolysis
Glucose
(only 2 ATP)
Pyruvate

Anerobic glycolysis (No O2) Lactate No Oxygen?


1. Porduce 2 ATP.
2. One of the 2 ATP (50%), is produced by the enzyme Yes )
Pyruvate Kinase.
3. Red blood cells only have Anaerobic Glycolysis ❤ (only 2 Pyruvate dehydrogenase / decarboxylase
Acetyl CoA
ATP). No Aerobic glycolysis. No mitochondria.

Aerobic glycolysis
4. No Pyruvate kinase = RBC loss of 50% of ATP = cell
membrane rupture (disintegrate) = haemolytic anaemia. Krebs cycles / TCA
(NADH/ FADH)

Electron transport chain


(mitochondria)

36-38 ATP
Salah Lashin ! $
0991907360
Glycolysis ‫ﺗﻛﺳﯾر اﻟﺳﻛر‬

Anerobic glycolysis
Untrained people often have muscle pain after sprints as a result Glucose
of lactate accumulation. This can be caused by intensification of (only 2 ATP)
the following biochemical process:
Pyruvate
A. Lipogenesis
B. Gluconeogenesis
C. Pentose phosphate pathway Lactate No Oxygen?
D. Glycolysis
E. Glycogenesis Yes )

Diseases of respiratory system and circulatory disorders impair Pyruvate dehydrogenase / decarboxylase
Acetyl CoA
the transport of oxygen, thus causing hypoxia. Under these
conditions the energy metabolism is carried out by anaerobic

Aerobic glycolysis
glycolysis. As a result, the following substance is generated and Krebs cycles / TCA
accumulated in blood: (NADH/ FADH)
A. Lactic acid
B. Pyruvic acid
C. Glutamic acid Electron transport chain
D. Citric acid (mitochondria)
E. Fumaric acid
36-38 ATP
Salah Lashin ! $
0991907360
Glycolysis ‫ﺗﻛﺳﯾر اﻟﺳﻛر‬

Anerobic glycolysis
A 7-year-old boy is diagnosed with anemia. Laboratory analysis Glucose
detects pyruvate kinase deficiency in his erythrocytes. What
process is disturbed in this boy playing the main role in anemia (only 2 ATP)
development in this case? Pyruvate
A. Anaerobic glycogenolysis
B. Gluconeogenesis No
Lactate Oxygen?
C. Deaminization of amino acids
D. Anaerobic glycolysis
E. Decarboxylation of amino acids Yes )
Pyruvate dehydrogenase / decarboxylase
General catabolism pathway of biological macromolecules Acetyl CoA
includes besides tricarbonic acid cycle and mitochondrial

Aerobic glycolysis
respiratory chain, the process of pyruvate oxidative
Krebs cycles / TCA
decarboxylation. What is the product of pyruvate oxidative
(NADH/ FADH)
decarboxylation?
A. Citrate
B. Lactate
Electron transport chain
C. Alpha ketoglutarate
(mitochondria)
D. Acetyl-CoA
E. Malonyl-CoA
36-38 ATP $
Salah Lashin !
0991907360
Glycolysis ‫ﺗﻛﺳﯾر اﻟﺳﻛر‬

Anerobic glycolysis
Glucose
A 7-year-old girl has signs of anemia. Laboratory testing (only 2 ATP)
determined the deficiency of pyruvate kinase in her
erythrocytes. In this case the main role in anemia Pyruvate
development belongs to the disturbance of a certain process.
What process is disturbed in this girl? Lactate No Oxygen?
A. Oxidative phosphorylation
B. Tissue respiration
Yes )
C. Peroxide decomposition
D. Anaerobic glycolysis Pyruvate dehydrogenase / decarboxylase
Acetyl CoA
E. Amino acid deamination

Aerobic glycolysis
There is a large amount of glucose oxidation metabolites Krebs cycles / TCA
dissolved in the cytoplasm of myocytes. Name one such (NADH/ FADH)
metabolite that converts directly into lactate:
A. Glycerophosphate
B. Oxaloacetate Electron transport chain
C. Glucose 6 phosphate (mitochondria)
D. Fructose 6 phosphate
E. Pyruvate
36-38 ATP $
Salah Lashin !
0991907360
Glycolysis ‫ﺗﻛﺳﯾر اﻟﺳﻛر‬
People, who for a long time remained in hypodynamic state, develop

Anerobic glycolysis
intense pain in the muscles after a physical exertion. What is the most Glucose
likely cause of this pain?
(only 2 ATP)
A. Increased content of ADP in muscles
B. Decreased content of lipids in muscles Pyruvate
C. Intensive breakdown of muscle proteins
D. Accumulation of lactic acid in muscles
E. Accumulation of creatinine in muscles Lactate No Oxygen?

When blood circulation in the damaged tissue is restored, lactate


accumulation stops and glucose consumption decelerates. These
Yes )
metabolic changes are caused by activation of the following process: Pyruvate dehydrogenase / decarboxylase
A. Glycogen biosynthesis Acetyl CoA
B. Anaerobic glycolysis

Aerobic glycolysis
C. Lipolysis
D. Gluconeogenesis Krebs cycles / TCA
E. Aerobic glycolysis (NADH/ FADH)

An experiment demonstrates that Jensen sarcoma leads to a significant


increase in glucose uptake from the tumor afferent artery, while tumor Electron transport chain
efferent vein has high levels of lactic acid. This phenomenon indicates:
(mitochondria)
A. Intensified anaerobic glycolysis
B. Intensified oxidizing processes
C. Intensified protein oxidation
36-38 ATP $
D. Salah
Decreased
Lashin !
anaerobic glycolysis
E. 0991907360
Decreased oxidizing processes
A 6-year—old girl exhibits marked sign of hemolytic Terminology
anemia biochemical analysis her erythrocytes show Oxidants / free radiacal ‫اﻟﻣؤﻛﺳدات‬
deficiency of glucose 6- phosphate dehydrogenase enzyme Counteraction ‫ﻣﺿﺎدة‬
what metabolic process is disturbed in this patient and has
leading role in the development of this pathology:
A. Pentose phosphate pathway Glycolysis
ATP
B. Tissue respiration ‫ﺗﻛﺳﯾر اﻟﺳﻛر‬
C. Gluconeogenesis

‫ﺗﻔرﯾﻌﺔ‬
D. Anaerobic glycolysis G6PD
Enzyme
Oxidative phosphorylation

Pentose phosphate pathway


E.

NADPH
A patient suffering from malaria has developed hemolytic Activates
anemia after taking primaquine antimalarial drug.
Hereditary insufficiency of the following enzyme in
Glutathione
erythrocytes will be observed in this case:
A. Glucose 6-phosphate dehydrogenase
B. Triosephosphate isomerase
C. Lipase Protect RBC Form?
D. Fructose 1-phosphate aldolase Oxidant damage of lipid peroxidation (h2o2)
Salah Lashin !
E. Phosphofructokinase = haemolysis
0991907360
It is known that individuals with genetically caused deficiency Terminology
of glucose- 6-phosphate dehydrogenase may develop RBC Oxidants / free radiacal ‫اﻟﻣؤﻛﺳدات‬
hemolysis in response to the administration of some antimalarial Counteraction ‫ﻣﺿﺎدة‬
drugs. Manifestation Manifestation of adverse reactions to drugs
is called:
A. Sensibilization Glycolysis
ATP
B. Allergy ‫ﺗﻛﺳﯾر اﻟﺳﻛر‬
C. Idiosyncrasy
D. Tachyphylaxis

‫ﺗﻔرﯾﻌﺔ‬
E. Tolerance G6PD
Enzyme

Pentose phosphate pathway


A 3-year-old child with elevated body temperature has taken
aspirin and developed increased hemolysis of erythrocytes. In NADPH
this case hemolytic anemia can be caused by congenital
Activates
deficiency of the following enzyme:
A. Glucose 6-phosphate dehydrogenas
Glutathione
B. Glycerol-phosphate dehydrogenase
C. Glucose 6-phosphatase
D. Glycogen phosphorylase
E. Gamma- glutamyl transferase Protect RBC Form?
Oxidant damage of lipid peroxidation (h2o2)
Salah Lashin ! = haemolysis
0991907360
Terminology
Oxidants / free radiacal ‫اﻟﻣؤﻛﺳدات‬
A 7-year-old child presents with marked signs of hemolytic Counteraction ‫ﻣﺿﺎدة‬
anemia. Biochemical analysis of erythrocytes determined low
concentration of NADPH and reduced glutathione. What
enzyme is deficient in this case leading to the
biochemical changes and their clinical manifestations? Glycolysis
ATP
A. Hexokinase ‫ﺗﻛﺳﯾر اﻟﺳﻛر‬
B. Fructokinase
C. Pyruvate kinase

‫ﺗﻔرﯾﻌﺔ‬
D. Glucose-6-phosphate dehydrogenase G6PD
Enzyme
E. Lactate dehydrogenase

Pentose phosphate pathway


It is known that pentose-phosphate pathway actively functions NADPH
in the erythrocytes. What is the main function of this metabolic Activates
pathway in the erythrocytes?
A. Activation of microsomal oxidation Glutathione
B. Counteraction to lipid peroxidation
C. Neutralization of xenobiotics
D. Oxidation of glucose into lactate
Protect RBC Form?
E. Increase of lipid peroxidation
Oxidant damage of lipid peroxidation (h2o2)
Salah Lashin ! = haemolysis
0991907360
Glycolysis ‫ﺗﻛﺳﯾر اﻟﺳﻛر‬

Anerobic glycolysis
Krebs / TCA / Citric acid cycles
Glucose
1. Start with the the process of oxidative decarboxylation = (only 2 ATP)
1. Conversion of pyruvate into Acetyl CoA by the Pyruvate
enzyme:
2. Pyruvate dehydrogenase / decarboxylase Oxygen?
3. This enzyme requires 5 coenzyme to work:
Thymine (B1), FAD (B2), NAD (B3), CoA, Lipoic Yes )
acid. (The Lovely Coenzymes For Nerds')
Pyruvate dehydrogenase / decarboxylase
Acetyl CoA
2. Acetyl CoA should meet with oxaloacetate to form citric

Aerobic glycolysis
acid.
Krebs cycles / TCA
3. Pyruvate carboxylase: produce Oxaloacetate from and to (NADH/ FADH)
pyruvate for krebs cycle / gluconeogenesis.
4. FAD (Flavin cofactor) and NAD (niacin cofactor).
Carries the energy in krebs cycle inform of FADH2 and Electron transport chain
NADH. (mitochondria)
5. Amphibolic: has to pathways: lipid and ATP synthesis.
6. Breaking down of GABA converts to succinate and
Salah into
Lashin ! cycle 36-38 ATP
enters Krebs
0991907360 $
Oxidative decarboxylation of pyruvic acid is catalyzed by a Krebs / TCA / Citric acid cycles
multienzyme complex with several functionally linked
coenzymes. Name this complex: 1. Start with the the process of oxidative decarboxylation =
A. Thymidine diphosphate (TDP), flavin adenine dinucleotide 1. Conversion of pyruvate into Acetyl CoA by the
(FAD), coenzyme A (CoASH), nicotine amide adenine enzyme:
dinucleotide (NAD), lipoic acid 2. Pyruvate dehydrogenase / decarboxylase
B. Flavin adenine dinucleotide (FAD),tetrahydrofolic acid,
pyridoxal-5-phosphate,thymidine diphosphate (TDP), choline 3. This enzyme requires 5 coenzyme to work:
C. Nicotine amide adenine dinucleotide (NAD), pyridoxal-5- Thymine (B1), FAD (B2), NAD (B3), CoA, Lipoic
phosphate, thymidine diphosphate (TDP), methylcobalamin, acid. (The Lovely Coenzymes For Nerds')
biotin
D. Coenzyme A (CoASH), flavin adenine dinucleotide (FAD),
pyridoxal-5- phosphate, tetrahydrofolic acid, carnitine 2. Acetyl CoA should meet with oxaloacetate to form citric
E. Lipoic acid, tetrahydrofolic acid, pyridoxal-5-phosphate, acid.
methylcobalamin
3. Pyruvate carboxylase: produce Oxaloacetate from and to
It has been determined that one of a pesticide components is pyruvate for krebs cycle / gluconeogenesis.
sodium arsenate that blocks lipoic acid. Enzyme activity can be 4. FAD (Flavin cofactor) and NAD (niacin cofactor) are the
impaired by this pesticide. Name this enzyme: products of krebs cycle wich carrie the energy to the
A. Pyruvate dehydrogenase complex electron trasport chain for energy production.
B. Microsomal oxidation 5. Amphibolic: has to pathways: lipid and ATP synthesis.
C. Methemoglobin reductase
D. Glutathione peroxidase
6. Breaking down of GABA converts to succinate and
E. Glutathione !
reductase
Salah Lashin enters into Krebs cycle
0991907360
Patients with alcoholism receive the bulk of their calories Krebs / TCA / Citric acid cycles
with alcoholic drinks. They may develop a characteristic
thiamine deficiency (Wernicke-Korsakoff syndrome) that 1. Start with the the process of oxidative decarboxylation =
causes nervous system dysfunction, psychoses and memory 1. Conversion of pyruvate into Acetyl CoA by the
loss. This process is associated with decreased activity of enzyme:
the following enzyme:
A. Transaminase 2. Pyruvate dehydrogenase / decarboxylase
B. Hexokinase 3. This enzyme requires 5 coenzyme to work:
C. Pyruvate dehydrogenase Thymine (B1), FAD (B2), NAD (B3), CoA, Lipoic
D. Alcohol dehydrogenase acid. (The Lovely Coenzymes For Nerds')
E. Aldolase
2. Acetyl CoA should meet with oxaloacetate to form citric
Congenital pyruvate carboxylase deficiency causes physical
acid.
and mental retardation in children and leads to early death.
It is characterized by lactic acidemia, lactaciduria, and a 3. Pyruvate carboxylase: produce Oxaloacetate from and to
number of metabolic disorders. Among others, inhibition of pyruvate for krebs cycle / gluconeogenesis.
the following 4. FAD (Flavin cofactor) and NAD (niacin cofactor) are the
occurs: products of krebs cycle wich carrie the energy to the
A. Citric acid cycle and gluconeogenesis electron trasport chain for energy production.
B. Glycolysis and glycogenolysis 5. Amphibolic: has to pathways: lipid and ATP synthesis.
C. Glycogenesis and glycogenolysis 6. Breaking down of GABA converts to succinate and
D. Lipolysis and lipogenesis enters into Krebs cycle
Salah Lashin !
E. Pentose-phosphate pathway and glycolysis
0991907360
Acetyl CoA
A 60-year-old patient with diabetes mellitus was
Pyruvate dehydrogenase / decarboxylase
found to have ketoacidosis. Biochemically
ketoacidosis in diabetes mellitus leads to decreased Krebs cycles / TCA
acetyl-CoA utilization due to deficiency of: (NADH/ FADH)
A. Oxaloacetate
B. Acetate
C. Succinate
D. Aspartate
E. Alanine

The TCA cycle is a pathway designed to burn away


carboxylic acids as two moles of CO2.TCA cycle to
be continuous requires the regeneration of:
A. Oxaloacetic acid
B. Pyruvic acid
C. α-oxoglutaric acid
D. Malic acid
E. Succinic acid

Salah Lashin !
0991907360
Krebs / TCA / Citric acid cycles

What coenzyme of flavin-dependent dehydrogenases 1. Start with the the process of oxidative decarboxylation =
participates in the reactions of tricarboxylic acid cycle?
1. Conversion of pyruvate into Acetyl CoA by the
A. Flavin adenine dinucleotide (FAD)
enzyme:
B. Flavin mononucleotide (FMN)
C. Nicotinamide-adenine dinucleotide (NAD+) 2. Pyruvate dehydrogenase / decarboxylase
D. Thymidine diphosphate (TDP) 3. This enzyme requires 5 coenzyme to work:
E. Heme Thymine (B1), FAD (B2), NAD (B3), CoA, Lipoic
acid. (The Lovely Coenzymes For Nerds')
Examination of a patient revealed II grade obesity. It is
known that he consumes a lot of sweets and rich food, has
2. Acetyl CoA should meet with oxaloacetate to form citric
sedentary way of life. That’s why anabolic metabolism
acid.
has the priority in his organism. Which of the following
pathways is amphibolic? 3. Pyruvate carboxylase: produce Oxaloacetate from and to
A. Cycle of tricarboxylic acids pyruvate for krebs cycle / gluconeogenesis.
B. Glyconeogenesis 4. FAD (Flavin cofactor) and NAD (niacin cofactor) are the
C. Lipolysis products of krebs cycle wich carrie the energy to the
D. Glycolysis electron trasport chain for energy production.
E. Fatty acids oxidation 5. Amphibolic: has to pathways: lipid and ATP synthesis.
6. Breaking down of GABA converts to succinate and
Salah Lashin ! enters into Krebs cycle
0991907360
Decarboxylation of glutamate induces production of Krebs / TCA / Citric acid cycles
gamma-aminobutyric acid (GABA ) neurotransmitter. After
breakdown, GABA is converted into a metabolite of the 1. Start with the the process of oxidative decarboxylation =
citric acid cycle, that is: 1. Conversion of pyruvate into Acetyl CoA by the
A. Succinate enzyme:
B. Citric acid
2. Pyruvate dehydrogenase / decarboxylase
C. Malate
D. Fumarate 3. This enzyme requires 5 coenzyme to work:
E. Oxaloacetate Thymine (B1), FAD (B2), NAD (B3), CoA, Lipoic
acid. (The Lovely Coenzymes For Nerds')

2. Acetyl CoA should meet with oxaloacetate to form citric


acid.
3. Pyruvate carboxylase: produce Oxaloacetate from and to
pyruvate for krebs cycle / gluconeogenesis.
4. FAD (Flavin cofactor) and NAD (niacin cofactor) are the
products of krebs cycle wich carrie the energy to the
electron trasport chain for energy production.
5. Amphibolic: has to pathways: lipid and ATP synthesis.
6. Breaking down of GABA converts to succinate and
Salah Lashin ! enters into Krebs cycle
0991907360
Electron transport chain / Respiratory chain – mitochondria

• Function: Oxidative phosphorylation (ATP synthesis)


• Components:
1. Cytochrome oxidase:
1. Function: Transfer of reduced equivalents to molecular
oxygen.
2. Poison:
1. Cyanide. Antidote: Nitrates → Methemoglobinemia.
2. Sodium fluoride.
2. NADH-coenzyme Q reductase:
1. Function: transfer of electrons from NADH to coenzyme Q.
2. Poison: Rotenone (insecticide).
3. ATP synthase:
• Poison: oligomycin.

Salah Lashin !
0991907360
Name the supramolecular multienzyme complex that is Rotenone is known to inhibit respiratory chain. What
integrated into the lipid layer of inner mitochondrial complex of mitochondrial respiratory chain is inhibited by
membrane that creates conditions for redox reactions: this substance?
A. Carboxypeptidase A. NADH-coenzyme Q reductase
B. Hexokinase B. Cytochrome oxidase
C. Pyruvate kinase C. Coenzyme Q - cytochrome c reductase
D. Respiratory chain D. Succinate-coenzyme Q reductase
E. G protein transducer E. Adenosine triphosphate synthetase

Cytochrome oxidase is a hemeprotein that is an end Electrons release their energy, when they are being
component of the mitochondrial respiratory chain. What transported in the course of tissue respiration. This energy is
reaction is catalyzed with this enzyme? used for the following process:
A .Transfer of reduced equivalents to molecular oxygen A. Oxidative phosphorylation
B. Cytochrome synthesis B. Lipid mobilization
C. Transfer of reduced equivalents to ubiquinone C. Microsomal oxidation
D. Cytochrome splicing D. Peroxidation
E. Adenosine triphosphate synthesis E. Substrate-level phosphorylation

Salah Lashin !
0991907360
Urea / Ornithine cycle
80 . Ammonia is extremely toxic for human CNS. What is
the main way of ammonia neutralization in the nervous • Function: converte toxic amonium which is very toxic
tissue? speacially for the brian into less toxic urea.
• Location: Liver.
A. Urea synthesis
• Amonia Transport
B. Ammonium salts synthesis
1. Muscle: Alanine
C. Glutamine synthesis 2. Intestine: NH3 – portal vein
D. Transamination 3. Most other organs: glutamine
E. Formation of paired compounds
• Genetic disorders:
1. Deficiency of Carbamoyl phosphate synthetase I:
• → not forming of Carbamoyl phosphate →
accumulation of ammonia in blood →
hyperammonemia → Symptoms.
2. Deficiency of Ornithine transcarbamoylase:
• → not forming citrulline (joining of Carbamoyl
phosphate + Ornithine) →
• Accumulation of Carbamoyl phosphate ⬆
• Enters pyrimidine synthesis pathway → increase
formation of orotic acid → orotic aciduria.
3. Deficiency of argininosuccinate synthase 1:
• Accumulation of citrulline → Citrullinemia.

• Symptom: lethargy, vomiting, and hyperventilation; and if not


Salah Lashin ! treated, progress to hepatic coma, respiratory failure, and death.
0991907360 • Indicators: decrease BUN (blood urea nitrogen).
Damage
Clotting Cascade
Stage one:
• Goal: Prothrombinase formation.
Platalets Clotting cascade Thrombus
• Consists of:
1. Internal pathway:
• Factors XII XI, IX, VIII
• Deficiencies:
• Hemophilia A:
• Factor VIII → Thromboplastin deficiency.
• Symptoms: Deep bleeding (ex, hemarthrosis)
• Hemophilia B: factor IX
• Note: Both are X linked recessive.
2. External mechanism:
• III – VIIa – Xa ( 3+7=10 ").

Stage Two:
• Goal: Thrombin formation
• How?: prothrombinase and Ca and vitamin K convert prothrombin into
Thrombin

Stage Three:
• Goal: Fibrin formation
• How? Lashin !
Thrombin
Salah and Ca convert Fibrinogen into Fibrin,
• Pathology: Pancreatitis will destroy fibrin (activation of fibrinolysis)
0991907360
Damage
81. A 70-year-old man presents with atherosclerosis that
was complicated by leg thrombosis. He developed a
gangrene in the toes of his left foot. The onset of thrombus
Platalets Clotting cascade Thrombus
formation is most likely associated with:
A. Adhesion of platelets
B. Fibrinogen to fibrin conversion
C. Decreased heparin synthesis
D. Prothrombin io thrombin conversion
E. Prothrombinase activation

Related
A 70-year-old patient suffers from atherosclerosis complicated
by the lower limb thrombosis that has caused gangrene on his
left toes. What is the most likely cause of the thrombosis origin?
A. Thrombocyte adhesion
B. Prothrombinase activation
C. Transformation of prothrombin into thrombin
Salah Lashin ! D. Transformation of fibrinogen into fibrin
0991907360 E. Impaired heparin synthesis
82.. Microtraumas of oral mucosa occur daily during eating.
However, bleeding in such cases quickly stops because of:
A. Thromboplastin
B. Mucin
C. Heparin
D. Heparin anti-factor
E. Lysozyme

Salah Lashin !
0991907360
After pancreatic surgery the patient developed Clotting Cascade
hemorrhagic syndrome with disturbed 3rd stage of blood • Stage one:
clotting. What will be the most likely mechanism of the • Goal: Prothrombinase formation.
• Consists of:
hemostatic disorder?
1. Internal pathway:
A. Fibrinolysis activation • Factor XII XI, IX, VIII
B. Decrease of prothrombin synthesis 2. External mechanism:
C. Decrease of fibrinogen synthesis • III – VIIa – Xa ( 3+7=10 * )
D. Qualitative abnormalities of fibrinogenesis • Stage Two:
E. Fibrin-stabilizing factor deficiency • Goal: prothrombinase and Ca and vitamin K
convert prothrombin into Thrombin.
. A patient is diagnosed with hereditary coagulopathy that • Stage Three:
is characterised by factor VIII deficiency. Specify the • Goal: Thrombin and Ca convert Fibrinogen into
Fibrin.
phase of blood clotting during which coagulation will be
• Pathology: Pancreatitis will destroy fibrin
disrupted in the given case: (activation of fibrinolysis)
A. Thromboplastin formation
B. Thrombin formation Note
C. Fibrin formation Choice C and D will be valid If there is a
D. Clot retraction production problem in the is liver or genetics.
E. -Salah Lashin !
0991907360
Clotting Cascade
Stage one:
A patient with tissue trauma was taken a blood sample for
• Goal: Prothrombinase formation.
the determination of blood clotting parameters. Specify the
• Consists of:
right sequence of extrinsic pathway activation.
1. Internal pathway:
A. III – VIIa – Xa
• Factors XII XI, IX, VIII
B. III – IV – Xa • Deficiencies:
C. IV – VIII: TF – Xa
• Hemophilia A: Factor VIII →
D. IV – VIIa – Xa
Thromboplastin deficiency.
E. III – VIII: TF – Xa
• Hemophilia B: factor IX
• Note: Both are X linked recessive.
A 3-year-old boy with pronounced hemorrhagic syndrome
2. External mechanism:
has no antihemophilic globulin A (factor VIII) in the blood • III – VIIa – Xa ( 3+7=10 *).
plasma. Hemostasis has been impaired at the following Stage Two:
stage: • Goal: prothrombinase and Ca and vitamin K convert
A. Internal mechanism of prothrombinase activation prothrombin into Thrombin.
B. External mechanism of prothrombinase activation Stage Three:
C. Conversion of prothrombin to thrombin • Goal: Thrombin and Ca convert Fibrinogen into Fibrin.
D. Conversion of fibrinogen to fibrin • Pathology: Pancreatitis will destroy fibrin (activation of
E. Blood clot retraction fibrinolysis)
Salah Lashin !
0991907360
After a tooth extraction, a child developed bleeding that Clotting Cascade
persisted for the next 6 hours. Analysis of the hemostasis Stage one:
system revealed a sharp decrease in the levels of blood • Goal: Prothrombinase formation.
coagulation factor VIII. What pattern of inheritance does • Consists of:
this disease have? 1. Internal pathway:
A. Incomplete dominance • Factors XII XI, IX, VIII
B. Autosomal dominant • Deficiencies:
C. Autosomal recessive • Hemophilia A: Factor VIII →
D. Sex chromosome-linked Thromboplastin deficiency.
E. Polygenic • Hemophilia B: factor IX
• Note: Both are X linked recessive.
A patient suffers from the haemorrhagic syndrome that 2. External mechanism:
shows itself in frequent nasal bleedings, posttraumatic and • III – VIIa – Xa ( 3+7=10 *).
spontaneous intracutaneous and intra-articular Stage Two:
haemorrhages. After a laboratory study a patient was • Goal: prothrombinase and Ca and vitamin K convert
diagnosed with the type B haemophilia. This disease is prothrombin into Thrombin.
provoked by the deficit of the following factor of blood Stage Three:
coagulation: • Goal: Thrombin and Ca convert Fibrinogen into Fibrin.
A. IX • Pathology: Pancreatitis will destroy fibrin (activation of
B. VIII fibrinolysis)
C. XI
D. V
Salah Lashin !
E. VII
0991907360
Clotting Cascade
Stage one:
A couple has a son with haemophilia. The parents are • Goal: Prothrombinase formation.
healthy but the maternal grandfather also has • Consists of:
haemophilia. Specify the type of inheritance: 1. Internal pathway:
A. Recessive sex-linked • Factors XII XI, IX, VIII
B. Recessive autosomal • Deficiencies:
C. Dominant sex-linked • Hemophilia A: Factor VIII →
D. Semidominance Thromboplastin deficiency.
E. Autosomal dominant • Hemophilia B: factor IX
• Note: Both are X linked recessive.
A patient was diagnosed with a monogenic hereditary 2. External mechanism:
disease. Name this disease: • III – VIIa – Xa ( 3+7=10 *).
A. Hemophilia Stage Two:
B. Hypertension • Goal: prothrombinase and Ca and vitamin K convert
C. Peptic ulcer disease of the stomach prothrombin into Thrombin.
D. Poliomyelitis Stage Three:
• Goal: Thrombin and Ca convert Fibrinogen into Fibrin.
E. Hymenolepiasis
• Pathology: Pancreatitis will destroy fibrin (activation of
fibrinolysis)
Salah Lashin !
0991907360
Damage

A 70-year-old patient suffers from atherosclerosis


complicated by the lower limb thrombosis that has Platalets Clotting cascade Thrombus
caused gangrene on his left toes. What is the most
likely cause of the thrombosis origin?
A. Thrombocyte adhesion
B. Prothrombinase activation
C. Transformation of prothrombin into thrombin
D. Transformation of fibrinogen into fibrin
E. Impaired heparin synthesis

After pancreatic surgery the patient developed


hemorrhagic syndrome with disturbed 3rd stage of
blood clotting. What will be the most likely
mechanism of the hemostatic disorder?
A. Fibrinolysis activation
B. Decrease of prothrombin synthesis
C. Decrease of fibrinogen synthesis
D. Qualitative abnormalities of fibrinogenesis
E. Fibrin-stabilizing factor deficiency

Salah Lashin !
0991907360
Clotting Cascade
Stage one:
• Goal: Prothrombinase formation.
• Consists of:
1. Internal pathway:
• Factors XII XI, IX, VIII
• Deficiencies:
A newborn baby has numerous hemorrhages. Blood • Hemophilia A: Factor VIII → Thromboplastin
coagulation tests reveal increased prothrombin time. deficiency.
The child is most likely to have a disorder of the • Hemophilia B: factor IX
following biochemical process: • Note: Both are X linked recessive.
A. Production of gammacarboxyglutamate 2. External mechanism:
B. Conversion of homocysteine to methionine • III – VIIa – Xa ( 3+7=10 *).
C. Conversion of methylmalonyl CoA to succinyl
CoA Stage Two:
D. Degradation of glutathione • Goal: Thrombin formation
E. Hydroxylation of proline • How?: prothrombinase and Ca and vitamin K convert prothrombin
into Thrombin

Stage Three:
• Goal: Fibrin formation
• How? Thrombin and Ca convert Fibrinogen into Fibrin,
Salah Lashin ! • Pathology: Pancreatitis will destroy fibrin (activation of
0991907360 fibrinolysis)
A patient presents with a pinpoint pupil that does not dilate,
when the lighting changes. Where was the central nervous
system damaged?
A. Lateral geniculate body in the diencephalon
B. Lateral horn of the spinal cord's gray matter at the level of
C8-Th1
C. Pes pedunculi at the level of the inferior colliculi in the
midbrain tectum
D. Cerebral peduncles at the level of the superior colliculi in
the midbrain tectum
E. Superior colliculi in the midbrain tectum

Salah Lashin !
0991907360
84 . An older woman has broken her arm four times
already. What substance makes bones more fragile, if its
concentration is increased?
A. Extracellular fluid
B. Water
C. Organic substances
D. Binding substance
E. Nonorganic substances

Salah Lashin ! Need confermation!


0991907360
Digestive system %
85. A patient was referred for duodenal endoscopy. Stomach
1. Epithelium: Columnar glandular
It detected an inflammation of the major duodenal
2. Parietal cells - Body:
papilla and disturbed bile secretion in the intestinal 1. Histology: contains numerous canliculi and mitocondrial for regulating the
lumen. In what part of the duodenum were these work according to income of food and Histamine.
disturbances detected? 2. Function:
1. HCL (acid): increased activity causes heart burn and activates
A. Descending pepsinogen into pepsin.
B. Superior 2. Intrinsic factor / Gasteromucoprotein: help in B12 absrobtion.
C. Horizontal 3. Chief cells: pepsinogen
4. G cells –Antrum / pylorus:
D. Duodenal bulb
• Gastrin: Activates Parietal cells.
E. Ascending • Anteroctomy or removel of pylurus part will cause issuse
5. Cervical mucocytes: Regeneration of gastric epithelium.
Dodenum
6. S cells:
• Secretin: When HCL comes to duodenum, it stimulates pancreas to secrete
water and bicarbonate to neutralize the pH, so intestine epithelium wouldn’t
be damaged.
7. I cells:
• Cholecystokinin / pancreozymin:
1. Stimulate the Pancreas → Pancreatic enzymes (juice)
2. Stimulate the gallbladder → bile acid.
8. Tubuloalveolar glands (Brunner gland): Secretes bicarbonate (alkaline).
9. Endocrine cells: Present in basal part of the duodenum.
10. Paneth cells: Antibacterial Function (ban bacteria)
11. Descendening part: single longitudinal fold + Major papilla
Salah Lashin ! Ileum
1. (Payer's patches).
0991907360 2. Meckel’s diverticulum.
Hemolysis

Indiruct / unconjugated
bilirubin

= Choledochus
= Bile excreting

Decending

Salah Lashin ! = Major papilla /


0991907360 Hepato-pancreatic ampulla
A woman with a tumour of the pancreas has developed mechanic C. Ductus hepaticus dexter et ductus cysticus
jaundice due to compression of a bile-excreting duct. Which duct D. Ductus hepaticus communis et ductus cysticus
is compressed?
A. Ductus hepaticus communis
B. Ductus cysticus Due to the blockage of the common bile duct (which was
C. Ductus choledochus radiographically confirmed), the biliary flow to the duodenum was
D. Ductus hepaticus dexter stopped. We should expect the impairment of:
E. Ductus hepaticus sinister A. Carbohydrate hydrolysis
B. Protein absorption.
A 43-year-old patient has acute pancreatitis with concomitant C. Fat emulsification.
disruption of common bile duct patency. What condition can it D. Secretion of hydrochloric acid.
result in? E. Salivation inhibition.
A. Hepatocellular jaundice
B. Hemolytic jaundice
C. Mechanical jaundice During the endoscopy the inflammation of a major papilla of the
D. Hepatic coma duodenum and the disturbances of bile secretion were found. In
E. Portal hypertension which part of duodenum were the problems found?
A. Descendent part
Roentgenologically confirmed an obstruction of common bile duct B. Ascendant part
that prevents bile from inflowing to the duodenum. What process C. Bulb
is likely to be disturbed? D. Upper horizontal part
A. Protein absorption E. Lower horizontal part
B. Fat emulgation
C. Carbohydrate hydrolysis Obturative jaundice developed in a 60-year-old patient because of
E. Salivation inhibition malignant tumour of the big papillary of the duodenal. Lumen of
what anatomical structure is squeezed with tumour?
A patient with cholelithiasis fell ill with mechanic jaundice. A. Hepatopancreatic ampulla
Examination revealed that the stone was in the common bile duct. B. Cystic duct
What bile-excreting ducts make up the obturated duct? C. Common hepatic duct
SalahA.Lashin !
Ductus hepaticus communis et ductus choledochus D. Right hepatic duct
0991907360
B. Ductus hepaticus dexter et sinister E. Left hepatic duct
A woman with a tumour of the pancreas has developed mechanic C. Ductus hepaticus dexter et ductus cysticus
jaundice due to compression of a bile-excreting duct. Which duct D. Ductus hepaticus communis et ductus cysticus
is compressed?
A. Ductus hepaticus communis
B. Ductus cysticus Due to the blockage of the common bile duct (which was
C. Ductus choledochus radiographically confirmed), the biliary flow to the duodenum was
D. Ductus hepaticus dexter stopped. We should expect the impairment of:
E. Ductus hepaticus sinister A. Carbohydrate hydrolysis
B. Protein absorption.
A 43-year-old patient has acute pancreatitis with concomitant C. Fat emulsification.
disruption of common bile duct patency. What condition can it D. Secretion of hydrochloric acid.
result in? E. Salivation inhibition.
A. Hepatocellular jaundice
B. Hemolytic jaundice
C. Mechanical jaundice During the endoscopy the inflammation of a major papilla of the
D. Hepatic coma duodenum and the disturbances of bile secretion were found. In
E. Portal hypertension which part of duodenum were the problems found?
A. Descendent part
Roentgenologically confirmed an obstruction of common bile duct B. Ascendant part
that prevents bile from inflowing to the duodenum. What process C. Bulb
is likely to be disturbed? D. Upper horizontal part
A. Protein absorption E. Lower horizontal part
B. Fat emulgation
C. Carbohydrate hydrolysis Obturative jaundice developed in a 60-year-old patient because of
E. Salivation inhibition malignant tumour of the big papillary of the duodenal. Lumen of
what anatomical structure is squeezed with tumour?
A patient with cholelithiasis fell ill with mechanic jaundice. A. Hepatopancreatic ampulla
Examination revealed that the stone was in the common bile duct. B. Cystic duct
What bile-excreting ducts make up the obturated duct? C. Common hepatic duct
A. Salah
Ductus hepaticus !
Lashin communis et ductus choledochus D. Right hepatic duct
B. Ductus hepaticus dexter et sinister E. Left hepatic duct
0991907360
86 . Toxic damage of hepatocytes and disturbed Edema
protein synthesis has caused a sharp drop in the (Accumulation of fluid in the interstitial / intracellular space)
patient's plasma albumin levels and markedly low
oncotic pressure. What phenomenon will develop as a 1. Increase in hydrostatic / dynamic pressure: ‫ﺿﻐط اﻟدﻓﻊ‬
1. Mechanism: Pressure on the vessel wall = Increase filtration to
result of these changes?
the interstitial space = edema.
A. Decreased ESR
2. Causes:
B. Edema development 1. Standing for a long time.
C. Increased rate of diuresis 2. Heart failure → Decreased CO → poor blood circulation
D. Decreased diuresis leads to pressure down in the vein →.
E. Decreased blood density 3. Valvular damage: blood acumulates in the veins.
2. Decrease in oncotic pressure: ‫ﺿﻐط اﻟﺳﺣب‬
• Mechanism: No Albumin = No oncotic pressure = edema.
• Causes:
1. Glumerulonephritis / nephrotic syndrome = proteinuria.
2. Diet: no protein intake.
3. Liver issues.
3. Inflammation: histamine = vasodilation = more blood filtration ( ⬆
Capillary permeability) = edema.
4. Increased work of antidiuretic hormones: (ADH and Aldosterone) =
No diuresis = Fluid accumulation = Edema.
5. Combined (mixed): two causes.
Salah Lashin ! Treatment: diuretics like furosemide = loss water = less edema.
0991907360
87. Due to a traumatic brain injury, functionally a woman Epiphysis cerebri / Pineal gland
disturbed pineal gland. What functions will be disturbed in
this woman? presents with
1. Function: Secretes Melatonin, and
A. Heart rate
B . Cardiac cycle
controls the circadian rhythms (sleep
C. Menstrual cycle cycle).
D. Sleep-wake cycle 2. Histology: Contains pinealocytes and
E. Respiratory rate astrocytes.

Salah Lashin !
0991907360
Epiphysis cerebri / Pineal gland:
Function:
Secretes Melatonin, and controls the circadian
A specimen presents an endocrine system organ covered rhythms.
with capsule made of connective tissue. Septa branch off Histology:
from the capsule inwards and divide the organ into lobules. Contains pinealocytes and astrocytes.
Each lobule consists of two cell types: neurosecretory
pinealocytes (polygonal cells with processes) located in the
center and gliacytes (astrocytes) located at the periphery.
What organ is represented in this specimen?
A. Hypothalamus
B. Pituitary gland
C. Epiphysis
D. Thyroid gland
E. Adrenal medulla

Salah Lashin !
0991907360
88. A patient was treating chronic edema syndrome with Diuritics
furosemide. In the process he developed disturbed cation ‫ﻣدرات اﻟﺑول‬
composition of blood plasma. What should be used to correct 1. Uses:
the cation composition? 1. Hypertension
A. Lithium carbonate 2. Toxicity
B. Sodium chloride 3. Edema
C. Sodium bicarbonate 2. General Mechanism of action: Na excretion.
D. Potassium chloride
3. Types:
E. Calcium chloride 1. Loop:
1. Drug: Furosimide (strongest)
2. Use: for cerebral and pulmonary/lung
edema and sever poisoning.
3. Side effect: hypokalemia ( low K),
2. Thiazide:
1. Drug: Hydrochlorothiazide.
2. Side effect: hypokalemia ( low K) and
hypochloremia.
3. K sparing:
1. Drug: spironolactone.
2. MOA: aldosterone atagonist.
3. Increase K work in blood.
Salah Lashin ! 4. Use: help other diuretics + heart failure.
0991907360
89 . During an appointment with a doctor, a patient says
that at his own discretion he takes an antiallergic medicine
that significantly diminishes the signs of allergy, but causes
sleepiness instead. What H1 receptor blocker does the
patient take?
A. Ranitidine
B. Loratadine
C. Tavegyl (Clemastine)
D. Dimedrol (Diphenhydramine) Antihistamine
E. Cromolyn sodium (Disodium cromoglycate) Mast cells blocker:
1. Ketotifen
2. Cromolyn sodium

H1 blockers (allergy):
1. First generation: Dimedrol / Benadryl / (diphenhydramine)
• Use: has effect on the CNS so it’s usefull if the patient
needs to sleep.
2. Second generation: Loratadine:
• Use: No or less CNS effect, so it’s usefull if the patient
needs to stay awake (Train station worker, student).
H2 blockers (to treat ulcer):
Famous Run 2 ulcer !
1. Famotidine
Salah Lashin !
2. Ranitidine
0991907360
90. Laboratory analysis of blood respiratory function
determined that the CO2 transport has worsened. It is likely to
be caused by an enzyme deficiency. What enzyme is deficient in
this case?
A. Protein kinase
B. Adenylate cyclase
C. 2,3-Diphosphoglycerate
D. Phosphorylase
E. Carbonic anhydrase

Salah Lashin !
0991907360
Genetic disease
91. Genealogical analysis of a child with myotonic Nature of the disease
dystrophy determined that this disease is present in every 1. Dominant - ‫ﻣﮭﯾﻣن‬:
generation, occurs equally frequent in the relatives of both • Strong: Always wins when comes with other gene. So, it
genders, the risk of inheriting this disease is equal no appear in all generation ( ‫)ﻣش ﺑﯾﻌﺗق ﺣد‬.
matter which parent is affected. If one of the parents is • Represeted by: Capital letter A.
heterozygous for this disease and the other parent is 2. Recessive - ‫ﻣﺗﻧﺣﻲ‬:
healthy, the risk of them giving birth to a sick child is 50%. • Weak: Always loses when comes with another healthy gene
unless it’s alone.
What type of disease inheritance is it?
• Represeted by: Small letter a.
A. Autosomal recessive Genocopy of the diseases
B. Y-linked 3. Homozygous:
C. Autosomal dominant 1. Unmixed ‫ﺻﺎﻓﻲ‬
D. X-linked recessive 2. Example: aa or AA.
E. X-linked dominant 4. Heterozygous:
1. hybrid , mixed ‫ﻣﺷﻛل‬
2. Genocopy: Aa.
Questions
1. How can a person get infected with a dominant disease?
• If The person have one copy of 2 copies ( ‫)ﻣش ﺑﯾﻌﺗق ﺣد‬.
• Single copy (Heterozygous / Aa).
• Both copies (Homozygous / AA).
2. How can a person get infected with a Recessive disease?
• It’s weak so you have to get 2 diseased copies.
Salah Lashin ! • Single copy (Heterozygous / Aa): only carrier.
0991907360 • Both copies (Homozygous / AA): diseased
Genetic disease
Nature of the disease
1. Dominant - ‫ﻣﮭﯾﻣن‬:
• Strong: Always wins when comes with another gene. So, it
appear in all generation ( ‫)ﻣش ﺑﯾﻌﺗق ﺣد‬.
• Represeted by: Capital letter A.
2. Recessive - ‫ﻣﺗﻧﺣﻲ‬:
• Weak: Always loses when comes with another healthy gene
unless it’s alone.
• Represeted by: Small letter a.
Genocopy of the diseases
3. Homozygous:
1. Unmixed ‫ﺻﺎﻓﻲ‬
2. Example: aa or AA.
4. Heterozygous:
1. hybrid , mixed ‫ﻣﺷﻛل‬
2. Genocopy: Aa.
Questions
1. How can a person get infected with a dominant disease?
• If The person have one copy of the 2 copies ( ‫)ﻣش ﺑﯾﻌﺗق ﺣد‬.
• Single copy (Heterozygous / Aa).
• Both copies (Homozygous / AA).
2. How can a person get infected with a Recessive disease?
• It’s weak so you have to get 2 diseased copies.
Salah Lashin ! • Single copy (Heterozygous / Aa): only carrier.
0991907360 • Both copies (Homozygous / aa): diseased
Salah Lashin !
0991907360
136. A certain syndrome manifests as damage to the teeth, Nature of the disease
hair and bones. Each generation has affected persons. Men 1. Dominant - ‫ﻣﮭﯾﻣن‬:
and women are affected with equal frequency. What is the • Strong: Always wins when comes with other gene. So, it
appear in all generation ( ‫)ﻣش ﺑﯾﻌﺗق ﺣد‬.
type of inheritance of this syndrome?
• Represeted by: Capital letter A.
A. Autosomal dominant 2. Recessive - ‫ﻣﺗﻧﺣﻲ‬:
B. X-linked recessive • Weak: Always loses when comes with another healthy gene
C. Autosomal recessive unless it’s alone.
D. X-linked dominant • Represeted by: Small letter a.
E. Y-linked
3. X-linked disorders – Recessive: Form mother to sons.
Genealogical analysis of a child with myotonic dystrophy 4. X-linked disorders – Dominant: Form mother to sons and
determined that this disease manifests in every generation, daughters.
is in equal measure present in the relatives of both genders,
the risk of inheriting this disease is equal no matter which
parent is affected. If one of the parents is heterozygous for
this disease and the other parent is healthy, the risk of them
giving birth to a sick child is 50%. What type of disease
inheritance is it?
A. Y linked
B. Autosomalrecessive
C. X linked recessive
D. X linked
Salah dominant
Lashin !
E. Autosomal
0991907360 dominant
Genetic disease
Nature of the disease
1. Dominant - ‫ﻣﮭﯾﻣن‬:
• Strong: Always wins when comes with other gene. So, it
appear in all generation ( ‫)ﻣش ﺑﯾﻌﺗق ﺣد‬.
A family of healthy students, who have arrived from • Represeted by: Capital letter A.
Africa, gave birth to a child with signs of anemia. The 2. Recessive - ‫ﻣﺗﻧﺣﻲ‬:
child has died shortly after. Examination has revealed that • Weak: Always loses when comes with another healthy gene
the child’s erythrocytes are abnormally crescent-shaped. unless it’s alone.
The disease is characrerized by autosomal recessive • Represeted by: Small letter a.
Genocopy of the diseases
inheritance. Determine the genotype of the child’s
3. Homozygous:
parents: 1. Unmixed ‫ﺻﺎﻓﻲ‬
A. Аа х аа 2. Example: aa or AA.
B. Аа х АА 4. Heterozygous:
C. АА х АА 1. hybrid , mixed ‫ﻣﺷﻛل‬
D. аа х аа 2. Genocopy: Aa.
E. Аа х Аа Questions
1. How can a person get infected with a dominant disease?
• If The person have one copy of 2 copies ( ‫)ﻣش ﺑﯾﻌﺗق ﺣد‬.
• Single copy (Heterozygous / Aa).
• Both copies (Homozygous / AA).
2. How can a person get infected with a Recessive disease?
• It’s weak so you have to get 2 diseased copies.
Salah
Salah Lashin !
Lashin • Single copy (Heterozygous / Aa): only carrier.
0991907360 • Both copies (Homozygous / AA): diseased
0991907360
92 . A person in a state of rest forsibly makes his own
respirations deep and frequent for 3-4 minutes. How does it
affect the acid-base balance of this person?
A. Mixed acidosis develops
B. Metabolic acidosis develops
C. Respiratory acidosis develops
D. Respiratory alkalosis develops
E. Metabolic alkalosis develops

Salah Lashin ! Check acid base balance video!


0991907360
93 . A patient has been suffering from bronchial Hypersensitivity ‫ﻓرط اﻟﺣﺳﺎﺳﯾﺔ‬
asthma for 15 years. What changes in the patient's
leukogram can be expected in this case? When the normally protective immune system has a harmful effect on
A. Leukopenia the body = ‫ﻏﺷوﻣﯾﮫ ﻣن ﺟﮭﺎز اﻟﻣﻧﺎﻋﺔ‬
B . Basophilia
C. Leukocytosis Type 1 Hypersensitivity
D. Left shift (Allergy/ Anaphlaxsis / Atopic / reagionic/ Immediate)
E. Eosinophilia
1. Definition: an abnormal immunological response to a normal
harmless stimulus (Allergen) e.g, food, pollen, drug or animals.
2. Mechanism:
• Allergen: Stimulates B cells = Secreste IgE = activates
1. Mast cells, basophils = secrets Histamine =
vasodilatation (allergy symptoms)
2. Eosinophils.
3. Later: it took becomes mucoid and fibrinoid swelling
3. Time: Very fast = within minutes
4. Management:
- IV Epinephrin/Adrenaline.
- Then cortisol and antihistamine.
5. Prevention:
Salah Lashin ! • Bezredka Method: Binding to IgE fixed to mast cells.
0991907360 • Desensitization: Intermittent administration of allergen.
94. A 38-year-old woman complains of
constant thirst, frequent urination, low
appetite, and headache. Her urine is
colorless, clear, slightly acidic, without
glucose. Her 24-hour diuresis is up to 12
liters. This condition can be caused by low
levels of a certain hormone. Name this
hormone:
A. Atrial natriuretic factor
B. Vasopressin
C. Noradrenaline
D. Glucagon
E. Insulin

Salah Lashin !
0991907360
Related
A patient complains of acute increase in
diuresis (up to 5-7 liters of urine per 24 hours).
Examination of vasopressin in this patient.
What cells have insufficient secretory activity
in this case?
A. Pituicytes
B. Neurosecretory cells of the hypothalamus
C. Endocrinocytes of the intermediate
pituitary
D. Endocrinocytes of the anterior pituitary
E. Pars tuberalis cells

Salah Lashin !
0991907360
95. A patient with streptococcal infection of the gingiva
was prescribed a drug with B-lactam ring in its structure.
What drug of those listed below belongs to this
pharmacological group?
A. Levomycetin (Chloramphenicol)
B. Streptomycin sulfate
C. Benzylpenicillin
D. Rifampicin
E. Erythromycin

Antibiotics #

Salah Lashin !
0991907360
96. A 38-year-old man with schizophrenia was treated with Terminology
psychoactive drugs for a long time. He complains of Psychosis: ‫ ھوس‬/ ‫دﻣﺎﻏﮫ ﻻﺣﺳﮫ‬
uncoordinated movements, hand tremor, and sleepiness. What
group of drugs can have such an effect?
A. Tranquilizers Drug: Aminazine
B. Adaptogens Use: Antipsychotic like schizophrenia
C. Neuroleptics MOA: Dopamine blocker.
D. Antidepressants Side effect: parkinsonism effect.
E. Psychomotor stimulants

Related
An alcoholic has alcoholic psychosis with evident psychomotor
agitation. What neuroleptic drug should be administered for
emergency aid?
A. Aminazine
B. Diazepam
C. Sodium bromide
Salah Lashin ! D. Reserpine
0991907360 E. Halothane
97. A 24-year-old woman was hospitalized with complaints of
headache, lumbar pain, face edema, and general weakness. One
month ago, she had a case of tonsillitis. On admission into the
hospital her blood pressure is 180/110 mm Hg. In urine: marked
proteinuria, microhematuria, leukocyturia. What type of
hypertension is it?
A. Hypertensive disease
B. Primary hypertension
C. Essential hypertension
D. Endocrine hypertension
E. Renal hypertension

Salah Lashin !
0991907360
What happens when kidneys Sense Low blood pressure?

1. Kindney secrestes Renin =


2. Renin forms angintinsin 2 =
3. Angintinsin 2 =
1. Vasoconstriction = Increase blood pressure . "
2. Activates aldosterone = brings more volume to the blood
( Na and water reabsorbtion) = increase blood pressure ".

This system is called RAAS (Renin-Angiotensin-Aldosterone System)

Salah Lashin !
0991907360
Review
Terminology
Stenosis: narrowing‫ﺗﺿﯾﯾق‬

What happens when kidneys there is atherosclerosis?

1. Atherosclerosis in the renal artery.


2. Less blood supply to kidneys.
3. kidneys thinks there is hypotension +,.
4. Kindney secrestes Renin =
5. Renin forms angintinsin 2 =
6. Angintinsin 2 =
1. Vasoconstriction = Increase blood pressure -.
2. Activates aldosterone = brings more volume to the blood ( Na and
water reabsorbtion) = increase blood pressure -.

This system is called RAAS (Renin-Angiotensin-Aldosterone System)

Salah Lashin !
0991907360
98. To relax the skeletal muscles for repositioning of bone
shards after a femoral fracture, the patient was given a muscle 1. Non-depolarizing muscle relaxant:
relaxant, which resulted in a respiratory arrest. After the patient • Tubocurarine (Curariform)
• Mechanism: blockage of N-cholinergic
was transfused with fresh citrated blood, his breathing restored.
receptors of the synaptic membrane
What muscle relaxant was he administered? • Antidote: Proserin / Neostigmine
A. Tubocurarine chloride (Acetylcholinesterase inhibitor).
B. Pancuronium bromide 2. Depolarizing muscle relaxant:
C. Pipecuronium bromide • Dithylinum (short acting)
D. Dithylin (Suxamethonium) • Mechanism:
E. Atracurium besilate • Over-depolarized N-cholinergic receptors
= Muscle is overstimulated =
• Then, muscle desensitize (muscle
desensitization).
• That’s why it cause muscle contraction at
the beginning.
• Note: Pseudocholinesterase /
butyrylcholinesterase are responsible for
destroying dithylinum. If this enzyme is
deficiney, the patient will have increased
effect of dithylinum and can cause
respiratory arrest
• Antidote: Pseudocholinesterase /
Salah Lashin ! butyrylcholinesterase
0991907360
Muscle relaxants

1. Non-depolarizing muscle relaxant:


• Tubocurarine (Curariform)
• Mechanism: blockage of N-cholinergic
receptors of the synaptic membrane
• Antidote: Proserin / Neostigmine
(Acetylcholinesterase inhibitor).
2. Depolarizing muscle relaxant:
• Dithylinum (short acting)
• Mechanism:
• Over-depolarized N-cholinergic receptors
= Muscle is overstimulated =
• Then, muscle desensitize (muscle
desensitization).
• That’s why it cause muscle contraction at
the beginning.
• Note: Pseudocholinesterase /
butyrylcholinesterase are responsible for
destroying dithylinum. If this enzyme is
deficiney, the patient will have increased
effect of dithylinum and can cause
respiratory arrest
Salah Lashin ! • Antidote: Pseudocholinesterase /
0991907360 butyrylcholinesterase
Related
Curarelike substances (dithylinum) make it impossible for 1. Non-depolarizing muscle relaxant:
skeletal muscles to contract because they block: • Tubocurarine (Curariform)
• Mechanism: blockage of N-cholinergic
A. Membrane conduction of excitement
receptors of the synaptic membrane
B. Central synapses • Antidote: Proserin / Neostigmine
C. Ganglionic synapses (Acetylcholinesterase inhibitor).
D. Neuromuscular synapses 2. Depolarizing muscle relaxant:
E. Proprioceptors • Dithylinum (short acting)
• Mechanism:
A patient with dislocated jaw was given a short-actining muscle • Over-depolarized N-cholinergic receptors
= Muscle is overstimulated =
relaxant by a doctor. Name this drug • Then, muscle desensitize (muscle
A. Cytitonum (Cytisine) desensitization).
B. Papaverine hydrochloride • That’s why it cause muscle contraction at
the beginning.
C. Procaine
• Note: Pseudocholinesterase /
D. Dithylinum(Suxamethonium chloride) butyrylcholinesterase are responsible for
E. Pyridostigmine hydrobromide destroying dithylinum. If this enzyme is
deficiney, the patient will have increased
effect of dithylinum and can cause
respiratory arrest
• Antidote: Pseudocholinesterase /
Salah Lashin ! butyrylcholinesterase
0991907360
Related Muscle relaxants
During an operation a patient got injection of muscle relaxant
dithylinum. Relaxation of skeletal muscles and inhibition of 1. Non-depolarizing muscle relaxant:
• Tubocurarine (Curariform)
respiration lasted two hours. This condition was caused by • Mechanism: blockage of N-cholinergic receptors
absence of the following enzyme in blood serum: of the synaptic membrane
A. Catalase • Antidote: Proserin / Neostigmine
B. Butyrylcholin esterase (Acetylcholinesterase inhibitor).
C. Acetylcholinesterase 2. Depolarizing muscle relaxant:
D. Glucose 6-phosphatase • Dithylinum (short acting)
E. Glutathione peroxidase • Mechanism:
• Over-depolarized N-cholinergic receptors =
Muscle is overstimulated =
The first-aid center has received a victim of a traffic accident • Then, muscle desensitize (muscle
diagnosed with closed displaced fracture of the middle third of desensitization).
the thigh. For repositioning of the bone fragments the patient • That’s why it causes muscle contraction at
received 10ml of 2% dithylinum solution intravenously, which the beginning.
resulted in the prolonged period of apnoea and muscle • Note: Pseudocholinesterase /
relaxation. What enzyme is deficient, resulting in such butyrylcholinesterase are responsible for
pharmacogenetic enzymopathy? destroying dithylinum. If this enzyme is
deficiney, the patient will have increased
A. Glucose 6-phosphate dehydrogenase
effect of dithylinum and can cause
B. N-acetyltransferase respiratory arrest
C. Pseudocholinesterase • Antidote: Pseudocholinesterase /
D. Salah Lashin ! reductase
Methaemoglobin butyrylcholinesterase
E. 0991907360
Uridine diphosphate glucanosyltransferase
Muscle relaxants

1. Non-depolarizing muscle relaxant:


Review • Tubocurarine (Curariform)
• Mechanism: blockage of N-cholinergic receptors of the
synaptic membrane
• Antidote: Proserin (Acetylcholinesterase inhibitors)
2. Depolarizing muscle relaxant:

During gastric resection the patient received mixed


anesthesia with tubocurarine chloride muscle relaxant.
To restore unassisted respiration in the patient, the
patient was given proserin. What pharmacological
group does this drug belong to?
A. Muscarinic agonists
B. Cholinesterase inhibitors
C. Angiotensin-converting-enzyme inhibitors
D. Calcium channel blockers
E. Muscarinic antagonists

Salah Lashin !
0991907360
Anthrax
1. Vaccine: STI live
2. Test:
1. Ascoli’s thermo precipitation.
2. Anti-anthrax fluorescent serum.

3. Feature:
1. Spore forming = Can stay in the ground for a long time
2. That’s why infection is common with cattle and agricultural workers.
3. Disease: Anthrax
1. Pulmonary anthrax: Pulmonary infarction
2. Cutaneous anthrax: Painless necrotic black ulcer in center surrounded by
edema (eschar)
3. Gastrointestinal anthrax.
4. Haemorrhagic meningitis: Cardinal's cap

Salah Lashin !
0991907360
99. A 40-year-old man, a butcher, died of sepsis. On his right
cheek there is a dense dark-red cone-shaped infiltration, 6 cm in
size, with a black scab in its center. The right half of his face
and neck are markedly swollen and dense. In the infiltration,
microscopy detects a peracute sero-hemorrhagic inflammation.
Epidermis and its underlying layers are necrotic in the center of
the infiltration. What diagnosis was made by the pathologist?
A. Anthrax
B. Plague
C. Phlegmon of the neck Anthrax
1. Vaccine: STI live
D. Furuncle
2. Test:
E. Tularemia has 1. Ascoli’s thermo precipitation.
2. Anti-anthrax fluorescent serum.

3. Feature:
1. Spore forming = Can stay in the ground for a long time
2. That’s why infection is common with cattle and agricultural
workers.
3. Disease: Anthrax
1. Pulmonary anthrax: Pulmonary infarction
2. Cutaneous anthrax: Painless necrotic black ulcer in
center surrounded by edema (eschar)
Salah Lashin ! 3. Gastrointestinal anthrax.
0991907360 4. Haemorrhagic meningitis: Cardinal's cap
100. A 28-year-old woman has seropurulent conjunctivitis and
complains of painful urination. A conjunctival scrape was
obtained for analysis. Microscopy shows inclusions in the
cytoplasm of the epithelial cells. Inoculation of the pathologic Chlamydia
material onto nutrient media produced no growth. However, • Features:
inoculation of chicken embryos by the yolk sac route allowed • lacks muramic acid
obtaining the culture of the causative agent. What • Intracellular.
microorganism is the most likely causative agent? • Disease:
A. Chlamydia 1. Conjunctivitis
B. Adenovirus 2. Pneumonia
C. Diphtheroid • Transmission:
D. Hemolytic streptococcus 1. Sexual contact
E. Mycoplasma 2. At birth
3. Hand- to-eye contact and flies.

Salah Lashin !
0991907360
Clostridium Family
101. A bacteriological laboratory studied canned 1. Anerobic bacteria ‫ﻻ ھواﺋﯾﺔ‬
meat that caused severe toxicoinfection. 2. Secretes Exotoxins.
Microscopy of the culture grown on Kitt-Tarozzi 3. Hate air (anerobic).
medium shows Gram-positive spore-forming 4. Kitt-Tarozzi method.
bacilli that resemble a tennis racket. What 5. Neutraliztion test.
diagnosis will be made by the doctor? 6. Spore forming.
A. Tularemia 7. Tennis racket
B. Dysentery 8. Transmission: Canned food and honey.
C. Typhoid fever Clostridium botulinum:
D. Clamidiosis 1. Secretes botulinum exotoxin: a neurotoxin which inhibits the
E. Botulism release Acetylcholine =
2. No muscle contraction (flaccid paralysis ‫)ﺷﻠل ارﺗﺧﺎﺋﻲ‬.
1. No swallowing, No movement, No respiration, No eye
accommodation.
3. Note: Botox is also used in cosmetics to remove wrinkles ‫اﻟﺗﺟﻌدات‬.
Clostridium tetani:
1. Secretes tetanus exotoxin: a neurotoxin which inhibits the release
of inhibitory neurotransmitter =
1. No inhibition to the muscles =
2. Muscles spams (Spastic paralysis).
Clostridium perfringens.
Salah Lashin ! 1. Causes: Gas gangrene
0991907360 2. Produces Homolysis in the blood.
102. Hypovitaminosis C causes decreased formation of
organic matrix and disturbs the collagen synthesis, because
this vitamin takes part in the processes of:
A. Proline hydroxylation
B . Lysine carboxylation
C. Arginine hydroxylation
D. -
E. Proline carboxylation

Salah Lashin !
0991907360
Vitamin C (Ascorbic acid)
Function:
1. Hydroxylation of proline and lysine amino acids of
collagen.
2. Absorbtion of iron
Deficieny:
1. Scurvy: Defect in collagen = Bleeding gum +
Subcutaneous hemorrhage
2. Iron deficiency.
3. Note: Vitamin K deficiency can also cause bleeding
but different reason = clotting factor disorders.
Supplement:
• Ascorbic acid + Rutin (increase absorption of vitamin
C).

Salah Lashin !
0991907360
Related

To assess the rate of collagen disintegration during certain connective tissue


disturbances, it is necessary to measure the urine content of the following:
A. Oxyproline
B. Ornithine
C. Proline
D. Lysine
E. Urea

Osteolaterism is characterized by a decrease in collagen strength caused by much


less intensive formation of cross-links in the collagen fibrils. This phenomenon is
caused by hypoactivity of the following enzyme:
A. Lysyl oxidase
B. Monoamino-oxidase
C. Prolyl hydroxylase
D. Lysyl hydroxylase
E. Collagenase

A 34-year-old patient has a history of periodontitis. As a result of increased


collagen degradation, there is a significantly increased urinary excretion of one of
the amino acids. Which one?
A. Hydroxyproline
B. Valine
C. Alanine
D. Glycine
E. Serine
Salah Lashin !
0991907360
Pediatric examination of a 10-year- old child detected numerous
petechiae on the skin, bleeding gums, and low levels of vitamin
C in urine. What process is disturbed in this case?
A. Collagen synthesis
B. Proteoglycan synthesis
C. Proteoglycan breakdown
D. Collagen breakdown
E. Hyaluronidase activation

Hydroxyproline is an important amino acid in the structure of


collagen. This amino acid forms in the result of proline
hydroxylation. What vitamin takes part in the process of proline
hydroxylation?
A. B6
B. B2
C. B1
D. D
E. C

Salah Lashin !
0991907360
103. A 2-year-old child drank eyedrops from the first-aid Muscarinic Agonists drugs
kit. The child's condition is severe: there are excessive
sweating and salivation, asthmatic breathing, cough,
pinpoint pupils, dull heart sounds, and bradycardia. The
child has diarrhea, the intestinal peristalsis is intensified,
the blood pressure is low. What drug has caused the
poisoning?
A. Pilocarpine hydrochloride
B. Atropine
C. Anaprilin (Propranolol)
D. Platyphylline hydrotartrate
E. Sulfacyl-sodium (Sulfacetamide)

Salah Lashin !
0991907360
Organelles ‫ﻣﻛوﻧﺎت اﻟﺧﻠﯾﺔ‬
1. Mitochondria:
104 . The substances are excreted from the • Function: ATP synthesis (by ATP synthase) in electron trasport chain.
cell, when membrane structure of the Golgi • Contains: 2 memberane with cristea.
• Inhertited only from mother to sons and daughters.
apparatus connects to the cell membrane. • Kreans-Sayre: is one of the disease that affect Mitochondria.
The content of this structure is then
2. Granular / rough endoplasmic reticulum:
expelled from the cell. This process is
• Contains the ribosomes which is the place of translation (protein production), for examples:
called: ceruloplasmin / transferrin protein.
A. Facilitated diffusion 3. Golgi complex: Packaging and secretary function by exocytosis attached to the plasmalemma.
B. Exocytosis 4. Ribosomes:
C. Osmosis
1. Procaryotes (bacteria): 30s and 50s subunits
D. Pinocytosis
2. Eucaryotes (humans): 40s and 60s subunits
E. Endocytosis
3. Location: in the Granular / rough endoplasmic reticulum.
4. Function: Translation = protein synthesis = Regeneration.
5. Glycosomes:
1. Location: Granular and Agranular endoplasmic reticulum
2. Function: synthesize glycogen in muscles and liver.
6. Lysosomes:
• Contains: hydrolytic enzymes which are destroying enzymes (bactericidal)
• Location: Present in saliva and mucus.
• Function:
1. Immune function: Killing bacteria (Non-specific immunity).
Salah Lashin ! 2. Heterophagy: The process of digesting extracellular material by lysosomes
0991907360 3. Autophagy: The process of digesting intracellular material by lysosomes
105 . A patient with bronchial asthma developed acute
respiratory insufficiency. What type of respiratory
insufficiency develops in such cases?
A. Restrictive disturbance of alveolar ventilation
B. Obstructive disturbance of alveolar ventilation
C. Deregulatory disturbance of alveolar ventilation
D. Perfusion insufficiency
E. Diffuse insufficiency

Salah Lashin ! Check COPD!


0991907360
106 . Hemoglobin of an adult person (HBA1) is a
tetrameric protein, consisting of two alpha- and two beta-
peptide chains. Such protein structure is called:
A. -
B. Secondary
C. Primary
D. Quaternary Tetrameric protein
E. Tertiary A protein with a quaternary structure of four subunits (tetrameric)

Salah Lashin !
0991907360
Diabetics mellitus
107. A 62-year-old woman developed cataract 1. Eye issues: If glucose acumulates in eyes will cause retinal disorder → glaucoma,
(lenticular opacity) against the background of Cataract and Microangiopathy → blindness.
2. Increased keto acids = Kussmaul’s respiration.
diabetes mellitus. What type of protein modification 3. Polyuria.
occurs in case of diabetic cataract development! 4. Type 1 diabetes: beta cells of pancreas don’t secrete insulin.
A. Glycosylation 5. Has incomplete oxidation
B. Methylation 6. Ketoacidic coma: increased ketone bodies → Aceton smell → Metabolic acidosis
7. Hyperglycaemic / hyperosmolar coma: Glucose ~ 20 micromole/l
C. ADP-ribosylation 8. Deficiency of insulin → proteolysis → aminoacidemia
D. Phosphorilation 9. Renal diabetes: increased glucose levels in urine due to disturbance in glucose
E. Limited proteolysis reabsorption from proximal convoluted tubules.
Tests:
1. Blood sugar: Normal Glucose: 3.3 – 5.5. Diabetes: > 12.
2. Glycated haemoglobin: the test use to detect history of diabetics (≥ 6.5).
3. Glucose tolerance: is measured after fasting glucose test.
Treatment:
1. Insulin (replacment therapy).
1. Use:
1. Type 1 diabetes mellitus.
2. Inhibit the process of ketosis
3. Note: Rapid-acting insulin is used during coma.
2. Side effect: Hypoglycemia = coma (Carbohydrate starvation).
3. Antidote: Glucose or adrenaline.
2. Glibenclamide:
1. Use: type 2
2. Sulphonyl urease derivate
3. Stimulates generation of insulin from beta cells.
Salah Lashin ! 3. Metformin:
1. Use: type 2 (drug of choice).
0991907360
108. In an experiment on lab rats, elcetrical brain
stimulation caused hungry animals to refuse food. What
brain structure was stimulated?
A. Amygdaloid nuclei
B .Ventromedial nucleus of the hypothalamus
C. Hippocampus
D. Lateral hypothalamic area
E. Globus pallidus

Salah Lashin !
0991907360
In an experiment, cerebral neurons of a test animal were
electrostimulated, which resulted in hypophagia (refusal to
take food). Where in the brain were the electrodes placed?
A. Adenohypophysis
B. Hypothalamus
C. Red nucleus
D. Thalamus
E. Neurohypophysis

A 30-year-old driver had a traffic accident, in which he


received a traumatic brain injury. He started complaining of
thirst and increased 24-hour urine output. What part of the brain
was injured in this case?
A. Reticular formation
B. Precentral gyrus
C. Cerebellum
D. Parietal lobe
E. Hypothalamus

Salah Lashin !
0991907360
A 60-year-old patient was diagnosed with hypothalamic
lateral nuclei stroke. What changes in patient’ss behavior
maybe expected?
1.The rejection of food
2.Aggressive behaviour
3.Depression
4.Thirst
5.Unsatisfied hunger

Certain brain structures of test animals have been subjected to


electrostimulation, as an experiment, resulting in
development of polyphagia (abnormal desire to consume
excessive amounts of food) in test animals. Electrodes have
been inserted in the following brain structures:
A. Lateral nuclei of hypothalamus
B. Ventromedial nuclei of hypothalamus
C. Supraoptic nuclei of hypothalamus
D. Adenohypophysis
E. Red nuclei

Salah Lashin !
0991907360
Vegetative abnormalities in the sleep, heat regulation, all
kinds of metabolism, diabetes insipidus are developing in the
patient due to grouth of the tumour in the III ventricle of
brain. Irritation of the nucleus of what part of the brain can
cause this symptoms?
A. Hypothalamus
B. Cerebral peduncles (cruces cerebri)
C. Mesencephalic tegmentum
D. Pons cerebelli
E. Medulla

Person felt thirsty after staying in heat for a long time.


Signals of what receptors caused it first of all?
A. Osmoreceptors of hypothalamus (then it will secrete
vasopressin)
B. Sodium receptors of hypothalamus
C. Osmoreceptors of the liver
D. Glucoreceptors of hypothalamus
E. Baroreceptors of aortic arch

Salah Lashin !
0991907360
109. On examination the patient was determined to have a
strong, balanced, inert type of higher nervous activity
Types of characters:
according to Pavlov's classification. What temperament
1. Sanguine: Stable emotion (can adapt)
according to Hippocrates is it?
2. Phlegmatic: Higher nerves activity + Pavlov’s
A. Melancholic
B . Sanguine
C. -
D. Choleric
E. Phlegmatic

Related
Psychological evaluation determined that a person is able to
quickly adapt to changing situation, has good memory, is
emotionally stable, possesses of high working ability. This
person is the most likely to be:
A. Sanguine
B. Choleric
C. Melancholic
D. Phlegmatic
Salah Lashin ! E. Phlegmatic with melancholic traits.
0991907360
110. On the second year of his life a boy started Cell functions
1. Melanocytes: responsible for producing melanin. ‫ﺻﺑﻐﺔ اﻟﻣﯾﻼﻧﯾن‬
developing frequent respiratory diseases and ulcerative 2. Platelets: forms the clot to prevent hemorrhage.
skin lesions. It was determined that immunoglobulins 3. Pinealocytes: secretes melatonin.
of all classes are practically absent in the child's blood. 4. Piriform / Pyriform : Present in the cerebellum – middle layer.
The described syndrome is based on decreased Leucocytes:
1. Eosinophilia (increase): Allergy/type 1 hypersensitivity (Asthma) & helminthic
functional activity of a certain cell population. Name infection.
this cell population: 2. Schwann cells: glial cells responsible for regeneration of neurons and formation
A. Neutrophils of myelin.
3. Mast cells: Secrete Histamin and heparin = vasodilation (inflammation).
B. T lymphocytes 4. Basophils: Secrete Histamin and heparin = vasodilation (inflammation).
C. Macrophages 5. Antigen-presenting cells: Macrophages, monocytes: presents the antigen on its
D. B lymphocytes MHC to the T-helper cells.
E. NK cells 6. Neutrophilia (increase) : Acute inflammation. (eg, bacterial infection) + Necrosis
+ Pus
7. Monocytosis (increase):
1. Chronic inflammation (eg, inflammatory bowel disease & TB).
2. Main producers of endogenous pyrogen: interleukin 1 (increase
temperature).
8. B - cells/Plasma cells: Secretes antibodies / immunglubuline to fight bacteria
9. Macrophages: phagocytosis of bacteria + Secrete interleukin 1 ( increase
temperature / Fever).
10. T- cells
1. T-killer / Suppressor : Recognize and kill foreign bodies liker new organ.
2. Natural killers cells: protects body from tumor like t killer cells.
3. T-helper: Stimulate B cells and Eosinophils. Contains CD4. Killed by HIV
Salah Lashin ! An#gen virus.

0991907360
111. Gene expression is regulated by various mechanisms.
What DNA segments activate the gene expression, when
induced?
A. Spacer
B. Enhancer Specific Transcription Factors
C. Silencer • Defintion: Regulatroy protein which bind to DNA enhancers or silencer to
D. Terminator regulate gene expression.
E. Attenuator • Note:
• If they bind to the enhancer region of DNA → Activate gene gene
expression (trasncribiton)
• If they bind to the enhancer region of DNA → Inhibits gene gene
(trasncribiton).

Salah Lashin !
0991907360
112. A patient with burn disease developed DIC syndrome
as a complication. What stage of DIC syndrome is likely in
this case, if Lee White clotting time is less than 3 minutes?
A. Hypercoagulation
B. Transient
C. Fibrinolysis
D. Terminal
E. Hypocoagulation

DIC (Disseminated intravascular coagulation)


Pathogenesis:
1. Activation of the coagulation cascade =
2. Formation of hunge amounts of cloting =
1. Deficiency of Plateletes (thrombocytopenia)
2. Deficiency of the clotting factors (decreased fibrinogen
levels).
3. Bleeding = shock = death.
Causes:
1. Sepsis (bacterial infection of blood).
Salah Lashin !
2. Trauma.
0991907360
2 days after labour a woman developed shock along with DIC (Disseminated intravascular coagulation)
DIC syndrome that caused her death. Autopsy revealed Pathogenesis:
purulent endomyometritis, regional purulent lymphangitis, 1. Activation of the coagulation cascade =
lymphadenitis and purulent thrombophlebitis. There were 2. Formation of hunge amounts of cloting =
also dystrophic alterations and interstitial inflammation of 1. Deficiency of Plateletes (thrombocytopenia)
parenchymal organs. What is the most likely diagnosis? 2. Deficiency of the clotting factors (decreased
A. Hydatid mole fibrinogen levels).
B. Syphilis 3. Bleeding = shock = death.
C. Tuberculosis of genital organs Causes:
D. Chorioadenoma destruens 1. Sepsis (bacterial infection of blood).
E. Septicemia 2. Trauma.

A 43-year-old woman against the background of septic shock


developed thrombocytopenia; her blood fibrinogen levels are
low; fibrin degradation products appeared. The patient
developed petechial hemorrhages. What is the cause of these
changes?
A. Exogenous intoxication
B. DIC syndrome
C. Platelet production disorder
D. Autoimmune thrombocytopenia
E. Salah Lashin !diathesis
Hemorrhagic
0991907360
113. During meiosis the process of autosomal cell division
Chromosomal abnormalities
was disturbed, which resulted in formation of an ovum
with an extra 18 chromosome. The ovum was fertilized by
Autosomal chromosome disorders
a normal spermatozoon. The resulting child will have the
1. Edwards’ syndrome: 18 +1
following syndrome:
2. Patau’s syndrome: 13 +1
A. Klinefelter syndrome
3. Down’s syndrome: 21 +1
B. Turner syndrome
4. Cri du chat: 5 arm deletion
C. Edwards syndrome
D. Down syndrome
Sex chromosome disorders
E. Patau syndrome
1. Klinefelter: XXY
2. Superwoman: XXX
3. Turner: 0X

Cytogenetic / Karyotyping
Used to detect any chromosomal defects

Salah Lashin !
0991907360
Digestive system %
114. During a surgery, Meckel's a diverticulum was Stomach
1. Epithelium: Columnar glandular
detected in the patient. Where in the gastrointestinal
2. Parietal cells - Body:
tract is it located? 1. Histology: contains numerous canliculi and mitocondrial for regulating the
A. Ileum work according to income of food and Histamine.
B. Cecum 2. Function:
1. HCL (acid): increased activity causes heart burn and activates
C. Duodenum pepsinogen into pepsin.
D. Jejunum 2. Intrinsic factor / Gasteromucoprotein: help in B12 absrobtion.
E. Sigmoid colon 3. Chief cells: pepsinogen
4. G cells –Antrum / pylorus:
• Gastrin: Activates Parietal cells.
• Anteroctomy or removel of pylurus part will cause issuse
5. Cervical mucocytes: Regeneration of gastric epithelium.
Dodenum
6. S cells:
• Secretin: When HCL comes to duodenum, it stimulates pancreas to secrete
water and bicarbonate to neutralize the pH, so intestine epithelium wouldn’t
be damaged.
7. I cells:
• Cholecystokinin / pancreozymin:
1. Stimulate the Pancreas → Pancreatic enzymes (juice)
2. Stimulate the gallbladder → bile acid.
8. Tubuloalveolar glands (Brunner gland): Secretes bicarbonate (alkaline).
9. Endocrine cells: Present in basal part of the duodenum.
10. Paneth cells: Antibacterial Function (ban bacteria)
11. Descendening part: single longitudinal fold + Major papilla
Salah Lashin ! Ileum
1. (Payer's patches).
0991907360 2. Meckel’s diverticulum.
Blood grouping (ABO)
115. The blood group of a 30-year-old man was determined Gnerenal rule:
before a surgery. His blood is Rh positive. The standard sera for • Doesn’t agglutinate with X, So answer is X.
groups Oαß (I), Aß (II), and Bα (III) caused no erythrocyte • Doesn’t agglutinate with anything, So answer is 0.
1. 0 (I):
agglutination. Therefore, his blood group is:
1. Universal doner.
A. AB (IV) 2. Doesn’t cause agglutination with anything. Neither
B. Bα (III) anti-A or anti-B reagents
C. Aß (II) 2. A (II):
D. Oaß (I) • No agglutination with Serum A (II)
E. – 3. B (III):
• No agglutination with Serum B (III)
4. AB (IV):
• Agglutination reaction is positive with anti-A and
anti-B.

( II ) ( III ) ( IV) (I)

Salah Lashin !
0991907360
Blood grouping (ABO)
Gnerenal rule:
• Doesn’t agglutinate with X, So answer is X.
• Doesn’t agglutinate with anything, So answer is 0.
1. 0 (I):
1. Universal doner.
2. Doesn’t cause agglutination with anything. Neither anti-A or anti-B reagents
( II ) ( III ) ( IV) (I) 2. A (II):
• No agglutination with Serum A (II)
3. B (III):
• No agglutination with Serum B (III)
4. AB (IV):
• Agglutination reaction is positive with anti-A and anti-B.

Rh / D Factor: Agglutination reaction is positive with anti-D.


Rh / D Factor ‫اﻧت ﻋدو ﻣﺎ ﺗﺟﮭل‬
Questions
1. What happens if mother is Rh negative?
• If the mom doesn’t have Rh Factor and the baby does,
• Mom will make antibodies against Rh.
• The first child will survive; however the second child will have hemolysis.

2. how can the kid be Rh+ if mom is Rh –?


• From dad.
3. In which case would you fear about hemolysis in the kid?
• if the mom is Rh - and the dad is Rh +.
Salah Lashin !
0991907360
Blood grouping (ABO)
Related Gnerenal rule:
• Doesn’t agglutinate with X, So answer is X.
During AB0 blood grouping by using zoliclons • Doesn’t agglutinate with anything, So answer is 0.
(diagnostic monoclonal antibodies), hemagglutination 1. 0 (I):
did not occur with any of the zoliclons. What is the 1. Universal doner.
blood group of the patient under examination? 2. Doesn’t cause agglutination with anything. Neither
A. 0(I) anti-A or anti-B reagents
B. A (II) 2. A (II):
C. B (III) • No agglutination with Serum A (II)
D. AB (IV) 3. B (III):
• No agglutination with Serum B (III)
4. AB (IV):
Before a surgery, the blood type of a patient is being • Agglutination reaction is positive with anti-A and
determined according to the ABO system, using anti-B.
monoclonal antibodies against blood group antigens.
Neither anti-A and anti-B reagents caused agglutination. ( II ) ( III ) ( IV) (I)
What blood type is it?
A. A2 (II)
B. AB (IV)
C. A1 (II)
D. 0 (I)
E. B (III)

Salah Lashin !
0991907360
Blood grouping (ABO)
Gnerenal rule:
Determining a patient’s blood group with monoclonal test- • Doesn’t agglutinate with X, So answer is X.
• Doesn’t agglutinate with anything, So answer is 0.
reagents revealed positive agglutination reaction to anti-A
1. 0 (I):
and anti-B reagents, and negative reaction to anti-D. What 1. Universal doner.
blood group does this patient have? 2. Doesn’t cause agglutination with anything. Neither anti-
A. II (А) Rh (+) A or anti-B reagents
B. IV (АВ) Rh (-) 2. A (II):
C. III (В) Rh (-) • No agglutination with Serum A (II)
D. IV (АВ) Rh (+) 3. B (III):
E. I (0) Rh (+) • No agglutination with Serum B (III)
4. AB (IV):
• Agglutination reaction is positive with anti-A and anti-
Blood group of a 30-year-old man has been determined
B.
before a surgery. The blood was Rhesus-positive. • Rh / D Factor: Agglutination reaction is positive with anti-D.
Agglutination did not occur with standard 0 (I), А (II), and
( II ) ( III ) ( IV) (I)
В (III) serums. The blood belongs to the following group:
A. В (III)
B. А (II)
C. 0 (I)
D. АВ (IV)
E. -

Salah Lashin !
0991907360
Blood grouping (ABO)
A woman with the III (В), Rh (-) blood group gave birth Gnerenal rule:
to a child with the II (А) blood group. The child is • Doesn’t agglutinate with X, So answer is X.
• Doesn’t agglutinate with anything, So answer is 0.
diagnosed with hemolytic disease of newborn caused by
1. 0 (I):
rhesus incompatibility. What blood group and Rh does the 1. Universal doner.
father have? 2. Doesn’t cause agglutination with anything. Neither anti-
A. II (А), Rh (+) A or anti-B reagents
B. I (0), Rh (+) 2. A (II):
• No agglutination with Serum A (II)
C. III (B), Rh (+)
3. B (III):
D. I (0), Rh (-) • No agglutination with Serum B (III)
E. II (A), Rh (-) 4. AB (IV):
• Agglutination reaction is positive with anti-A and anti-
A 25-year-old woman at her third pregnancy with B.
• Rh / D Factor: Agglutination reaction is positive with anti-D.
impending miscarriage was brought to the hospital. What
combination of Rh-factor of the mother and the fetus
can be the cause of this condition? ( II ) ( III ) ( IV) (I)
A. Mother Rh (-), fetus Rh (+)
B. Mother Rh (-), fetus Rh (-)
C. Mother Rh (+), fetus Rh (-)
D. Mother Rh (+), fetus Rh (+)
E. -

Salah Lashin !
0991907360
Blood grouping (ABO)
Gnerenal rule:
• Doesn’t agglutinate with X, So answer is X.
• Doesn’t agglutinate with anything, So answer is 0.
The patient's blood group being determined using monoclonal 1. 0 (I):
test reagents. Agglutination reaction is positive with anti-A and 1. Universal doner.
2. Doesn’t cause agglutination with anything. Neither
anti-B reagents. Name the blood group of this patient: anti-A or anti-B reagents
A. AB (IV) 2. A (II):
B. B (III) Rh - • No agglutination with Serum A (II)
C. O (I) Rh + 3. B (III):
• No agglutination with Serum B (III)
D. A (II) Rh +
4. AB (IV):
• Agglutination reaction is positive with anti-A and
ABO blood group is being determined. Erythrocyte agglutination anti-B.
occurred when standard sera of group I and group II were introduced
into the blood being analysed, while group III serum caused no
agglutination. What agglutinogens do these erythrocytes have? ( II ) ( III ) ( IV) (I)
A. B
B. A
C. C
D. A and B
E. D and C

Salah Lashin !
0991907360
116. A young man underwent an IFA test for HIV Serological Tests – indirect antibodies / antigen
antibodies and received a positive result. However, he
insists on a test that would be able to definitively prove the 1. Fluorescence immunoassay: Very Fast + Highly Specific
presence of antibodies to this virus in him. What test + fluenza + cholera
should be conducted to confirm the diagnosis? 2. Enzyme linked immunosorbent assay (ELISA) /
A. Hemagglutination inhibition test Enzymoimmunoassay:
B. Passive hemagglutination test 1. Technique: uses peroxidase or alkaline phosphatase
C. PCR to bound to the antibodies.
D. Immunofluorescence 2. Uses: Hepatitis B.
E. Immunoblotting 3. HIV (and Western blot for confirm)
3. Agglutination reaction: (Antibodies adhere on the
antigen = Adhesion)
1. Wright: Brucellosis agglutination (Blood).
2. Widal: typhoid fever of Salmonella > 1:160.
3. Aglutnin: pertussis (whooping couph).
4. Dysentery: 4 times increase in antibody titer =
Confirmed diagnosis.

Direct: looks for mircro-organism or its DNA


• PCR: DNA Gene amplification used for paternity testing.
Salah Lashin ! • Bacteriological: Pseudotuberculosis and Dysentery.
0991907360
Related
When checking donor blood at the blood
transfusion station, antibodies to human
immunodeficiency virus were found in the
blood serum of one of the donors. What
method is recommended for confirmation of
HIV infection diagnosis?
A. Enzyme linked immunosorbent assay
B. Electron microscopy
C. Western blot (immunoblotting)
D. Immunofluorescence
E. Radioummunoassay

Salah Lashin !
0991907360
117. Histology of a biopsy material obtained from the
Righ ventricular heart failure
liver of a 67-year- old man, who for a long time has
Causes:
been suffering from chronic diffuse obstructive
1. Increased pulmonary pressure. Ex, COPD
emphysema, revealed the following morphological
diseases.
changes: central veins are dilated; sinusoids in the
2. Intense physical activity = too much preload.
center of hepatic lobules are hyperemic and exhibit
3. Tricuspid stenosis.
signs of capillarization; a portion of hepatocytes has
Symptoms:
undergone dystrophic changes; moderate perivascular
• Increase in the preload (venous return) =
sclerosis; periportally one can observe hepatocytes with
venous congestion = edema and cyanosis.
signs of fatty degeneration. What type of liver damage
• Tricuspid insufficiency.
is it?
• Nutmeg liver- in chronic states
A. Fatty hepatosis
• Central hepatocytes are dystrophic and
B. Nutmeg liver
Peripheral are enlarged
C. Brimstone liver
D. Portal cirrhosis
Left ventricular heart failure
E. Goose liver
Causes:
• Increase in Afterload (hypertention).
• Mitral stenosis
Symptoms:
• No blood pumping to the body .
Salah Lashin !
0991907360
Righ ventricular heart failure
Causes:
1. Increased pulmonary pressure. Ex, COPD
diseases.
2. Intense physical activity = too much preload.
3. Tricuspid stenosis.
Symptoms:
• Increase in the preload (venous return) =
venous congestion = edema and cyanosis.
• Tricuspid insufficiency.
• Nutmeg liver- in chronic states
• Central hepatocytes are dystrophic and
Peripheral are enlarged

Left ventricular heart failure


Causes:
• Increase in Afterload (hypertention).
• Mitral stenosis
Symptoms:
• No blood pumping to the body .
Salah Lashin !
0991907360
Related
A 63 year old male patient who had been suffering from chronic
diffuse obstructive disease, pulmonary emphysema, for 15 years Righ ventricular heart failure
died from cardiac insufficiency. Autopsy revealed nutmeg liver Causes:
cirrhosis, cyanotic induration of kidneys and spleen, ascites, 1. Increased pulmonary pressure. Ex, COPD
edemata of lower limbs. These changes of internal organs are diseases.
typical for the following disease: 2. Intense physical activity = too much preload.
A. Chronic right-ventricular insufficiency 3. Tricuspid stenosis.
B. Acute right-ventricular insufficiency Symptoms:
C. Chronic left-ventricular insufficiency • Increase in the preload (venous return) =
D. Acute left-ventricular insufficiency venous congestion = edema and cyanosis.
E. General cardiac insufficiency • Nutmeg liver in chronic states (swelling).
• Tricuspid insufficiency.
A 50-year-old patient suffers from essential hypertension. After
a physical stress he experienced muscle weakness, Left ventricular heart failure
breathlessness, cyanosisf of lips, skin and face. Respiration was Causes:
accompanied by distinctly heard bubbling rales. What • Increase in Afterload (hypertention).
mechanism underlies the development of this syndrome? • Mitral stenosis
A. Acute left-ventricular failure Symptoms:
B. Chronic right-ventricular failure • No blood pumping to the body .
C. Chronic left-ventricular failure
D. Collapse
Salah Lashin !
E. Cardiac tamponade
0991907360
A 63-year-old man, who has been suffering from chronic
diffuse obstructive pulmonary emphysema for 15 years, died
Righ ventricular heart failure
of progressive heart failure. Autopsy shows nutmeg liver
Causes:
cirrhosis, cyanotic induration of kidneys and spleen, ascites,
1. Increased pulmonary pressure. Ex, COPD
and edemas of the lower limbs. What type of heart failure can
diseases.
be characterized by such changes in the internal organs?
2. Intense physical activity = too much preload.
A. Acute global heart failure
3. Tricuspid stenosis.
B. Chronic atrial failure
Symptoms:
C. Chronic heart failure
• Increase in the preload (venous return) =
D. Acute left ventricular failure
venous congestion = edema and cyanosis.
E. Acute right ventricular failure
• Nutmeg liver in chronic states (swelling).
• Tricuspid insufficiency.
A patient who for a long time was suffering from rheumatism
and had mitral stenosis died of cardiopulmonary failure.
Left ventricular heart failure
Autopsy revealed brown induration of the lungs. What
Causes:
circulatory disorder leads to such changes in the lungs?
• Increase in Afterload (hypertention).
A. Chronic left ventricular failure
• Mitral stenosis
B. Chronic right ventricular failure
Symptoms:
C. Acute left ventricular failure
• No blood pumping to the body .
D. Acute right ventricular failure
E. Portal hypertension
Salah Lashin !
0991907360
A patient died with signs of heart failure. Autopsy shows the
following: postinfarction cardiosclerosis, cardiac hypertrophy, and
dilated cardiac chambers. The liver is enlarged, has smooth surface, Righ ventricular heart failure
is plethoric on section, covered in dark red specks against the
Causes:
brown-tinted background of the tissue. Histologically, the central
1. Increased pulmonary pressure. Ex, COPD
veins of the hepatic lobules are plethoric; there is erythrocyte
diapedesis into the perivascular space. Central hepatocytes are
diseases.
dystrophic, while peripheral hepatocytes are enlarged and affected 2. Intense physical activity = too much preload.
by fatty degeneration. What process occurred in the patient's liver? 3. Tricuspid stenosis.
A. Nutmeg liver Symptoms:
B. Pseudonutmeg liver • Increase in the preload (venous return) =
C. Amyloidosis venous congestion = edema and cyanosis.
D. Hepatic cirrhosis • Nutmeg liver in chronic states (swelling).
E. Hepatic steatosis • Tricuspid insufficiency.

Autopsy of a 73-year-old man who had been suffering from the Left ventricular heart failure
coronary heart disease along with cardiac insufficiency for a long Causes:
time revealed: nutmeg liver, brown induration of lungs, cyanotic • Increase in Afterload (hypertention).
induration of kidneys and spleen. What kind of circulation disorder • Mitral stenosis
was the cause of such effects? Symptoms:
A. General chronic venous congestion
• No blood pumping to the body .
B. Arterial hyperaemia
C. General acute venous congestion
D. Acute Lashin !
Salahanaemia
E. Chronic anaemia
0991907360
Related Righ ventricular heart failure
During preparation of a patient to a heart surgery it was Causes:
necessary to measure pressure in heart chambers. In one of them 1. Increased pulmonary pressure. Ex, COPD
pressure varied from 0 mm Hg up to 120 mm Hg within one diseases.
cardiac cycle. What heart chamber is it? 2. Intense physical activity = too much preload.
A. Left ventricle 3. Tricuspid stenosis.
B. Right ventricle Symptoms:
C. Right atrium • Increase in the preload (venous return) =
D. Left atrium venous congestion = edema and cyanosis.
• Nutmeg liver in chronic states (swelling).
Increased aortic blood pressure created an overload of the • Tricuspid insufficiency.
cardiac muscle. In what cardiac structure does the muscle wall
respond to the irritation in this case? Left ventricular heart failure
A. Left atrium Causes:
B. Left ventricle • Increase in Afterload (hypertention).
C. Right ventricle • Mitral stenosis
D. Right atrium Symptoms:
E. Venous sinus • No blood pumping to the body .

Blood pressure
Left ventricle (systematic circulation): 0 to 120 Hg /mm
Salah
RightLashin ! (systematic circulation): 0 to 30 Hg /mm
ventricle
0991907360
Liver cirrhosis

Salah Lashin !
0991907360
Analysis of a punction biopsy material of liver revealed Terminology
hepatocyte dystrophy with necroses as well as sclerosis with Pseudo- lier = ‫ﻛﺎذب‬
disorder of beam and lobulous structure, with formation of
pseudolobules and regenerative nodes. What is the most Liver cirrhosis
probable diagnosis: Chronic Fibrotic liver.
A. Liver cirrhosis 1. Causes:
B. Chronic hepatosis 1. Alchohol, viral hepatitis =
C. Chronic hepatitis 2. Chronic inflammation =
D. Progressive massive liver necrosis 3. Chronic hepatitis.
E. Acute hepatitis 4. Fibrosis of the hepatic tissue.
5. Liver cirrhosis.
On histological examination of biopsy material taken from the 2. Morphology
liver of a woman, who for a long time had been suffering from 1. Regenrative nodules
viral hepatitis type B, the pathologist detected diffuse hepatic 2. pseudolobules
fibrosis with formation of porto-portal and portocentral fibrotic
septa and disturbance of the liver lobular structure (development
of pseudolobules). What process can be characterized by the
given morphological changes?
A. Hepatic cirrhosis
B. Chronic hepatitis
C. Hepatocellular carcinoma
D. Acute hepatitis
Salah Lashin !
E. Cholestasis
0991907360
118. A patient presents with ptosis (drooping eyelid), Muscles
divergent strabismus, disturbed accommodation, and 1. Pupil (‫)ﻓﺗﺣﺔ اﻟﻌﯾن‬:
dilated pupils. It indicates a damage to the nuclei of a 1. Pupil constrictor (sphincter):
certain pair of the cranial nerves. What pair of nerves 1. Function: constrict the pupils. (miosis)
is damaged? 2. Innervation: Oculomotor CN 3
A. III 2. Pupil dilator (Radial):
B. V 1. Function: dilates the pupils. (Mydriasis)
C. IV 2. Innervation: ophthalmic nerve of CN5 (trigeminal).
D. VI 2. Lens crystalline (‫ )اﻟﻌدﺳﺔ‬:
E. VII 1. Ciliary muscle
1. Function: accommodation / adaptation
2. Innervation: Oculomotor CN 3
3. Contains muscarinic receptor, so blocked by Atropine.
3. Extraocular :
• Levator Palpebrae:
• Function: Elevate the upper eyelid.
• Innervation: Oculomotor CN 3

Histology
Retinal layers:
• Outer layer: pigmented epithelium tightly adhere to vascular
layer.
Salah Lashin ! • Contains photoreceptor layers (Rods & Cons)
0991907360
Eye collection

Muscles
1. Pupil (‫)ﻓﺗﺣﺔ اﻟﻌﯾن‬:
1. Pupil constrictor (sphincter):
1. Function: constrict the pupils. (miosis)
2. Innervation: Oculomotor CN 3
2. Pupil dilator (Radial):
1. Function: dilates the pupils. (Mydriasis)
2. Innervation: ophthalmic nerve of CN5 (trigeminal).
2. Lens crystalline (‫ )اﻟﻌدﺳﺔ‬:
1. Ciliary muscle
1. Function: accommodation / adaptation
2. Innervation: Oculomotor CN 3
3. Contains muscarinic receptor, so blocked by Atropine.
3. Extraocular :
• Levator Palpebrae:
• Function: Elevate the upper eyelid.
• Innervation: Oculomotor CN 3

Histology
Retinal layers:
Outer !
Salah• Lashin layer: pigmented epithelium tightly adhere to vascular layer.
• Contains photoreceptor layers (Rods & Cons)
0991907360
Muscles
1. Pupil (‫)ﻓﺗﺣﺔ اﻟﻌﯾن‬:
1. Pupil constrictor (sphincter):
1. Function: constrict the pupils. (miosis)
2. Innervation: Oculomotor CN 3
2. Pupil dilator (Radial):
A 25-year-old woman complains of deteriorating vision. 1. Function: dilates the pupils. (Mydriasis)
Examination revealed a defect in accommodation, the 2. Innervation: ophthalmic nerve of CN5 (trigeminal).
pupil is dilated and unresponsive to light. What muscles 2. Lens crystalline (‫ )اﻟﻌدﺳﺔ‬:
are functionally disturbed in this case? 1. Ciliary muscle
A. Iris sphincter muscle, ciliary muscle 1. Function: accommodation / adaptation
B. Iris dilator muscle, ciliary muscle 2. Innervation: Oculomotor CN 3
C. Superior oblique muscle, ciliary muscle 3. Contains muscarinic receptor, so blocked by Atropine.
D. Lateral rectus muscle, iris sphincter muscle 3. Extraocular :
E. Iris sphincter and iris dilator muscles • Levator Palpebrae:
• Function: Elevate the upper eyelid.
• Innervation: Oculomotor CN 3

Histology
Retinal layers:
• Outer layer: pigmented epithelium tightly adhere to vascular
layer.
Salah Lashin ! • Contains photoreceptor layers (Rods & Cons)
0991907360
A hospital has received a 24-year-old man, who had received a
penetrating wound to the eye, which has caused the
vitreous body to run out. As the result of this, retinal detachment
occurred What retinal layer was tightly adherent to the vascular
tunic of the eye and did not detach?
A. Retinal pigment epithelium
B. Layer of rods and cones
C. Ganglion cell layer
D. Outer nuclear layer
E. Inner nuclear layer

Histologic examination of an eye specimen shows multilayer


structure. The outermost layers represented by special pigment
epithelium which is composed of cuboidal epithelium
containing cells that absorbs light. The photoreceptor layer
contains photo-sensitive outer segments of Rods and cones
which of the following eye structure is mentioned?
A. Retina
B. Ciliary body
C. Sclera
D. Choroid
Salah Lashin !
E. 0991907360
Iris
A woman came to the ophthalmologist with complaints Eye collection
of deteriorating vision. Examination revealed disturbed Muscles
process of accommodation of the eye. What anatomical 1. Pupil (‫)ﻓﺗﺣﺔ اﻟﻌﯾن‬:
structure is functionally disturbed in this patient? 1. Pupil constrictor (sphincter):
A. M. dilatator pupillae 1. Function: constrict the pupils. (miosis)
B. Corpus vitreum 2. Innervation: Oculomotor CN 3
C. M. ciliaris 2. Pupil dilator (Radial):
1. Function: dilates the pupils. (Mydriasis)
D. M.sphincter pupillae
2. Innervation: ophthalmic nerve of CN5 (trigeminal).
E. Lig. pectinatum iridis
2. Lens crystalline (‫ )اﻟﻌدﺳﺔ‬:
1. Ciliary muscle
A patient with an eye trauma was prescribed a substance 1. Function: accommodation / adaptation
that induces a long-term (up to 10 days) relaxation of 2. Innervation: Oculomotor CN 3
the accommodation muscles. Name this substance: 3. Contains muscarinic receptor, so blocked by Atropine.
A. Methacin
B. Pirenzepine
C. Pilocarpine hydrochloride
D. Atropine sulfate
E. Scopolamine hydrobromide

Salah Lashin !
0991907360
Visual pathways

Crossing of image (Visual fields):


1. Medial image wit lateral retinal
2. lateral Image wit Medial retinal

Crossing of optic nerves:


1. Lateral retinal: lateral optic nerve → No decussation
→ Same side tract → Lateral geniculate body.
2. Medial retinal: Nasal / Medial optic nerve →
crossing (decussation) in optic chiasm → opposite
tract → Lateral geniculate body.
Questions
What is the tract of of left side image?
• Right.
What is the tract of of right-side image ?
• Left.

Salah Lashin !
0991907360
A patient demonstrates functional loss of nasal halves of the retinas.
What area of visual pathways is affected?
A. Optic chiasm
B. Left optic tract
C. Right optic tract
D. Left optic nerve
E. Right optic nerve

A patient has disturbed vision in the lateral visual fields of both eyes
(bilateral hemianopsia). What nerve structure is affected?
A. Optic nerves
B. Left optic tract
C. Optic chiasm
D. Right optic tract
E. Retina

A 75-year-old-female patient with complaints of visual impairment has


been delivered to the ophthalmologic department. Objective
examination revealed a brain tumor in area of the left optic tract. The
patient has a visual field defect in the following area:
A. Left half of both eyes retina
B. Right half of both eyes retina
C. Left and right halves of the left eye retina
D.Salah
Left and right!
Lashin halves of the right eye retina
E. 0991907360
Left and right halves of both eyes retina
119. After exacerbation of chronic cholecystitis, the patient
developed acute jaundice. During ECG it was noted that against
the background of normal sinus rhythm (heart rate is 51/min.)
there are periodical extrasystoles. What mechanism is the most
likely cause of disturbed electrical activity of the heart?
A. Bile acid stimulation of the vagal receptors
B. Stimulation of the conductive system by the toxins that were
not neutralized in the liver
C. Bile acid stimulation of the sinus node
D. Bile acid damage to the myocardium
E. Bile acid damage to the sinus node

Related
A 44-year-old patient with obstructive jaundice has been
admitted to a hospital with the symptoms of cholemic
syndrome. On the ECG arrhythmia shows up. What kind of
arrhythmia is the patient most likely to have?
A. Sinus bradycardia
B. Sinus tachycardia
C. Atrial premature contraction
Salah Lashin ! D. Ventricular premature contraction
0991907360 E. Atrioventricular block
120. Acquired immunodeficiencies often are caused by an
infection, where causative agents reproduce directly in the
cells of the immune system, destroying them in the
process. It is characteristic of the following diseases:
A. Q fever, typhus
B. Dysentery, cholera
C. Poliomyelitis, hepatitis A
D. Infectious mononucleosis, AIDS
E. Tuberculosis, mycobacteriosis

Related
Examination of a patient, who for a long time was taking
glucocorticoids, detected lymphopenia. How can the functional
state of the patient!s immune system be characterized?
A. Anaphylaxis
B. Congenital immunodeficiency
C. Autoantigen tolerance
Salah Lashin ! D. Primary immunodeficiency
0991907360 E. Secondary immunodeficiency
Analgetics (painkillers) #
121. A patient with inoperable lung cancer accompanied Narcotics (opioids) $
by unbearable pain was prescribed an analgesic. Against • Work on the opiate / Mu receptor through G inhibitor
the background of analgesic therapy the patient developed protein.
• Antidote (‫)اﻟﻣﺻل‬: Naloxone.
signs of intestinal obstruction. What analgesic could have
• Will cause withdrawal syndrome (‫ )أﻋراض اﻻﻧﺳﺣﺎب‬if you
caused this complication?
stop ‘cessation’ it.
A. Morphine 1. Morphine:
B. Fentanyl 1. Use: Sever pain.
C. Analgin (Metamizole) 2. Side effect:
D. Omnopon (Papaveretum) 1. Respiratory depresstion. (Ventilatory
E. Promedol (Trimeperidine) dysregulation)
2. Chain-Stoke’s respiration.
3. Miosis ‫ﺿﯾق ﺣدﻗﺔ اﻟﻌﯾن‬
4. Ventilatory dysregulation
5. Constipation / Intestinal obstruction ‫اﻣﺳﺎك‬.

2. Promedol (Trimeperidine):
1. Uses:
1. Antispasmodic and urolithiasis ‫ﻣﺿﺎد ﻟﻠﺷد‬
2. Has less addiction (‫ )ادﻣﺎن‬effect so, it’s used
with kids
3. Used cardiogenic shock.
3. Fentanyl:
1. Uses: Sever pain can’t be managed with Morphine
Salah Lashin ! or promedol.
0991907360 2. Droperidol can potentiate it (‫)ﺗﻘوﯾﺔ‬
122. A man is 33 years old. His condition lasts
for 10 years already. Periodically he makes an
appointment with the doctor, complaining of
acute stomachache, seizures, and vision
impairment. Similar signs are observed in the
patient's relatives. Patient's urine is red. He was
hospitalized with diagnosis of porphyria. This
condition can be caused by disturbed synthesis
of: acute intermittent
A. Collagen
B. Bile acids
C. Prostaglandins
D. Heme
E. Insulin

Salah Lashin !
0991907360
Related
A patient, who suffers from congenital erythropoietic
porphyria, has skin photosensitivity. The
accumulation of what compound in the skin cells can
cause it?
A. Uroporphyrinogen 1
B. Protoporphyrin.
C. Uroporphyrinogen 2
D. Coproporphyrinogen 3
E. Heme

A mother consulted a doctor about her 5-year-old


child who develops erythemas, vesicular rash and
skin itch under the influence of sun. Laboratory
studies revealed decreased iron concentration in the
blood serum, increased uroporphyrinogen I excretion
with the urine. What is the most likely inherited
pathology in this child?
A. Erythropoietic porphyria
B. Methemoglobinemia
C. Hepatic porphyria
Salah Lashin !
D. Coproporphyria
E. Intermittent
0991907360 porphyria
Related
A patient, who suffers from congenital erythropoietic
porphyria, has skin photosensitivity. The
accumulation of what compound in the skin cells can
cause it?
A. Uroporphyrinogen 1
B. Protoporphyrin
C. Uroporphyrinogen 2
D. Coproporphyrinogen 3
E. Heme

Patients with erythropoietic porphyria (Gunther’s


disease ) have teeth that fluoresce red on exposure to
ultraviolet light; photosensitive skin; red urine. This
disease is associated with the lack of the following
enzyme:
A. Uroporphyrinogen-III cosynthase
B. Uroporphyrinogen-I synthase
C. Delta-aminolevulinate synthase
D. Uroporphyrinogen decarboxylase
E. Ferrochelatase
Salah Lashin !
0991907360
123. For oxidation of fatty acids, a transport system is
necessary. Such system must include an alkanolamine that
transports fatty acids through the mitochondrial membrane.
Name this compound:
A. Cardiolipin Carnitine
B .Creatinine • Function: Transport of FFA (free fatty acids)
C. Carnitine from cytosol to the mitochondria
D. Carbamoyl phosphate
E. Carnosine

Salah Lashin !
0991907360
A patient with high rate of obesity was advised to use carnitine A 1-year-old child with the symptoms of affection of limb
as a food additive in order to enhance "fat burning". What is the and trunk muscles had been admitted to a hospital.
role of carnitine in the process of fat Examination revealed muscle carnitine deficiency. The
oxidation? biochemical basis of this pathology is a disruption of the
A. Transport of FFA (free fatty acids) from cytosol to the following process:
mitochondria A. Transport of fatty acids to mitochondria
B. Transport of FFA from fat depots to the tissues B. Regulation of Ca2+ level in mitochondria
C. It takes part in one of reactions of FFA beta-oxidation C. Substrate phosphorylation
D. FFA activation D. Utilization of lactic acid
E. Activation of intracellular lipolysis E. Oxidative phosphorylation

A sportsman needs to improve his sporting results. He was


recommended a drug containing carnitine. What process is
activated by this compound in the first place?
A. Transport of glucose
B. Transport of amino acids
C. Transport of calcium ions
D. Transport of fatty acids
E. Transport of vitamin K
Salah Lashin !
0991907360
124. Ultrasound of a pregnant woman functioning shows Fetal circulation
normal of the fetal cardiovascularsystem and ductus 1. Ductus arteriosus / Arterial canal/ Botallo’s duct:
arteriosus. What vessels are connected by the ductus • Function: Connect between: Pulmonary trunk
arteriosus? and aorta
A. Umbilical vein and aorta • Pathology: Patent Ductus arteriosus (Non-
B. Pulmonary trunk and aorta closure after birth)
C. Umbilical vein and umbilical artery 2. Foramen ovale:
D. Pulmonary trunk and superior vena cava • Function: Connect Between the left and right
E. Pulmonary trunk and inferior vena cava atria(Interatrial septum)
• Pathology: Patent foramen ovale.

Salah Lashin !
0991907360
cogenital Heart diseases

Fetal circulation
1. Ductus arteriosus / Arterial canal/ Botallo’s duct:
• Function: Connect between: Pulmonary trunk and aorta
• Pathology: Patent Ductus arteriosus (Non-closure after birth)
2. Foramen ovale:
• Function: Connect Between the left and right atria(Interatrial
septum)
• Pathology: Patent foramen ovale.

Salah Lashin !
0991907360
Ultrasonography of a pregnant revealed no abnormalities in the
cardiovascular system of the fetus, the ductus arteriosus had a Fetal circulation
normal function. What vessels does it connect? 1. Ductus arteriosus / Arterial canal/ Botallo’s duct:
A. Pulmonary trunk and aorta
• Function: Connect between: Pulmonary trunk
B. Pulmonary trunk and superior vena cava
C. Pulmonary trunk and inferior vena cava and aorta
D. Umbilical vein and aorta • Pathology: Patent Ductus arteriosus (Non-
E. Umbilical vein and umbilical artery closure after birth)
2. Foramen ovale:
Ultrasound of a 1.5-year-old child showed a non-union of the • Function: Connect Between the left and right
foramen ovale. Where in the heart is this anatomic structure atria(Interatrial septum)
located? • Pathology: Patent foramen ovale.
A. Interventricular septum
B. Right ventricular wall
C. Left ventricular wall
D. Interatrial septum

A 2-year-old child presents with physical retardation and


frequent pneumonias. The child was diagnosed with non-
closure of the arterial canal. Hemodynamics disturbance in this
case is caused by communication between the:
A. Aorta and pulmonary trunk
B. Pulmonary trunk and pulmonary veins
C. Superior vena cava and aorta
D. Superior vena cava and pulmonary trunk
E. Salah
AortaLashin !
and pulmonary veins
0991907360
Fetal circulation
1. Ductus arteriosus / Arterial canal/ Botallo’s duct:
Examination of a teenager revealed a congenital heart • Function: Connect between: Pulmonary trunk
disease, namely the functioning of Botallo’s duct. In the and aorta
prenatal period of development this duct connects the • Pathology: Patent Ductus arteriosus (Non-
following organs: closure after birth)
A. Pulmonary trunk and aorta 2. Foramen ovale:
B. Right and left ventricle • Function: Connect Between the left and right
C. Aorta and inferior vena cava atria(Interatrial septum)
D. Right and left atrium • Pathology: Patent foramen ovale.
E. Pulmonary trunk and superior vena cava

Examination of a child detected a patent foramen ovale.


Where is this foramen located?
A. Between the right atrium and right ventricle
B. Between the left atrium and left ventricle
D. Between the left and right ventricles
E. In the region of the mitral valve
F. Between the left and right atria

Salah Lashin !
0991907360
125. In response to muscle stretching, its reflex contraction Reflexes
occurs. This reflex reaction begins with stimulation of the
following receptors: 1. Knee-jerk / Ankle-jerk / myotatic reflexes:
A. Golgi tendon organ 1. Function: one muscle contract and the opposite
B. Tactile receptors muscle relax
C. Articular receptors 2. Spinal nerve origin: L2, L3 and L4.
D. Nociceptors 3. Check for
E. Muscle spindles 1. Muscle sensory (Muscle spindles / Strech
receptor (proprioceptor).
2. Conduction of afferent and efferent
neurons.
4. Note: sensation of muscles is called
proprioception and conducted through dorsal
tracts.
2. Diving reflex: Reflex apnea
3. Viscero-motor reflexes:
1. Definition: Between organ and and muscles.
2. Example: peptic ulcer and change in position
4. Viscerovisceral reflex
1. Definition: Between organ and organ.
2. Example: peptic ulcer and heart pain
Salah Lashin ! 5. Aschner reflex: press on the eye to decrease heart rate
0991907360 6. Fatigue reflex: Nerve centres.
126 .A 49-year-old woman had a long history of chronic
glomerulonephritis, of which she died. Autopsy shows that
her kidneys are 7x3x2.5 cm in size, have weight of 65.0 g,
are dense and finely granular. Fibrinous inflammation of
serous and mucous tunics, dystrophic degeneration of
parenchymal organs, and cerebral edema were detected as
well. What complication caused such changes in the serous
tunics and internal organs?
A. Thrombocytopenia
B. Anemia
C. DIC syndrome
D. Sepsis
E. Uremia

Salah Lashin !
0991907360
Corynebacterium Diphthria ‫اﻟﺧﺎﻧق‬
127. A sample of the discharge from the Features:
affected pharyngeal mucosa was obtained • Coryn-: club-like shape / Roman numeral five
from a sick child, provisionally diagnosed • Neisser method
• Volutin granules.
with diphtheria. A smear was prepared and
• Tellurite/Löffler's medium.
stained. Microscopy detected there yellow
• Can be toxogenic or not.
bacilli with dark-blue thickened ends. What • Phage conversion: the process the make it toxic.
structural element of a microbial cell was • Estimation ‫ ﺗﻘدﯾر‬of toxigenic properties to know if it’s toxigenic by
detected in the obtained microorganisms? coagulated equine serum.
A. Flagella Virulence measurement – Mortality:
B. Spores • DLM - (95%) and DLC – (100%), LD50 (50%) and Dl (No disease).
C. Plasmids Test:
D. Volutin granules • Agar gel precipitation test / Precipitation line (not ring)
Symptoms:
E. Capsule
• Edematous tonsils with a grayish film.
• Alterative Endcarditis/ myocarditis (Fatty degenration of the heart)
• In case of asphyxia ‫ = اﺧﺗﻧﺎق‬Need to do Tracheostomy in Omotracheale
triangle.
Treatment / Therapy ‫ﻋﻼج‬:
• Antitoxin.
• Type 3 hypersensitivity. (Serum sickness)
Prevention / Vaccine ‫وﻗﺎﯾﺔ‬:
• Anatoxin (the Diphtheria toxin but treated with formalin).
Check for immunity:
Salah Lashin ! • look for antitoxin titer.
0991907360 • If you have enough antitoxin in the body, there is no need for vaccination.
128. In the 1970s scientists determined that severe cases of Heme
neonatal jaundice are caused by disturbed conjugation of
bilirubin in hepatocytes. What substance is used for
spleen
conjugate formation? Hemolysis
A. Sulfuric acid
B. Uric acid
C. Pyruvic acid
Indirct / Unconjugated Bililrubin
D. Lactic acid
E. Glucuronic acid
Liver (UDP)
Dirct / Conjugated Bililrubin
Bilirubin diglucuronide

Bile duct cholelithiasis

Intes1ne
(strecobiliin)

Bile (Acholic) + No strecobiliin no color (Pale feces)


Salah Lashin !
0991907360
Heme

Terminology
spleen Cholic: bile
Hemolysis Pale: ‫ﺷﺎﺣب‬
Colorless: ‫ﺑدون ﻟون‬
Jaundice: Yellow ‫اﻟﺻﻔرة‬

Indirct / Unconjugated Bililrubin


Jaundice = bilirubin
Increase indirect / Unconjegated bilirubin
Liver (UDP) 1. Hemolytic jaundice
2. Parenchymatous jaundice
Dirct / Conjugated Bililrubin
1. Gilbert’s disease or
Bilirubin diglucuronide
2. Crigler-Najjar syndromes or
3. Newborns =
Bile duct cholelithiasis 1. UDP-glucuronyl transferase deficiency =
2. No transfer of unconjugated into conjugated =
Intestine 3. Increase levels of Unconjugated bilirubin
(strecobiliin)
Increase direct / conjugated bilirubin
3. Mechanical / Obstructive / Compression jaundice
• Indicators: Acholic (steatorrhea) or Pale colorless feces.
Bile (Acholic) + No strecobiliin no color (Pale feces)

Salah Lashin !
0991907360
Laboratory analysis revealed UDP- glucuronyl transferase
deficiency in the patient. What blood values can confirm this A patient came to the doctor with complaints of general
enzymopathy? weakness and sleep disturbances. Objectively the patient’s skin
A. Hyperbilirubinemia is yellow. In blood there is increased concentration of direct
B. Indicanuria bilirubin and bile acids. Acholic stool is observed. What
C. Phenylketonuria condition can be characterized by these changes?
D. Ketoacidosis A. Mechanical jaundice
E. Uremia B. Hemolytic jaundice
C. Parenchymatous jaundice
Detoxification of bilirubin occurs in the membranes of D. Familial nonhemolytic (Gilbert’s) syndrome
endoplasmic reticulum of hepatocytes. Bilirubin is secreted by E. Chronic cholecystitis
hepatocytes into bile for the most part as:
A. Bilirubin diglucuronide
Examination of a chemical plant worker who had had a
B. Unconjugated bilirubin
poisoning revealed an increase in total bilirubin concentration at
C. Bilirubin monoglucuronide
the expense of indirect fraction. Feces and urine are
D. Indirect reacting bilirubin
characterized by high stercobilin concentration. The level of
A 25-year-old patient with a hereditary enzymopathy (Gilbert’s direct bilirubin in blood plasma is normal. What type of
disease) has a disorder of bilirubin conjugation in liver. What jaundice is the case?
enzyme is not synthesized in this patient? A. Hemolytic
A. UDP-glucuronyl transferase B. Obstructive
B. UDP-glucose pyrophosphorylase C. Hepatic
C. UDP-glycogen transferase D. Parenchymatous
Salah Lashin
D. Ornithine !
carbomoyltransferase E. Mechanical
E. Amidophosphoribosyltransferase
0991907360
Heme

Transfusion of Rh-incompatible blood resulted in hemolytic spleen


jaundice development in the patient. What laboratory blood Hemolysis
value confirms this type of jaundice?
A. Accumulation of urobilinogen
B. Reduction of unconjugated bilirubin Indirct / Unconjugated Bililrubin
C. Accumulation of unconjugated bilirubin
D. Reduction of stercobilin
E. Reduction of conjugated bilirubin Liver (UDP)
Dirct / Conjugated Bililrubin
A patient presents with acute attack of cholelithiasis. Bilirubin diglucuronide
Laboratory examination of the patient’s feces will show the
following in this case: Bile duct cholelithiasis
A. Positive reaction to stercobilin
B. Connective tissue Intes1ne
C. Partially digested cellulose (strecobiliin)
D. Negative reaction to stercobilin
E. Starch granules
Bile (Acholic) + No strecobiliin no color (Pale feces)

Salah Lashin !
0991907360
129. A man, who died of internal bleeding
(hemoperitoneum), had a dark red sponge-like node 15x10
cm in size in the subcapsular region of the liver. The node
is clearly separated from the surrounding tissues.
Microscopically, the node tissue consists of large vascular
thin-walled cavities, lined with endotheliocytes and filled
with liquid or coagulated blood. What type of tumor is it?
A. Capillary hemangioma
B. Hemangiopericytoma
C. Venous hemangioma
D. Cavernous hemangioma
E. Lymphangioma

Salah Lashin !
0991907360
130. Hemopoietic stem cells were damaged by radiation Terminology
exposure, which resulted in disturbed formation of certain • -Clast = kils
connective tissue cells. Name these connective tissue cells: • -Blast = builds
A. Adipocytes
B. Macrophages
C. Fibroblasts
D. Osteoblasts
E. Melanocytes

Related Related
In course of an experiment a big number of stem cells Chronic inflammation of gingiva resulted in excessive growth
of red bone marrow was in some way destructed. of connective tissue fibers. What cell elements are leading in the
Regeneration of which cell populations in the loose development of this condition?
connective tissue will be inhibited? A. Fibroblasts
A. Of macrophags B. Osteoblasts
B. Of fibroblasts C. Fibrocytes
C. Of pigment cells D. Macrophages
D.Salah
Of lipocytes
Lashin ! E. Osteoclasts
E.0991907360
Of pericytes
Terminology
• -Clast = kils
Related • -Blast = builds
Chronic inflammation of gingiva resulted in excessive growth of
connective tissue fibers. What cell elements are leading in the
development of this condition? 1. Fibroblast: build and deposites the fibers.
A. Fibroblasts 2. Fibrocytes: The deactivated form of fibroblast.
B. Osteoblasts
C. Fibrocytes
D. Macrophages Mineralization vs demineralization
E. Osteoclasts 1. Mineralization / Calification:
1. Calcitonin
An excessive bone tissue loss is often observed in older people, 2. Alkaline phosphatase
which indicates osteoporosis development. What bone tissue 3. Vitamin D
cells are activated resulting in the development of this disease? 4. Osteoblast
A. Osteoblasts
B. Osteocytes 2. Demineraliztion / Resorption / Decalcification:
C. Tissue basophils 1. PTH
D. Macrophages 2. Acid phosphatase
E. Osteoclasts 3. Osteoclasts
4. Vitamin D.
Salah Lashin !
0991907360
131. A patient with a chemical burn has developed
esophageal stenosis. The patient presents with acute weight
loss due to problematic food intake. In blood test:
erythrocytes - 3.0 1012/L, Hb 106 g/L, total protein - 57
g/L. What type of starvation does this patient suffer from?
Starvation
A. Incomplete starvation
Complete: without food or water. (Earthquake)
B. Absolute starvation
Incomplete: decreased content of protein. (Burn)
C. Water starvation
D. Complete starvation
E. Protein starvation

Salah Lashin !
0991907360
Related
A chemical burn caused esophagus stenosis. Difficulty of
ingestion led to the abrupt loss of weight. In blood: 3, 0 · 1012/l,
Hb - 106 g/l, crude protein - 57 g/l. What type of starvation is it?
A. Incomplete Starva8on
B. Proteinic Complete: without food or water. (Earthquake)
C. Complete Incomplete: decreased content of protein. (Burn)
D. Water
E. Absolute

A victim of an earthquake has been remaining under debris for 7


days without food or water. What type of starvation is it?
A. Complete
B. Complete with continued hydration
C. Quantitative
D. Qualitative
E. Incomplete

Salah Lashin !
0991907360
132. Autopsy of the body of a 52-year old man, who had a TB
long history of tuberculous prostatitis and died of Diagnoisis:
meningoencephalitis, detected a large number of dense 1. Ziehl-Neelsen stain: acid fast bacteria like
gray nodules 0.5-1 mm in diameter in the pia mater at the tuberculosis.
basal and lateral surfaces of the brain, spleen, kidneys, and 2. Mantux test: Allergy skin test contains tuberculin
liver. Histologically, these nodules consist of epithelioid, and check for tuberculin antibodies.
lymphoid, and a small number of giant cells with 1. Use: to detect any antibody immunity for
horseshoe-shaped nuclei located at the periphery of the tuberculosis and the need for BCG vaccine.
cell. These changes indicate: 2. Response: Cellular / Type 4 hypersensitivity
A. Miliary tuberculosis (Mononuclear cells, T-lymphocytes).
B. Macrofocal disseminated tuberculosis
C. Secondary tuberculosis 3. Miliary tuberculosis: a form of tuberculosis that is
D. Peracute tuberculous sepsis characterized by a wide dissemination into the
E. Septicopyemia human body and by a tiny lesions (1–5 mm).

4. Features:
1. Hourseshoe / crescent granuloma.
1. Note: syphilis also causes granuloma but
not horseshoe.

Salah Lashin !
0991907360
Related
A 3-year-old child with meningeal symptoms died. Postmortem
macroscopy of the pia matter revealed miliary nodules which
were microscopically represented by a focus of caseous necrosis
with masses of epithelioid and lymphoid cells with large cells
containing crescent-shaped peripheral nuclei situated between
them. Specify the type of meningitis in the child:
A. Tuberculous
B. Syphilitic
C. Brucellar
D. Grippal
E. Meningococcal

Salah Lashin !
0991907360
133. A patient is suspected to have typhoid fever. For two
weeks no laboratory diagnosis was made. What material
must be sent to the laboratory for bacteriological analysis
during the third week after the onset of the disease?
A. Nasal mucus
B. Pharyngeal mucus
C. Gastric lavage waters
D. Sputum
E. Feces and urine

Salah Lashin !
0991907360
Salmonella
Typhoid fever/enteric fever ‫ﺣﻣﻰ اﻟﺗﯾﻔود‬
1. Disease: Typhoid fever/enteric fever
2. Pathogen: Bacteria Salmonella typhi and paratyphi.
• Colorless/Achromic ‫ﻋﺪﯾﻤﺔ اﻟﻠﻮن‬
• Movable/ motile ‫ﻣﺘﺤﺮﻛﺔ‬
• From: Eggs
• Transmission: food toxicity.
• Widal test titter/ enteric fever titter ≥ 1:160 (less
than that = potential carrier.
• Sample: Blood serum
3. Location: Small intestine - ileum
4. Pathophysiology:
• Fever (non-specific symptoms)
• Necrosis of the Peyer patches of the iluem =
obstruction.
• Zenker's Necrosis
• Maculopapular rash ‫طﻔﺢ ﺟﻠدي واﺳﻊ‬
• Ulceration with bleeding and perforation ‫ﺛﻘب‬
Salah Lashin !
0991907360
134. Lipoic acid was removed from the diet of test animals, Krebs / TCA / Citric acid cycles
which resulted in inhibition of pyruvate dehydrogenase
complex in these animals. What is the function of lipoic 1. Start with the the process of oxidative decarboxylation =
acid in relation to this enzyme? 1. Conversion of pyruvate into Acetyl CoA by the
A. Substrate enzyme:
B. Cofactor 2. Pyruvate dehydrogenase / decarboxylase
C. Inhibitor
D. Allosteric regulator 3. The enzyme requires 5 coenzyme/ cofactor:
E. Product Thymine (B1), FAD (B2), NAD (B3), CoA, Lipoic
acid. (The Lovely Coenzymes For Nerds')

2. Acetyl CoA should meet with oxaloacetate to form citric


acid.
3. Pyruvate carboxylase: produce Oxaloacetate from and to
pyruvate for krebs cycle / gluconeogenesis.
4. FAD (Flavin cofactor) and NAD (niacin cofactor) are the
products of krebs cycle wich carrie the energy to the
electron trasport chain for energy production.
5. Amphibolic: has to pathways: lipid and ATP synthesis.
6. Breaking down of GABA converts to succinate and
Salah Lashin ! enters into Krebs cycle
0991907360
Oxidative decarboxylation of pyruvic acid is catalyzed by a Krebs / TCA / Citric acid cycles
multienzyme complex with several functionally linked
coenzymes. Name this complex: 1. Start with the the process of oxidative decarboxylation =
A. Thymidine diphosphate (TDP), flavin adenine dinucleotide 1. Conversion of pyruvate into Acetyl CoA by the
(FAD), coenzyme A (CoASH), nicotine amide adenine enzyme:
dinucleotide (NAD), lipoic acid 2. Pyruvate dehydrogenase / decarboxylase
B. Flavin adenine dinucleotide (FAD),tetrahydrofolic acid,
pyridoxal-5-phosphate,thymidine diphosphate (TDP), choline 3. This enzyme requires 5 coenzyme to work:
C. Nicotine amide adenine dinucleotide (NAD), pyridoxal-5- Thymine (B1), FAD (B2), NAD (B3), CoA, Lipoic
phosphate, thymidine diphosphate (TDP), methylcobalamin, acid. (The Lovely Coenzymes For Nerds')
biotin
D. Coenzyme A (CoASH), flavin adenine dinucleotide (FAD),
pyridoxal-5- phosphate, tetrahydrofolic acid, carnitine 2. Acetyl CoA should meet with oxaloacetate to form citric
E. Lipoic acid, tetrahydrofolic acid, pyridoxal-5-phosphate, acid.
methylcobalamin
3. Pyruvate carboxylase: produce Oxaloacetate from and to
It has been determined that one of a pesticide components is pyruvate for krebs cycle / gluconeogenesis.
sodium arsenate that blocks lipoic acid. Enzyme activity can be 4. FAD (Flavin cofactor) and NAD (niacin cofactor) are the
impaired by this pesticide. Name this enzyme: products of krebs cycle wich carrie the energy to the
A. Pyruvate dehydrogenase complex electron trasport chain for energy production.
B. Microsomal oxidation 5. Amphibolic: has to pathways: lipid and ATP synthesis.
C. Methemoglobin reductase
D. Glutathione peroxidase
6. Breaking down of GABA converts to succinate and
E. Glutathione !
reductase
Salah Lashin enters into Krebs cycle
0991907360
135. DNA diagnostics-polymerase chain reaction - was Serological Tests – indirect antibodies / antigen
used during the forensic investigation of the Russian royal
family's remains and of the body of Ukrainian journalist 1. Fluorescence immunoassay: Very Fast + Highly
Georgiy Gongadze. Polymerase chain reaction method is Specific + fluenza + cholera
based on: 2. Enzyme linked immunosorbent assay (ELISA) /
A. Nucleotide composition analysis of mRNA Enzymoimmunoassay:
B. Gene amplification 1. Technique: uses peroxidase or alkaline phosphatase
C. Nucleotide composition analysis of tRNA to bound to the antibodies.
D. Nucleotide composition analysis of FRNA 2. Uses: Hepatitis B.
E. Amino acid compositional analysis 3. HIV (and Western blot for confirm)
3. Agglutination reaction: (Antibodies adhere on the
antigen = Adhesion)
1. Wright: Brucellosis agglutination (Blood).
2. Widal: typhoid fever of Salmonella > 1:160.
3. Aglutnin: pertussis (whooping couph).
4. Dysentery: 4 times increase in antibody titer =
Confirmed diagnosis.

Direct: looks for mircro-organism or its DNA


• PCR: DNA Gene amplification used for paternity testing.
Salah Lashin ! • Bacteriological: Pseudotuberculosis and Dysentery.
0991907360
136. Slow filling of the stomach or urinary bladder,
without exceeding the physiological norm, causes no
increased pressure in these organs. This phenomenon is
based on the following ability of the smooth muscles:
A. Excitability
B. Automatism
C. Plasticity
D. Contractility
E. Refractoriness

Salah Lashin !
0991907360
137. Karyotype of a 5-year-old boy has 46 chromosomes. Gene mutation: Change in nucleotides of DNA
One of the chromosomes in pair 15 is longer than it is 1. Inversion:
normal, because a part of a chromosome that belongs in • Turning of the DNA chain degree.
pair 21 is attached to it. What type of mutation is present in 2. Translocations: when chromosomes are broken, and the
this boy? broken part is connected to other chromosomes.
3. Deletion: Deletion of nucleotides.
A. Deletion
4. Insertion: Addition o Nucleotides.
B. Polyploidy
5. Polyploidy: increase in the number of chromosomes.
C. Duplication 1. Trisomy: +1 chromosome.
D. Translocation 6. Monosomy: - 1chromosome.
E. Inversion

Salah Lashin !
0991907360
Gene mutation: Change in nucleotides of DNA
1. Inversion:
• Turning of the DNA chain degree.
2. Translocations: when chromosomes are broken, and the
broken part is connected to other chromosomes.
3. Deletion: Deletion of nucleotides.
4. Insertion: Addition o Nucleotides.
5. Polyploidy: increase in the number of chromosomes.
1. Trisomy: +1 chromosome.
6. Monosomy: - 1chromosome.

Salah Lashin !
0991907360
Related
46 chromosomes were revealed on karyotype examination of the 5
year old girl. One of the 15th pair of chromosomes is longer than
usual due to connected chromosome from the 21 pair. What type
of mutation does this girl have?
A. Translocation
B. Deletion
C. Inversion
D. Insufficiency
E. Duplication

The mother’s karyotype has 45 chromosomes. It was determined


that translocation of chromosome 21 to chromosome 14 had
occurred. What disorder is likely to be observed in the child of
this woman if the father’s karyotype is normal?
A. Patau syndrome
B. Edward’s syndrome
C. Klinefelter syndrome
D. Morris syndrome (androgen insensitivity)
E. Down syndrome

Salah Lashin !
0991907360
Exposure to colchicine resulted in metaphase plate of a human containing 23
chromosomes more than it is normal. Name this mutation:
A. Polyploidy
B. Aneuploidy Gene mutation: Change in nucleotides of
C. Polyteny DNA
D. Inversion 1. Inversion:
E. Translocation • Turning of the DNA chain degree.
2. Translocations: when chromosomes are broken,
A 2-year-old boy is diagnosed with Down syndrome. What chromosomal and the broken part is connected to other
changes can cause this disease? chromosomes.
A. Trisomy 21 3. Deletion: Deletion of nucleotides.
B. Trisomy 13 4. Insertion: Addition o Nucleotides.
C. Trisomy X
5. Polyploidy: increase in the number of
D. Trisomy 18
chromosomes.
E. Monosomy X 1. Trisomy: +1 chromosome.
6. Monosomy: - 1chromosome.
Examination of an 18-year-old girl revealed the following features: ovarian
hypoplasia, broad shoulders, narrow hips, shortening of the lower extremities,
webbed neck. Mental development is normal. The patient has been diagnosed
with Turner’s syndrome. What chromosomal abnormality does this patient
have?
A. Monosomy X
B. Trisomy X
C. Trisomy 13
D. Trisomy 18
Salah Lashin !
E. Nullisomy
0991907360X
A mutation has occurred in a cell in the first exon of the structural gene.
The number of nucleotide pairs changed from 290 to 250. Name this
type of mutation:
A. Deletion
B. Inversion
C. Duplication Gene mutation: Change in nucleotides of
D. Translocation DNA
E. Nullisomy
1. Inversion:
Part of the DNA chain turned about 180 degrees due to gamma radiation. • turning of the DNA chain degree.
What type of mutation took place in the DNA chain? 2. Translocations: when chromosomes are broken,
A. Inversion and the broken part is connected to other
B. Deletion chromosomes.
3. Deletion: deletion of nucleotides.
C. Doubling
4. Insertion: Addition o Nucleotides.
D. Translocation
E. Replication

Due to gamma-radiation a segment of DNA chain rotated 180 C. What


mutation occurred in DNA?
A. Duplication
B. Translocation
C. Inversion
Salah Lashin !
D. Deletion
0991907360
E. Replication
Gene mutation: Change in nucleotides of DNA
1. Inversion:
• Turning of the DNA chain degree.
2. Translocations: when chromosomes are broken, and the
broken part is connected to other chromosomes.
3. Deletion: Deletion of nucleotides.
4. Insertion: Addition o Nucleotides.
5. Polyploidy: increase in the number of chromosomes.
Exposure to colchicine resulted in metaphase plate of a human 1. Trisomy: +1 chromosome.
containing 23 chromosomes more than it is normal. Name this 6. Monosomy: - 1chromosome.
mutation:
A. Polyploidy
B. Aneuploidy
C. Polyteny
D. Inversion
E. Translocation

Salah Lashin !
0991907360
Anti-inflammatory
138. A man with chronic hyperacid gastritis developed Anit- itis "
pain in the joints. Taking into account his concomitant Glucocorticoids (Steroids)
pathology, he was prescribed celecoxib for pain relief. Inhibits Arachidonic acid = No Prostaglandin. No Eeosinophils.
Selective action of this drug towards a certain enzyme Drugs: Prednisone.
Side effects:
ensures that it does not affect gastric mucosa. Name this 1. Decreased ACTH (pituitary thinks there is no need = negative
enzyme: feedback), so, when the patient stops the drug, it will result in
A. Phospholipase C adrenal insufficiency (withdrawal).
B. Cyclooxygenase 2 2. Peptic ulcer
C. Kallikrein 3. Hypokalaemia (Panangin is K supplement )
4. Gluconeogenesis = more sugar = more fat = hypertension.
D. Phospholipase A2
E. Cyclooxygenase 1 Nonsteroidal anti-inflammatory (NSAID)
Inhibits cyclooxygenase (COX-1 and COX-2) = No Prostaglandin

1. Non-selective (both COX-1 and COX-2):


1. Examples: Diclofenac, Ibuprofen and Aspirin, Indomethacin
2. Side effects: Causes stomach ulcers.
1. How? They inhibit Prostaglandin E2 which is good for
stomach. No PGE2 = ulcer.
2. Selective (COX-2):
1. Drug: Celecoxib “Selecoxib” (COX-2).
2. Drug of choice for patients with peptic ulcers.
3. Side effects: Thrombosis.
Antiprostaglandin (NSAID)
Salah Lashin ! 1. Butadion (Phenylbutazone)
0991907360 2. Use: Tropical / local anti-inflammatory
139. Fever development can be characterized by increased
levels of acute-phase proteins: ceruloplasmin, fibrinogen,
C-reactive protein. Specify the most likely mechanism of
this phenomenon:
A. Basophil degranulation in tissues
B. —
C. Interleukin-2-induced proliferation of T lymphocytes
D. Interleukin-1- stimulation of hepatocytes
E. Heat-induced destruction of body cells

Salah Lashin !
0991907360
140. A 25-year-old patient against the background of acute
pneumonia with fever of 40.1°C presents with leukocytosis
of 14.9 g/L with a marked left shift in the leukogram. What
factor directly increases both proliferation and
differentiation of leukocytes in the bone marrow?
A. Prostacyclin
B. Colony-stimulating factor
C . Interleukin-10
D. Tumor necrosis factor
E. Interleukin-1

Salah Lashin !
0991907360
141. A patient has a perforated ulcer on the anterior gastric
wall. What derivatives of the peritoneum will be
contaminated with gastric content in this case?
A. Bursa pregastrica
B. Bursa omentalis
C. Right mesenteric sinus
D. Bursa hepatica
E. Left mesenteric sinus

• Anterior wall of stomach: Bursa pregastrica


• Posterior wall of stomach: Bursa omentalis
through Winslow / epiploic / omental

Salah Lashin !
0991907360
Related
A 40-year-old patient has ulcer perforation in the
posterior wall of stomach. What anatomical
structure will blood and stomach content leak to?
A. Bursa omentalis
B. Bursa praegastrica
C. Right lateral channel (canalis lateralis dexter)
D. Left lateral channel (canalis lateralis sinister)
E. Bursa hepatica

Salah Lashin !
0991907360
Related
A surgeon should reach the omental bursa to perform an
operation on abdominal cavity. How can he reach this part
of peritoneal cavity without affecting the integrity of lesser
omentum?
A. Through the epiploic foramen
B. Through the right paracolic sulcus
C. Through the left paracolic sulcus
D. Through the right mesenteric sinus
E. Through the left mesenteric sinus

Winslow= epiploic = omental

Salah Lashin !
0991907360
142.A 55-year-old man with a many year history of mitral
insufficiency developed acute heart failure. What
pathophysiological type of heart failure can be observed in
this case?
A. Coronarogenic cardiac damage
B. Cardiac volume overload
C. Hypoxic cardiac damage
D. Acute cardiac tamponade
E. Cardiac pressure overload

Salah Lashin !
0991907360
Related
A 49-year-old man has mitral stenosis. What is the leading
mechanism of heart failure in this case?
A. Pressure overload
B. Volume overload
C. Myocardial tension
D. Myocardial injury
E. Fluid overload

An athlete (a long-distance runner) during competition has


developed acute heart failure. This pathology developed
due to:
A. Volume overload
B. Coronary blood flow disturbance
C. Direct damage to myocardium
D. Pericardial pathology
E. Pressure overload

Salah Lashin !
0991907360
143. Eubiotic colicin is used for disease treatment and Normal flora
prevention. This protein suppresses the growth of
Good?
pathogenic microorganisms. Colicin can be synthesized in
1. Protect you from harmful bacteria.
the intestine by nonpathogenic bacilli. What structure 2. Digest some of the food.
codes the ability of a bacterial cell to synthesize colicins? 3. Produce vitamin K.
A. Nucleus 4. NAD + replenishment: when fermenting
B. Ribosome pyruvate into
C. Nucleoid 5. Dysbacteriosis:
D. Mesosome 1. Definition: Decreased in normal flora
E. Plasmid results from prolonged use of wide
spectrum antibiotics.
2. Symptoms: infection, diarrhoea, and loss
of weight.
3. Treatment: Eubiotics
• Note: Eubiotic colicin can be
synthesized from plasmid.
Or bad?
1. Endogenous / Autoinfection infection: it is
when the infection is spread by a normal flora
of the body
2. Examples: Pseudomicelium and staph aurias.
Salah Lashin !
0991907360
144. A child that frequently develops tonsillitis and
pharyngitis presents with enlarged lymph nodes and
spleen. The child looks pale and pastose, with poorly
developed muscle tissue. Lymphocytosis is observed in the
child's blood. What type of diathesis is it?
A. Asthenic diathesis
B. Hemorrhagic diathesis
C. Neuro-arthritic diathesis
D. Lymphohypoplastic diathesis
E. Exudative catarrhal diathesis

Salah Lashin !
0991907360
145. Autopsy of the body of a woman with a history of Pathologic calcification
chronic dysentery includes histology of the internal organs. 1. Metastatic calcification:
Histologically, in the renal and myocardial stroma and 1. Definition: systemic calcium deposits that occur
parenchyma, gastric mucosa, and pulmonary connective in normal tissue, due to increased levels of
tissue there are amorphous violet deposits that produce calcium.
positive Von Kossa reaction. What complication developed 2. Causes: hypercalcemia due to
in this woman? hyperparathyroidism, colon diseases, tumor or
A. Metabolic calcification bone fracture.
B. Amyloidosis 2. Dystrophic calcification:
C. Hyalinosis 1. Definition: local calcium deposits that occur in
D. Dystrophic calcification dead tissue due to previous damage to tissue.
E. Metastatic calcification 2. Causes: tissue necrosis, tooth caries, syphilis.

Related
A 36-year-old patient with a several year history of chronic
pulpitis had undergone tooth extraction. Microscopic
examination of the pulp revealed some deep-purple structureless
areas which can be interpreted as:
A. Dystrophic calcification
B. Metabolic calcification
C. Metastatic calcification
Salah Lashin ! D. Lithiasis
0991907360 E. Dental tartar
Salah Lashin !
0991907360
146. Unlike the nerve cells that do not reproduce, the stem Terminology
cell can regenerate multiple times. The process of repeated 1. Differentiation: the ability of stem cells to produce
cell regeneration is called: new different types of cells. For example, embryo
A. Differentiation mesodermal cells can produce cells of skeletal
B. Apoptosis muscle tissue and dermal layer of skin.
C. Atrophy
D. Hypertrophy 2. Proliferation: the ability of cells to regrate and
E. Proliferation produce new cells of the same origin many times.
For example, proliferating of new cells at the site of
injury.

Related
As a result of expression of some genome components
the embryo cells acquire typical morphological,
biochemical and functional properties. Name this
process:
A. Differentiation
B. Capacitation
C. Reception
Salah Lashin ! D. Determination
0991907360 E. Induction
147. Auscultation shows a pathological noise in the second
intercostal region near the sternum. What valve is likely to Valve auscultation points
be damaged? 1. Pulmonary (Semilunar) valve.
A. Mitral valve • 2 – left
B. Tricuspid valve 2. Aortic (Semilunar) valve
C. Pulmonary valve • 2 – right
D. - 3. Tricuspid valve:
E. Aortic valve • Xiphoid process / 5th (right)
4. Mitral (bicuspid) valve:
• Heart apex – 5th midclavicular (Left)

Order of sounds
1. First sound of the heart S1:
• Closure of atrioventricular valves:
tricuspid and mitral
2. Second sound of the heart S2 :
• Closure of pulmonary and aortic.

3. Third heart sound – pathology :


• Rapid filling

Salah Lashin !
0991907360
Terminology
Auscultation ‫ﺳﻣﺎع‬
A patient on examination presents with prolonged I Valve ‫ﺻﻣﺎم‬
heart sound. This heart sound occurs as the result of:
A. Opening of the tricuspid valve Valve auscultation points
B. Closing of the atrioventricular valves 1. Pulmonary (Semilunar) valve.
• 2 – left
C. Opening of the mitral valve
2. Aortic (Semilunar) valve
D. Closing of the pulmonary valve • 2 – right
E. Closing of the aortic valve 3. Tricuspid valve:
• Xiphoid process / 5th (right)
During examination the doctor performs auscultation to 4. Mitral (bicuspid) valve:
assess the functioning of the patient’s mitral valve. • Heart apex – 5th midclavicular (Left)
Where can the sound of this valve be auscultated?
A. At the apex of the heart Order of sounds
B. At the edge of the sternum over the 5th costal 1. First sound of the heart S1:
cartilage on the right • Closure of atrioventricular valves:
C. At the edge of the sternum in the 2nd intercostal tricuspid and mitral
space on the right 2. Second sound of the heart S2 :
D. At the edge of the sternum in the 2nd intercostal • Closure of pulmonary and aortic.
space on the left
E. At the edge of the sternum over the 5th costal 3. Third heart sound – pathology :
Salah Lashin
cartilage ! left
on the • Rapid filling
0991907360
Heart auscultation revealed diastolic murmur in the II
intercostal space along the right parasternal line. This is the Valve auscultation points
evidence of the following valve pathology: 1. Pulmonary (Semilunar) valve.
A. Aortic valve • 2 – left
B. Bicuspid 2. Aortic (Semilunar) valve
C. Tricuspid • 2 – right
D. Valve of pulmonary trunk 3. Tricuspid valve:
• Xiphoid process / 5th (right)
During examination the doctor performs auscultation to 4. Mitral (bicuspid) valve:
assess the functioning of the patient’s mitral valve. Where • Heart apex – 5th midclavicular (Left)
can the sound of this valve be auscultated?
A. At the apex of the heart Order of sounds
B. At the edge of the sternum over the 5th costal cartilage 1. First sound of the heart S1:
on the right • Closure of atrioventricular valves:
C. At the edge of the sternum in the 2nd intercostal space on tricuspid and mitral
the right 2. Second sound of the heart S2 :
D. At the edge of the sternum in the 2nd intercostal space on • Closure of pulmonary and aortic.
the left
E. At the edge of the sternum over the 5th costal cartilage 3. Third heart sound – pathology :
on the left • Rapid filling
Salah Lashin !
0991907360
During examination the doctor performs auscultation
to assess the functioning of the patient’s mitral valve.
Where can the sound of this valve be auscultated?
Valve auscultation points
A. At the apex of the heart
1. Pulmonary (Semilunar) valve.
B. At the edge of the sternum over the 5th costal • 2 – left
cartilage on the right
2. Aortic (Semilunar) valve
C. At the edge of the sternum in the 2nd intercostal
• 2 – right
space on the right
3. Tricuspid valve:
D. At the edge of the sternum in the 2nd intercostal
• Xiphoid process / 5th (right)
space on the left
4. Mitral (bicuspid) valve:
E. At the edge of the sternum over the 5th costal
• Heart apex – 5th midclavicular (Left)
cartilage on the left
Order of sounds
1. First sound of the heart S1:
• Closure of atrioventricular valves:
Auscultation reveals that in the patient’s II intercostal
tricuspid and mitral
space along the parasternal line on the right the II
2. Second sound of the heart S2 :
heart sound is better heard than the I heart sound.
• Closure of pulmonary and aortic.
What valve produces this sound when closing?
A. Semilunar aortic valve
3. Third heart sound – pathology :
B. Semilunar pulmonary valve
• Rapid filling
C. valve
D.Salah
Tricuspid !
valve
Lashin
E. 0991907360
Bicuspid and tricuspid valves
Auscultation reveals that in the patient’s II intercostal space
along the parasternal line on the right the II heart sound is better Valve auscultation points
heard than the I heart sound. What valve produces this sound
1. Pulmonary (Semilunar) valve.
when closing?
A. Semilunar aortic valve
• 2 – left
B. Semilunar pulmonary valve 2. Aortic (Semilunar) valve
C. Bicuspid valve • 2 – right
D. Tricuspid valve 3. Tricuspid valve:
E. Bicuspid and tricuspid valves • Xiphoid process / 5th (right)
4. Mitral (bicuspid) valve:
Heart auscultation revealed diastolic murmur in the II • Heart apex – 5th midclavicular (Left)
intercostal space along the right parasternal line. This is the
evidence of the following valve pathology:
A. Aortic valve
Order of sounds
B. Bicuspid 1. First sound of the heart S1:
C. Tricuspid • Closure of atrioventricular valves:
D. Valve of pulmonary trunk tricuspid and mitral
2. Second sound of the heart S2 :
A patient during examination presents with prolongation of the • Closure of pulmonary and aortic.
II heart sound. The II heart sound occurs due to:
A. Opening of mitral valve
3. Third heart sound – pathology :
B. Opening of semilunar valve
C. Closure of semilunar valve • Rapid filling
D. Opening of tricuspid
Salah Lashin ! valve
E. Closure of tricuspid valve
0991907360
148. A person with mushroom poisoning. who accidentally ate a
fly agaric, was brought into the inpatient department. Beside
gastric lavage, activated charcoal, enteral administration of salt-
based laxatives, and detoxification infusions the patient was
prescribed atropine sulfate injections, which significantly
reduced the signs of poisoning. Specify the type of interaction
between muscarine (fly agaric alkaloid) and atropine sulfate:
A. -
B. Chemo-physical antagonism (antidotism)
C. Chemical antagonism
D. Direct functional one-way antagonism
E. Indirect functional antagonism

Salah Lashin !
0991907360
149 . A man has suffered a recurrent intramural myocardial
infarction. After a course of treatment and rehabilitation, he
was discharged from the hospital in a satisfactory condition
and registered for regular check-ups with his therapist. Two
years later he died in a car crash. What type of the
pathological process was detected in his myocardium
during the autopsy?
A. Hyperplasia
B. Atrophy
C. Large-focal cardiosclerosis
D. Small-focal cardiosclerosis
E. Necrosis

Salah Lashin !
0991907360
150. The molecules of mature mRNA are the carriers of
genetic information about the sequence, in which certain DNA and RNA:
amino acids must attach to each other. What is coded in the Polynucleotides
mRNA molecules? Proteins:
A. Secondary structure of carbohydrates Amino acids
B. Primary structure of a protein
C. Primary structure of carbohydrates
D. Primary structure of polynucleotides
E. Primary structure of lipids

Salah Lashin !
0991907360
❤ ' ‫اﻟﺣﻣد‬

“It always seems impossible un2l it’s done.”


‫ﻣﻧﺎ اﻟﻌﻣل وﻣن ﷲ اﻟﺗوﻓﯾق‬
"‫"وﻗل اﻋﻣﻠوا‬

Salah Lashin !
0991907360

You might also like